[obm-l] Re: [obm-l] Re: [obm-l] Re: [obm-l] Re: [obm-l] Re: [obm-l] Re: [obm-l] Geometria plana com desigualdade de médias?

2020-08-27 Por tôpico Anderson Torres
Em qua., 26 de ago. de 2020 às 18:29, Pedro José  escreveu:
>
> Boa noite!
> Anderson,
> achei legal a sua visão. Mas não consegui evoluir com nada.
> Todavia, fiquei com uma dúvida. Como x+y é um dos ângulos do triângulo temos 
> a restrição 0 E entendo que tanto para cotg(x) + cot(y) , como para tg(x) + tg(y) ocorrerá 
> um mínimo em x=y=K/2, onde x+y=k,k sendo um constante.
> Não acompanhei a sua dedução d quando um é mínimo o outro é máximo.

Eu não fui muito claro.

Você converteu o problema em "calcule o valor mínimo de cot(x)+cot(y)
com x+y fixo". Isso é essencialmente o mesmo que resolver o problema
"calcule o valor mínimo de tan(a)+tan(b) com a+b fixo" - pois sabendo
resolver um é só usar a mesma solução para x=90-a, y=90-b.

>
> Saudações,
> PJMS
>
> Em qui., 20 de ago. de 2020 às 22:40, Anderson Torres 
>  escreveu:
>>
>> Em qui., 20 de ago. de 2020 às 22:03, Anderson Torres
>>  escreveu:
>> >
>> > Em ter., 18 de ago. de 2020 às 19:51, Pedro José  
>> > escreveu:
>> > >
>> > > Boa noite!
>> > > Cláudio,
>> > > não consegui nada geométrico.
>> > > O máximo que atingi foi:
>> > > a/ha + b/hb + c/hc= [cotg(A1) +cotg (A2)]  + [cotg(B1) +cotg (B2)] + 
>> > > co[tg(C1) +cotg (C2)] com A1 + A2 = A; B1 + B2 + B e C1 + C2 = C.
>> > > Para ser mínimo cada termo entre colchetes deve ser mínimo, o que ocorre 
>> > > quando A1 = A2; B1 = B2 e C1 = C2. Logo P seria o encontro das 
>> > > bissetrizes e logo I.
>> > > Onde: A1= PAB e A2=PAC; B1=PBA e B2=PBC; C1=PCA e C2=PCB.
>> >
>> > Acho que daqui poderia sair uma interpretação mais escamoteada.
>> > Afinal, trigonometria é uma espécie de "ponto de contato" entre a
>> > geometria analítica e a sintética, entre a nuvem de desenhos e a de
>> > números.
>> >
>> > Acredito que a solução aqui seria arranjar uma interpretação
>> > geométrica desses colchetes de co-tangentes. Acredito que possamos
>> > apelar para Ptolomeu em algum momento ou para um macete de
>> > semelhanças, pois as projeções de um ponto sobre duas retas criam um
>> > quadrilátero cíclico.
>>
>> Acrescentando mais coisas: se queremos minimizar cot(x) +cot(y) com
>> x+y fixo, isto é equivalente a minimizar tan(90-x)+tan(90-y) com
>> 90-x+90-y fixo. Ou como maximizar tan(x) + tan(y) com x+y fixo.
>>
>> Geometricamente, tangente é cateto oposto dividido por cateto
>> adjacente. Logo uma soma de tangentes com catetos adjacentes iguais
>> equivale a uma soma de catetos opostos! Assim sendo, nosso problema
>> pode ser pensado da seguinte forma:
>>
>> Dados um ponto A e uma reta d fixos, temos que construir duas retas x
>> e y, com ângulo 'alfa' entre elas, ambas passando por A e tais que a
>> distância entre os pontos X e Y, que elas geram ao intersectar d, seja
>> mínima.
>>
>> Daí fica fácil argumentar que a altura por A também tem que ser a
>> bissetriz por A.
>>
>> No fundo do fundo é uma forma de geometrizar a solução trigonométrica.
>> A trigonometria se torna apenas um atalho.
>>
>> Vou formalizar isso mais tarde, com desenhos e tudo.
>>
>>
>>
>> >
>> > Isso até me lembra o famoso artigo do Shine sobre geometria cearense
>> > VS geometria paulista:
>> > https://cyshine.webs.com/geometria-2005.pdf
>> >
>> >
>> > >
>> > > Saudações,
>> > > PJMS
>> > >
>> > > Em ter., 18 de ago. de 2020 às 11:34, Claudio Buffara 
>> > >  escreveu:
>> > >>
>> > >> Será que tem uma demonstração mais geométrica e menos algébrica disso? 
>> > >> E que torne o resultado mais intuitivo?
>> > >> É razoável que o ponto P não esteja muito próximo de qualquer dos 
>> > >> lados, pois neste caso, se P se aproximasse do lado a, por exemplo, 
>> > >> a/h_a cresceria e a expressão se afastaria do valor mínimo.
>> > >> Mas, com lados não necessariamente congruentes, não é óbvio, a priori, 
>> > >> que P deva ser equidistante dos três.
>> > >> De fato, seria razoável esperar que P estivesse mais próximo do maior 
>> > >> lado e conjecturar, por exemplo, que o P que minimiza a expressão é tal 
>> > >> que a/h_a = b/h_b = c/h_c.
>> > >> O fato de P ser o incentro não me parece a conjectura mais evidente 
>> > >> neste caso.
>> > >>
>> > >>
>> > >> On Sun, Aug 16, 2020 at 10:11 AM Matheus Secco  
>> > >> wrote:
>> > >>>
>> > >>> Olá, Vanderlei.
>> > >>> Por Cauchy-Schwarz, temos
>> > >>>
>> > >>> (a/ha + b/hb + c/hc) * (a*ha + b*hb + c*hc) >= (a+b+c)^2.  (#)
>> > >>>
>> > >>> Como (a*ha + b*hb + c*hc) = 2S, onde S é a área de ABC, segue que a 
>> > >>> expressão a/ha + b/hb + c/hc é pelo menos 2p^2/S, onde p é o 
>> > >>> semi-perimetro.
>> > >>>
>> > >>> Por outro lado, a igualdade em (#) ocorre se, e somente se, ha = hb = 
>> > >>> hc, ou seja, quando P é o incentro do triângulo
>> > >>>
>> > >>> Abraços,
>> > >>> Matheus
>> > >>>
>> > >>> Em dom, 16 de ago de 2020 08:59, Professor Vanderlei Nemitz 
>> > >>>  escreveu:
>> > 
>> >  Bom dia!
>> > 
>> >  Tentei utilizar alguma desigualdade de médias aqui, mas não tive 
>> >  êxito. Alguém ajuda?
>> >  Muito agradecido!
>> > 
>> >  Seja P um ponto no int

[obm-l] Re: [obm-l] Re: [obm-l] Re: [obm-l] Re: [obm-l] Re: [obm-l] Geometria plana com desigualdade de médias?

2020-08-26 Por tôpico Pedro José
Boa noite!
Anderson,
achei legal a sua visão. Mas não consegui evoluir com nada.
Todavia, fiquei com uma dúvida. Como x+y é um dos ângulos do triângulo
temos a restrição 0 escreveu:

> Em qui., 20 de ago. de 2020 às 22:03, Anderson Torres
>  escreveu:
> >
> > Em ter., 18 de ago. de 2020 às 19:51, Pedro José 
> escreveu:
> > >
> > > Boa noite!
> > > Cláudio,
> > > não consegui nada geométrico.
> > > O máximo que atingi foi:
> > > a/ha + b/hb + c/hc= [cotg(A1) +cotg (A2)]  + [cotg(B1) +cotg (B2)] +
> co[tg(C1) +cotg (C2)] com A1 + A2 = A; B1 + B2 + B e C1 + C2 = C.
> > > Para ser mínimo cada termo entre colchetes deve ser mínimo, o que
> ocorre quando A1 = A2; B1 = B2 e C1 = C2. Logo P seria o encontro das
> bissetrizes e logo I.
> > > Onde: A1= PAB e A2=PAC; B1=PBA e B2=PBC; C1=PCA e C2=PCB.
> >
> > Acho que daqui poderia sair uma interpretação mais escamoteada.
> > Afinal, trigonometria é uma espécie de "ponto de contato" entre a
> > geometria analítica e a sintética, entre a nuvem de desenhos e a de
> > números.
> >
> > Acredito que a solução aqui seria arranjar uma interpretação
> > geométrica desses colchetes de co-tangentes. Acredito que possamos
> > apelar para Ptolomeu em algum momento ou para um macete de
> > semelhanças, pois as projeções de um ponto sobre duas retas criam um
> > quadrilátero cíclico.
>
> Acrescentando mais coisas: se queremos minimizar cot(x) +cot(y) com
> x+y fixo, isto é equivalente a minimizar tan(90-x)+tan(90-y) com
> 90-x+90-y fixo. Ou como maximizar tan(x) + tan(y) com x+y fixo.
>
> Geometricamente, tangente é cateto oposto dividido por cateto
> adjacente. Logo uma soma de tangentes com catetos adjacentes iguais
> equivale a uma soma de catetos opostos! Assim sendo, nosso problema
> pode ser pensado da seguinte forma:
>
> Dados um ponto A e uma reta d fixos, temos que construir duas retas x
> e y, com ângulo 'alfa' entre elas, ambas passando por A e tais que a
> distância entre os pontos X e Y, que elas geram ao intersectar d, seja
> mínima.
>
> Daí fica fácil argumentar que a altura por A também tem que ser a
> bissetriz por A.
>
> No fundo do fundo é uma forma de geometrizar a solução trigonométrica.
> A trigonometria se torna apenas um atalho.
>
> Vou formalizar isso mais tarde, com desenhos e tudo.
>
>
>
> >
> > Isso até me lembra o famoso artigo do Shine sobre geometria cearense
> > VS geometria paulista:
> > https://cyshine.webs.com/geometria-2005.pdf
> >
> >
> > >
> > > Saudações,
> > > PJMS
> > >
> > > Em ter., 18 de ago. de 2020 às 11:34, Claudio Buffara <
> claudio.buff...@gmail.com> escreveu:
> > >>
> > >> Será que tem uma demonstração mais geométrica e menos algébrica
> disso? E que torne o resultado mais intuitivo?
> > >> É razoável que o ponto P não esteja muito próximo de qualquer dos
> lados, pois neste caso, se P se aproximasse do lado a, por exemplo, a/h_a
> cresceria e a expressão se afastaria do valor mínimo.
> > >> Mas, com lados não necessariamente congruentes, não é óbvio, a
> priori, que P deva ser equidistante dos três.
> > >> De fato, seria razoável esperar que P estivesse mais próximo do maior
> lado e conjecturar, por exemplo, que o P que minimiza a expressão é tal que
> a/h_a = b/h_b = c/h_c.
> > >> O fato de P ser o incentro não me parece a conjectura mais evidente
> neste caso.
> > >>
> > >>
> > >> On Sun, Aug 16, 2020 at 10:11 AM Matheus Secco <
> matheusse...@gmail.com> wrote:
> > >>>
> > >>> Olá, Vanderlei.
> > >>> Por Cauchy-Schwarz, temos
> > >>>
> > >>> (a/ha + b/hb + c/hc) * (a*ha + b*hb + c*hc) >= (a+b+c)^2.  (#)
> > >>>
> > >>> Como (a*ha + b*hb + c*hc) = 2S, onde S é a área de ABC, segue que a
> expressão a/ha + b/hb + c/hc é pelo menos 2p^2/S, onde p é o semi-perimetro.
> > >>>
> > >>> Por outro lado, a igualdade em (#) ocorre se, e somente se, ha = hb
> = hc, ou seja, quando P é o incentro do triângulo
> > >>>
> > >>> Abraços,
> > >>> Matheus
> > >>>
> > >>> Em dom, 16 de ago de 2020 08:59, Professor Vanderlei Nemitz <
> vanderma...@gmail.com> escreveu:
> > 
> >  Bom dia!
> > 
> >  Tentei utilizar alguma desigualdade de médias aqui, mas não tive
> êxito. Alguém ajuda?
> >  Muito agradecido!
> > 
> >  Seja P um ponto no interior de um triângulo e sejam ha, hb e hc as
> distâncias de P aos lados a, b e c, respectivamente. Mostre que o valor
> mínimo de (a/ha) + (b/hb) + (c/hc) ocorre quando P é o incentivo do
> triângulo ABC.
> > 
> >  --
> >  Esta mensagem foi verificada pelo sistema de antivírus e
> >  acredita-se estar livre de perigo.
> > >>>
> > >>>
> > >>> --
> > >>> Esta mensagem foi verificada pelo sistema de antivírus e
> > >>> acredita-se estar livre de perigo.
> > >>
> > >>
> > >> --
> > >> Esta mensagem foi verificada pelo sistema de antivírus e
> > >> acredita-se estar livre de perigo.
> > >
> > >
> > > --
> > > Esta mensagem foi verificada pelo sistema de antivírus e
> > > acredita-se estar livre de perigo.
>
> --
> Esta mensagem foi verificada pelo sistema de antivírus e
>  ac

[obm-l] Re: [obm-l] Re: [obm-l] Re: [obm-l] Re: [obm-l] Geometria plana com desigualdade de médias?

2020-08-20 Por tôpico Anderson Torres
Em qui., 20 de ago. de 2020 às 22:03, Anderson Torres
 escreveu:
>
> Em ter., 18 de ago. de 2020 às 19:51, Pedro José  
> escreveu:
> >
> > Boa noite!
> > Cláudio,
> > não consegui nada geométrico.
> > O máximo que atingi foi:
> > a/ha + b/hb + c/hc= [cotg(A1) +cotg (A2)]  + [cotg(B1) +cotg (B2)] + 
> > co[tg(C1) +cotg (C2)] com A1 + A2 = A; B1 + B2 + B e C1 + C2 = C.
> > Para ser mínimo cada termo entre colchetes deve ser mínimo, o que ocorre 
> > quando A1 = A2; B1 = B2 e C1 = C2. Logo P seria o encontro das bissetrizes 
> > e logo I.
> > Onde: A1= PAB e A2=PAC; B1=PBA e B2=PBC; C1=PCA e C2=PCB.
>
> Acho que daqui poderia sair uma interpretação mais escamoteada.
> Afinal, trigonometria é uma espécie de "ponto de contato" entre a
> geometria analítica e a sintética, entre a nuvem de desenhos e a de
> números.
>
> Acredito que a solução aqui seria arranjar uma interpretação
> geométrica desses colchetes de co-tangentes. Acredito que possamos
> apelar para Ptolomeu em algum momento ou para um macete de
> semelhanças, pois as projeções de um ponto sobre duas retas criam um
> quadrilátero cíclico.

Acrescentando mais coisas: se queremos minimizar cot(x) +cot(y) com
x+y fixo, isto é equivalente a minimizar tan(90-x)+tan(90-y) com
90-x+90-y fixo. Ou como maximizar tan(x) + tan(y) com x+y fixo.

Geometricamente, tangente é cateto oposto dividido por cateto
adjacente. Logo uma soma de tangentes com catetos adjacentes iguais
equivale a uma soma de catetos opostos! Assim sendo, nosso problema
pode ser pensado da seguinte forma:

Dados um ponto A e uma reta d fixos, temos que construir duas retas x
e y, com ângulo 'alfa' entre elas, ambas passando por A e tais que a
distância entre os pontos X e Y, que elas geram ao intersectar d, seja
mínima.

Daí fica fácil argumentar que a altura por A também tem que ser a
bissetriz por A.

No fundo do fundo é uma forma de geometrizar a solução trigonométrica.
A trigonometria se torna apenas um atalho.

Vou formalizar isso mais tarde, com desenhos e tudo.



>
> Isso até me lembra o famoso artigo do Shine sobre geometria cearense
> VS geometria paulista:
> https://cyshine.webs.com/geometria-2005.pdf
>
>
> >
> > Saudações,
> > PJMS
> >
> > Em ter., 18 de ago. de 2020 às 11:34, Claudio Buffara 
> >  escreveu:
> >>
> >> Será que tem uma demonstração mais geométrica e menos algébrica disso? E 
> >> que torne o resultado mais intuitivo?
> >> É razoável que o ponto P não esteja muito próximo de qualquer dos lados, 
> >> pois neste caso, se P se aproximasse do lado a, por exemplo, a/h_a 
> >> cresceria e a expressão se afastaria do valor mínimo.
> >> Mas, com lados não necessariamente congruentes, não é óbvio, a priori, que 
> >> P deva ser equidistante dos três.
> >> De fato, seria razoável esperar que P estivesse mais próximo do maior lado 
> >> e conjecturar, por exemplo, que o P que minimiza a expressão é tal que 
> >> a/h_a = b/h_b = c/h_c.
> >> O fato de P ser o incentro não me parece a conjectura mais evidente neste 
> >> caso.
> >>
> >>
> >> On Sun, Aug 16, 2020 at 10:11 AM Matheus Secco  
> >> wrote:
> >>>
> >>> Olá, Vanderlei.
> >>> Por Cauchy-Schwarz, temos
> >>>
> >>> (a/ha + b/hb + c/hc) * (a*ha + b*hb + c*hc) >= (a+b+c)^2.  (#)
> >>>
> >>> Como (a*ha + b*hb + c*hc) = 2S, onde S é a área de ABC, segue que a 
> >>> expressão a/ha + b/hb + c/hc é pelo menos 2p^2/S, onde p é o 
> >>> semi-perimetro.
> >>>
> >>> Por outro lado, a igualdade em (#) ocorre se, e somente se, ha = hb = hc, 
> >>> ou seja, quando P é o incentro do triângulo
> >>>
> >>> Abraços,
> >>> Matheus
> >>>
> >>> Em dom, 16 de ago de 2020 08:59, Professor Vanderlei Nemitz 
> >>>  escreveu:
> 
>  Bom dia!
> 
>  Tentei utilizar alguma desigualdade de médias aqui, mas não tive êxito. 
>  Alguém ajuda?
>  Muito agradecido!
> 
>  Seja P um ponto no interior de um triângulo e sejam ha, hb e hc as 
>  distâncias de P aos lados a, b e c, respectivamente. Mostre que o valor 
>  mínimo de (a/ha) + (b/hb) + (c/hc) ocorre quando P é o incentivo do 
>  triângulo ABC.
> 
>  --
>  Esta mensagem foi verificada pelo sistema de antivírus e
>  acredita-se estar livre de perigo.
> >>>
> >>>
> >>> --
> >>> Esta mensagem foi verificada pelo sistema de antivírus e
> >>> acredita-se estar livre de perigo.
> >>
> >>
> >> --
> >> Esta mensagem foi verificada pelo sistema de antivírus e
> >> acredita-se estar livre de perigo.
> >
> >
> > --
> > Esta mensagem foi verificada pelo sistema de antivírus e
> > acredita-se estar livre de perigo.

-- 
Esta mensagem foi verificada pelo sistema de antiv�rus e
 acredita-se estar livre de perigo.


=
Instru��es para entrar na lista, sair da lista e usar a lista em
http://www.mat.puc-rio.br/~obmlistas/obm-l.html
=


[obm-l] Re: [obm-l] Re: [obm-l] Re: [obm-l] Re: [obm-l] Geometria plana com desigualdade de médias?

2020-08-20 Por tôpico Anderson Torres
Em ter., 18 de ago. de 2020 às 19:51, Pedro José  escreveu:
>
> Boa noite!
> Cláudio,
> não consegui nada geométrico.
> O máximo que atingi foi:
> a/ha + b/hb + c/hc= [cotg(A1) +cotg (A2)]  + [cotg(B1) +cotg (B2)] + 
> co[tg(C1) +cotg (C2)] com A1 + A2 = A; B1 + B2 + B e C1 + C2 = C.
> Para ser mínimo cada termo entre colchetes deve ser mínimo, o que ocorre 
> quando A1 = A2; B1 = B2 e C1 = C2. Logo P seria o encontro das bissetrizes e 
> logo I.
> Onde: A1= PAB e A2=PAC; B1=PBA e B2=PBC; C1=PCA e C2=PCB.

Acho que daqui poderia sair uma interpretação mais escamoteada.
Afinal, trigonometria é uma espécie de "ponto de contato" entre a
geometria analítica e a sintética, entre a nuvem de desenhos e a de
números.

Acredito que a solução aqui seria arranjar uma interpretação
geométrica desses colchetes de co-tangentes. Acredito que possamos
apelar para Ptolomeu em algum momento ou para um macete de
semelhanças, pois as projeções de um ponto sobre duas retas criam um
quadrilátero cíclico.

Isso até me lembra o famoso artigo do Shine sobre geometria cearense
VS geometria paulista:
https://cyshine.webs.com/geometria-2005.pdf


>
> Saudações,
> PJMS
>
> Em ter., 18 de ago. de 2020 às 11:34, Claudio Buffara 
>  escreveu:
>>
>> Será que tem uma demonstração mais geométrica e menos algébrica disso? E que 
>> torne o resultado mais intuitivo?
>> É razoável que o ponto P não esteja muito próximo de qualquer dos lados, 
>> pois neste caso, se P se aproximasse do lado a, por exemplo, a/h_a cresceria 
>> e a expressão se afastaria do valor mínimo.
>> Mas, com lados não necessariamente congruentes, não é óbvio, a priori, que P 
>> deva ser equidistante dos três.
>> De fato, seria razoável esperar que P estivesse mais próximo do maior lado e 
>> conjecturar, por exemplo, que o P que minimiza a expressão é tal que a/h_a = 
>> b/h_b = c/h_c.
>> O fato de P ser o incentro não me parece a conjectura mais evidente neste 
>> caso.
>>
>>
>> On Sun, Aug 16, 2020 at 10:11 AM Matheus Secco  
>> wrote:
>>>
>>> Olá, Vanderlei.
>>> Por Cauchy-Schwarz, temos
>>>
>>> (a/ha + b/hb + c/hc) * (a*ha + b*hb + c*hc) >= (a+b+c)^2.  (#)
>>>
>>> Como (a*ha + b*hb + c*hc) = 2S, onde S é a área de ABC, segue que a 
>>> expressão a/ha + b/hb + c/hc é pelo menos 2p^2/S, onde p é o semi-perimetro.
>>>
>>> Por outro lado, a igualdade em (#) ocorre se, e somente se, ha = hb = hc, 
>>> ou seja, quando P é o incentro do triângulo
>>>
>>> Abraços,
>>> Matheus
>>>
>>> Em dom, 16 de ago de 2020 08:59, Professor Vanderlei Nemitz 
>>>  escreveu:

 Bom dia!

 Tentei utilizar alguma desigualdade de médias aqui, mas não tive êxito. 
 Alguém ajuda?
 Muito agradecido!

 Seja P um ponto no interior de um triângulo e sejam ha, hb e hc as 
 distâncias de P aos lados a, b e c, respectivamente. Mostre que o valor 
 mínimo de (a/ha) + (b/hb) + (c/hc) ocorre quando P é o incentivo do 
 triângulo ABC.

 --
 Esta mensagem foi verificada pelo sistema de antivírus e
 acredita-se estar livre de perigo.
>>>
>>>
>>> --
>>> Esta mensagem foi verificada pelo sistema de antivírus e
>>> acredita-se estar livre de perigo.
>>
>>
>> --
>> Esta mensagem foi verificada pelo sistema de antivírus e
>> acredita-se estar livre de perigo.
>
>
> --
> Esta mensagem foi verificada pelo sistema de antivírus e
> acredita-se estar livre de perigo.

-- 
Esta mensagem foi verificada pelo sistema de antiv�rus e
 acredita-se estar livre de perigo.


=
Instru��es para entrar na lista, sair da lista e usar a lista em
http://www.mat.puc-rio.br/~obmlistas/obm-l.html
=


[obm-l] Re: [obm-l] Re: [obm-l] Re: [obm-l] Re: [obm-l] Geometria plana com desigualdade de médias?

2020-08-18 Por tôpico Claudio Buffara
Realmente, não era isso que eu estava procurando...  mas valeu! É outra
solução.


On Tue, Aug 18, 2020 at 7:51 PM Pedro José  wrote:

> Boa noite!
> Cláudio,
> não consegui nada geométrico.
> O máximo que atingi foi:
> a/ha + b/hb + c/hc= [cotg(A1) +cotg (A2)]  + [cotg(B1) +cotg (B2)] +
> co[tg(C1) +cotg (C2)] com A1 + A2 = A; B1 + B2 + B e C1 + C2 = C.
> Para ser mínimo cada termo entre colchetes deve ser mínimo, o que ocorre
> quando A1 = A2; B1 = B2 e C1 = C2. Logo P seria o encontro das bissetrizes
> e logo I.
> Onde: A1= PAB e A2=PAC; B1=PBA e B2=PBC; C1=PCA e C2=PCB.
>
> Saudações,
> PJMS
>
> Em ter., 18 de ago. de 2020 às 11:34, Claudio Buffara <
> claudio.buff...@gmail.com> escreveu:
>
>> Será que tem uma demonstração mais geométrica e menos algébrica disso? E
>> que torne o resultado mais intuitivo?
>> É razoável que o ponto P não esteja muito próximo de qualquer dos lados,
>> pois neste caso, se P se aproximasse do lado a, por exemplo,
>> a/h_a cresceria e a expressão se afastaria do valor mínimo.
>> Mas, com lados não necessariamente congruentes, não é óbvio, a priori,
>> que P deva ser equidistante dos três.
>> De fato, seria razoável esperar que P estivesse mais próximo do maior
>> lado e conjecturar, por exemplo, que o P que minimiza a expressão é tal que
>> a/h_a = b/h_b = c/h_c.
>> O fato de P ser o incentro não me parece a conjectura mais evidente neste
>> caso.
>>
>>
>> On Sun, Aug 16, 2020 at 10:11 AM Matheus Secco 
>> wrote:
>>
>>> Olá, Vanderlei.
>>> Por Cauchy-Schwarz, temos
>>>
>>> (a/ha + b/hb + c/hc) * (a*ha + b*hb + c*hc) >= (a+b+c)^2.  (#)
>>>
>>> Como (a*ha + b*hb + c*hc) = 2S, onde S é a área de ABC, segue que a
>>> expressão a/ha + b/hb + c/hc é pelo menos 2p^2/S, onde p é o
>>> semi-perimetro.
>>>
>>> Por outro lado, a igualdade em (#) ocorre se, e somente se, ha = hb =
>>> hc, ou seja, quando P é o incentro do triângulo
>>>
>>> Abraços,
>>> Matheus
>>>
>>> Em dom, 16 de ago de 2020 08:59, Professor Vanderlei Nemitz <
>>> vanderma...@gmail.com> escreveu:
>>>
 Bom dia!

 Tentei utilizar alguma desigualdade de médias aqui, mas não tive êxito.
 Alguém ajuda?
 Muito agradecido!

 Seja P um ponto no interior de um triângulo e sejam ha, hb e hc as
 distâncias de P aos lados a, b e c, respectivamente. Mostre que o valor
 mínimo de (a/ha) + (b/hb) + (c/hc) ocorre quando P é o incentivo do
 triângulo ABC.

 --
 Esta mensagem foi verificada pelo sistema de antivírus e
 acredita-se estar livre de perigo.
>>>
>>>
>>> --
>>> Esta mensagem foi verificada pelo sistema de antivírus e
>>> acredita-se estar livre de perigo.
>>
>>
>> --
>> Esta mensagem foi verificada pelo sistema de antivírus e
>> acredita-se estar livre de perigo.
>
>
> --
> Esta mensagem foi verificada pelo sistema de antivírus e
> acredita-se estar livre de perigo.

-- 
Esta mensagem foi verificada pelo sistema de antiv�rus e
 acredita-se estar livre de perigo.



[obm-l] Re: [obm-l] Re: [obm-l] Re: [obm-l] Geometria plana com desigualdade de médias?

2020-08-18 Por tôpico Pedro José
Boa noite!
Cláudio,
não consegui nada geométrico.
O máximo que atingi foi:
a/ha + b/hb + c/hc= [cotg(A1) +cotg (A2)]  + [cotg(B1) +cotg (B2)] +
co[tg(C1) +cotg (C2)] com A1 + A2 = A; B1 + B2 + B e C1 + C2 = C.
Para ser mínimo cada termo entre colchetes deve ser mínimo, o que ocorre
quando A1 = A2; B1 = B2 e C1 = C2. Logo P seria o encontro das bissetrizes
e logo I.
Onde: A1= PAB e A2=PAC; B1=PBA e B2=PBC; C1=PCA e C2=PCB.

Saudações,
PJMS

Em ter., 18 de ago. de 2020 às 11:34, Claudio Buffara <
claudio.buff...@gmail.com> escreveu:

> Será que tem uma demonstração mais geométrica e menos algébrica disso? E
> que torne o resultado mais intuitivo?
> É razoável que o ponto P não esteja muito próximo de qualquer dos lados,
> pois neste caso, se P se aproximasse do lado a, por exemplo,
> a/h_a cresceria e a expressão se afastaria do valor mínimo.
> Mas, com lados não necessariamente congruentes, não é óbvio, a priori, que
> P deva ser equidistante dos três.
> De fato, seria razoável esperar que P estivesse mais próximo do maior lado
> e conjecturar, por exemplo, que o P que minimiza a expressão é tal que
> a/h_a = b/h_b = c/h_c.
> O fato de P ser o incentro não me parece a conjectura mais evidente neste
> caso.
>
>
> On Sun, Aug 16, 2020 at 10:11 AM Matheus Secco 
> wrote:
>
>> Olá, Vanderlei.
>> Por Cauchy-Schwarz, temos
>>
>> (a/ha + b/hb + c/hc) * (a*ha + b*hb + c*hc) >= (a+b+c)^2.  (#)
>>
>> Como (a*ha + b*hb + c*hc) = 2S, onde S é a área de ABC, segue que a
>> expressão a/ha + b/hb + c/hc é pelo menos 2p^2/S, onde p é o
>> semi-perimetro.
>>
>> Por outro lado, a igualdade em (#) ocorre se, e somente se, ha = hb = hc,
>> ou seja, quando P é o incentro do triângulo
>>
>> Abraços,
>> Matheus
>>
>> Em dom, 16 de ago de 2020 08:59, Professor Vanderlei Nemitz <
>> vanderma...@gmail.com> escreveu:
>>
>>> Bom dia!
>>>
>>> Tentei utilizar alguma desigualdade de médias aqui, mas não tive êxito.
>>> Alguém ajuda?
>>> Muito agradecido!
>>>
>>> Seja P um ponto no interior de um triângulo e sejam ha, hb e hc as
>>> distâncias de P aos lados a, b e c, respectivamente. Mostre que o valor
>>> mínimo de (a/ha) + (b/hb) + (c/hc) ocorre quando P é o incentivo do
>>> triângulo ABC.
>>>
>>> --
>>> Esta mensagem foi verificada pelo sistema de antivírus e
>>> acredita-se estar livre de perigo.
>>
>>
>> --
>> Esta mensagem foi verificada pelo sistema de antivírus e
>> acredita-se estar livre de perigo.
>
>
> --
> Esta mensagem foi verificada pelo sistema de antivírus e
> acredita-se estar livre de perigo.

-- 
Esta mensagem foi verificada pelo sistema de antiv�rus e
 acredita-se estar livre de perigo.



[obm-l] Re: [obm-l] Re: [obm-l] Geometria plana com desigualdade de médias?

2020-08-18 Por tôpico Claudio Buffara
Será que tem uma demonstração mais geométrica e menos algébrica disso? E
que torne o resultado mais intuitivo?
É razoável que o ponto P não esteja muito próximo de qualquer dos lados,
pois neste caso, se P se aproximasse do lado a, por exemplo,
a/h_a cresceria e a expressão se afastaria do valor mínimo.
Mas, com lados não necessariamente congruentes, não é óbvio, a priori, que
P deva ser equidistante dos três.
De fato, seria razoável esperar que P estivesse mais próximo do maior lado
e conjecturar, por exemplo, que o P que minimiza a expressão é tal que
a/h_a = b/h_b = c/h_c.
O fato de P ser o incentro não me parece a conjectura mais evidente neste
caso.


On Sun, Aug 16, 2020 at 10:11 AM Matheus Secco 
wrote:

> Olá, Vanderlei.
> Por Cauchy-Schwarz, temos
>
> (a/ha + b/hb + c/hc) * (a*ha + b*hb + c*hc) >= (a+b+c)^2.  (#)
>
> Como (a*ha + b*hb + c*hc) = 2S, onde S é a área de ABC, segue que a
> expressão a/ha + b/hb + c/hc é pelo menos 2p^2/S, onde p é o
> semi-perimetro.
>
> Por outro lado, a igualdade em (#) ocorre se, e somente se, ha = hb = hc,
> ou seja, quando P é o incentro do triângulo
>
> Abraços,
> Matheus
>
> Em dom, 16 de ago de 2020 08:59, Professor Vanderlei Nemitz <
> vanderma...@gmail.com> escreveu:
>
>> Bom dia!
>>
>> Tentei utilizar alguma desigualdade de médias aqui, mas não tive êxito.
>> Alguém ajuda?
>> Muito agradecido!
>>
>> Seja P um ponto no interior de um triângulo e sejam ha, hb e hc as
>> distâncias de P aos lados a, b e c, respectivamente. Mostre que o valor
>> mínimo de (a/ha) + (b/hb) + (c/hc) ocorre quando P é o incentivo do
>> triângulo ABC.
>>
>> --
>> Esta mensagem foi verificada pelo sistema de antivírus e
>> acredita-se estar livre de perigo.
>
>
> --
> Esta mensagem foi verificada pelo sistema de antivírus e
> acredita-se estar livre de perigo.

-- 
Esta mensagem foi verificada pelo sistema de antiv�rus e
 acredita-se estar livre de perigo.



[obm-l] Re: [obm-l] Re: [obm-l] Geometria plana com desigualdade de médias?

2020-08-17 Por tôpico Professor Vanderlei Nemitz
Muito obrigado, Matheus!
Pensei nas outras desigualdades, menos em Cauchy-Schwarz.

Muito bom!

Em dom, 16 de ago de 2020 10:11, Matheus Secco 
escreveu:

> Olá, Vanderlei.
> Por Cauchy-Schwarz, temos
>
> (a/ha + b/hb + c/hc) * (a*ha + b*hb + c*hc) >= (a+b+c)^2.  (#)
>
> Como (a*ha + b*hb + c*hc) = 2S, onde S é a área de ABC, segue que a
> expressão a/ha + b/hb + c/hc é pelo menos 2p^2/S, onde p é o
> semi-perimetro.
>
> Por outro lado, a igualdade em (#) ocorre se, e somente se, ha = hb = hc,
> ou seja, quando P é o incentro do triângulo
>
> Abraços,
> Matheus
>
> Em dom, 16 de ago de 2020 08:59, Professor Vanderlei Nemitz <
> vanderma...@gmail.com> escreveu:
>
>> Bom dia!
>>
>> Tentei utilizar alguma desigualdade de médias aqui, mas não tive êxito.
>> Alguém ajuda?
>> Muito agradecido!
>>
>> Seja P um ponto no interior de um triângulo e sejam ha, hb e hc as
>> distâncias de P aos lados a, b e c, respectivamente. Mostre que o valor
>> mínimo de (a/ha) + (b/hb) + (c/hc) ocorre quando P é o incentivo do
>> triângulo ABC.
>>
>> --
>> Esta mensagem foi verificada pelo sistema de antivírus e
>> acredita-se estar livre de perigo.
>
>
> --
> Esta mensagem foi verificada pelo sistema de antivírus e
> acredita-se estar livre de perigo.

-- 
Esta mensagem foi verificada pelo sistema de antiv�rus e
 acredita-se estar livre de perigo.



[obm-l] Re: [obm-l] Geometria plana com desigualdade de médias?

2020-08-16 Por tôpico Matheus Secco
Olá, Vanderlei.
Por Cauchy-Schwarz, temos

(a/ha + b/hb + c/hc) * (a*ha + b*hb + c*hc) >= (a+b+c)^2.  (#)

Como (a*ha + b*hb + c*hc) = 2S, onde S é a área de ABC, segue que a
expressão a/ha + b/hb + c/hc é pelo menos 2p^2/S, onde p é o
semi-perimetro.

Por outro lado, a igualdade em (#) ocorre se, e somente se, ha = hb = hc,
ou seja, quando P é o incentro do triângulo

Abraços,
Matheus

Em dom, 16 de ago de 2020 08:59, Professor Vanderlei Nemitz <
vanderma...@gmail.com> escreveu:

> Bom dia!
>
> Tentei utilizar alguma desigualdade de médias aqui, mas não tive êxito.
> Alguém ajuda?
> Muito agradecido!
>
> Seja P um ponto no interior de um triângulo e sejam ha, hb e hc as
> distâncias de P aos lados a, b e c, respectivamente. Mostre que o valor
> mínimo de (a/ha) + (b/hb) + (c/hc) ocorre quando P é o incentivo do
> triângulo ABC.
>
> --
> Esta mensagem foi verificada pelo sistema de antivírus e
> acredita-se estar livre de perigo.

-- 
Esta mensagem foi verificada pelo sistema de antiv�rus e
 acredita-se estar livre de perigo.



Re: [obm-l] Re: [obm-l] Re: [obm-l] Geometria analítica

2019-07-18 Por tôpico Claudio Buffara
Se a reta for perpendicular a MN, intersectando o segmento no ponto P, digamos, 
então a solução é Q = P.
Isso pode ser visto sem cálculo. Apenas comPitágoras  e algebra 
(especificamente, a identidade:
 raiz(a) - raiz(b) = (a - b)/(raiz(a) +  raiz(b))

Pro caso da reta ser oblíqua, Pitágoras é substituído pela lei dos cossenos e a 
álgebra fica mais chatinha.

Abs

Enviado do meu iPhone

Em 17 de jul de 2019, à(s) 23:36, Rodrigo Ângelo  
escreveu:

> Acho que neste caso dá pra usar hipérboles 
> 
> Uma sequência de hipérboles que passam por M e N, com um foco em Q1, Q2, 
> ..., Qn tenderia à Q que maximiza a diferença entre distâncias quando as 
> retas que passam por MQ e NQ são perpendiculares, certo?
> 
>> On Tue, Jul 16, 2019, 1:50 PM Vanderlei Nemitz  wrote:
>> Com certeza! É que nesse caso os pontos estão em semiplanos opostos. 
>> Talvez seria isso que eu gostaria de perguntar. Será que nesse caso sim?
>> Mas e sem derivadas? Será possível resolver? Preciso apresentar a 
>> solução para alunos que não estudaram derivadas...
>> 
>> Muito obrigado!
>> 
>> Em ter, 16 de jul de 2019 Ã s 13:30, Claudio Buffara 
>>  escreveu:
>>> A resposta da 2a questão é NÃO. Pense em M e N próximos um do outro e 
>>> tão distantes da reta que o ângulo MQN é sempre agudo.
>>> 
>>> Abs
>>> 
>>> Enviado do meu iPhone
>>> 
>>> Em 16 de jul de 2019, Ã (s) 15:44, Vanderlei Nemitz  
>>> escreveu:
>>> 
>>> > Pessoal, é possível resolver a seguinte questão sem utilizar 
>>> > derivadas?
>>> > 
>>> > Determinar as coordenadas de um ponto Q pertencente à reta de 
>>> > equação y = 3x - 1 tal que a diferença de suas distâncias aos 
>>> > pontos M(4, 1) e N(0, 4) seja máxima.
>>> > 
>>> > A resposta mostra que o triângulo MQN é retângulo em Q, para que 
>>> > a diferença seja máxima. Isso ocorre sempre?
>>> > 
>>> > 
>>> > Muito obrigado!
>>> > 
>>> > Vander
>>> > 
>>> > -- 
>>> > Esta mensagem foi verificada pelo sistema de antivírus e 
>>> > acredita-se estar livre de perigo.
>>> 
>>> -- 
>>> Esta mensagem foi verificada pelo sistema de antivírus e
>>> Â acredita-se estar livre de perigo.
>>> 
>>> 
>>> =
>>> Instru�ões para entrar na lista, sair da lista e usar a lista em
>>> http://www.mat.puc-rio.br/~obmlistas/obm-l.html
>>> =
>> 
>> -- 
>> Esta mensagem foi verificada pelo sistema de antivírus e 
>> acredita-se estar livre de perigo.
> 
> -- 
> Esta mensagem foi verificada pelo sistema de antivírus e 
> acredita-se estar livre de perigo.

-- 
Esta mensagem foi verificada pelo sistema de antiv�rus e
 acredita-se estar livre de perigo.



[obm-l] Re: [obm-l] Re: [obm-l] Geometria analítica

2019-07-17 Por tôpico Rodrigo Ângelo
Acho que neste caso dá pra usar hipérboles

Uma sequência de hipérboles que passam por M e N, com um foco em Q1, Q2,
..., Qn tenderia à Q que maximiza a diferença entre distâncias quando as
retas que passam por MQ e NQ são perpendiculares, certo?

On Tue, Jul 16, 2019, 1:50 PM Vanderlei Nemitz 
wrote:

> Com certeza! É que nesse caso os pontos estão em semiplanos opostos.
> Talvez seria isso que eu gostaria de perguntar. Será que nesse caso sim?
> Mas e sem derivadas? Será possível resolver? Preciso apresentar a solução
> para alunos que não estudaram derivadas...
>
> Muito obrigado!
>
> Em ter, 16 de jul de 2019 às 13:30, Claudio Buffara <
> claudio.buff...@gmail.com> escreveu:
>
>> A resposta da 2a questão é NÃO. Pense em M e N próximos um do outro e tão
>> distantes da reta que o ângulo MQN é sempre agudo.
>>
>> Abs
>>
>> Enviado do meu iPhone
>>
>> Em 16 de jul de 2019, à(s) 15:44, Vanderlei Nemitz 
>> escreveu:
>>
>> > Pessoal, é possível resolver a seguinte questão sem utilizar
>> derivadas?
>> >
>> > Determinar as coordenadas de um ponto Q pertencente à reta de
>> equação y = 3x - 1 tal que a diferença de suas distâncias aos pontos
>> M(4, 1) e N(0, 4) seja máxima.
>> >
>> > A resposta mostra que o triângulo MQN é retângulo em Q, para que a
>> diferença seja máxima. Isso ocorre sempre?
>> >
>> >
>> > Muito obrigado!
>> >
>> > Vander
>> >
>> > --
>> > Esta mensagem foi verificada pelo sistema de antivírus e
>> > acredita-se estar livre de perigo.
>>
>> --
>> Esta mensagem foi verificada pelo sistema de antivírus e
>>  acredita-se estar livre de perigo.
>>
>>
>> =
>> Instru�ões para entrar na lista, sair da lista e usar a lista em
>> http://www.mat.puc-rio.br/~obmlistas/obm-l.html
>> =
>>
>
> --
> Esta mensagem foi verificada pelo sistema de antivírus e
> acredita-se estar livre de perigo.

-- 
Esta mensagem foi verificada pelo sistema de antiv�rus e
 acredita-se estar livre de perigo.



[obm-l] Re: [obm-l] Geometria analítica

2019-07-16 Por tôpico Vanderlei Nemitz
Com certeza! É que nesse caso os pontos estão em semiplanos opostos. Talvez
seria isso que eu gostaria de perguntar. Será que nesse caso sim?
Mas e sem derivadas? Será possível resolver? Preciso apresentar a solução
para alunos que não estudaram derivadas...

Muito obrigado!

Em ter, 16 de jul de 2019 às 13:30, Claudio Buffara <
claudio.buff...@gmail.com> escreveu:

> A resposta da 2a questão é NÃO. Pense em M e N próximos um do outro e tão
> distantes da reta que o ângulo MQN é sempre agudo.
>
> Abs
>
> Enviado do meu iPhone
>
> Em 16 de jul de 2019, à(s) 15:44, Vanderlei Nemitz 
> escreveu:
>
> > Pessoal, é possível resolver a seguinte questão sem utilizar
> derivadas?
> >
> > Determinar as coordenadas de um ponto Q pertencente à reta de equação
> y = 3x - 1 tal que a diferença de suas distâncias aos pontos M(4, 1) e
> N(0, 4) seja máxima.
> >
> > A resposta mostra que o triângulo MQN é retângulo em Q, para que a
> diferença seja máxima. Isso ocorre sempre?
> >
> >
> > Muito obrigado!
> >
> > Vander
> >
> > --
> > Esta mensagem foi verificada pelo sistema de antivírus e
> > acredita-se estar livre de perigo.
>
> --
> Esta mensagem foi verificada pelo sistema de antivírus e
>  acredita-se estar livre de perigo.
>
>
> =
> Instru�ões para entrar na lista, sair da lista e usar a lista em
> http://www.mat.puc-rio.br/~obmlistas/obm-l.html
> =
>

-- 
Esta mensagem foi verificada pelo sistema de antiv�rus e
 acredita-se estar livre de perigo.



[obm-l] Re: [obm-l] Re: [obm-l] Geometria Analítica

2017-08-22 Por tôpico Francisco Barreto
perdão

On Tue, 22 Aug 2017 at 20:04 Ralph Teixeira  wrote:

> Usando Geometria: seja M o ponto medio de AB. Note que M eh fixo.
>
> O Teorema de Apolonio
>  diz que
>
> PA^2+PB^2 = 2(PM^2+a^2)
>
> (obs: isso vale mesmo que P esteja na reta AB). Entao PM^2=k^2/2 - a^2 eh
> fixo. Assim, tipicamente o lugar geometrico de P eh um circulo de centro M
> e raio quadrado k^2/2 - a^2...
>
> Digo "tipicamente" porque temos que analisar se esse raio existe mesmo...
> Entao:
> a) Se k^2<2a^2, entao o L.G. serah vazio
> b) Se k^2=2a^2, entao o L.G. serah apenas o ponto M.
> c) Se k^2>2a^2, entao realmente dah aquele circulo que eu citei -- mas
> tecnicamente tem que ver se os pontos onde esse circulo corta a reta AB
> tambem servem, porque PAB nao seria tecnicamente um triangulo (resposta:
> sim, servem!).
>
> Usando Vetores: (uso  para produto interno)
> +=k^2
> 2-2-2++=k^2
> -=(k^2--)/2
> Agora complete quadrados
> -2+<(A+B)/2,(A+B)/2> = (k^2
> --)/2+<(A+B)/2,(A+B)/2>
>  = k^2/2  -<(A-B)/2,(A-B)/2> = k^2/2 - a^2
> ||P - (A+B)/2|| ^ 2 = k^2/2 - a^2
> Ou seja, a distancia de P a M=(A+B)/2 eh fixa e igual a k^2/2-a^2
>
> Abraco, Ralph.
>
> 2017-08-22 19:31 GMT-03:00 André Lauer :
>
>> Boa noite, preciso de ajuda no seguinte problema:
>> São dados dois pontos A e B. Determine o lugar geométrico de P tal que
>> d(A,P)^2 + d(P,B)^2 = k^2 onde k é uma constante dada.
>> Se d(A,B) = 2a, determine para que valores de k o problema tem solução.
>>
>> --
>> Esta mensagem foi verificada pelo sistema de antivírus e
>> acredita-se estar livre de perigo.
>>
>
>
> --
> Esta mensagem foi verificada pelo sistema de antivírus e
> acredita-se estar livre de perigo.

-- 
Esta mensagem foi verificada pelo sistema de antiv�rus e
 acredita-se estar livre de perigo.



[obm-l] Re: [obm-l] Geometria Analítica

2017-08-22 Por tôpico Ralph Teixeira
Usando Geometria: seja M o ponto medio de AB. Note que M eh fixo.

O Teorema de Apolonio 
diz que

PA^2+PB^2 = 2(PM^2+a^2)

(obs: isso vale mesmo que P esteja na reta AB). Entao PM^2=k^2/2 - a^2 eh
fixo. Assim, tipicamente o lugar geometrico de P eh um circulo de centro M
e raio quadrado k^2/2 - a^2...

Digo "tipicamente" porque temos que analisar se esse raio existe mesmo...
Entao:
a) Se k^2<2a^2, entao o L.G. serah vazio
b) Se k^2=2a^2, entao o L.G. serah apenas o ponto M.
c) Se k^2>2a^2, entao realmente dah aquele circulo que eu citei -- mas
tecnicamente tem que ver se os pontos onde esse circulo corta a reta AB
tambem servem, porque PAB nao seria tecnicamente um triangulo (resposta:
sim, servem!).

Usando Vetores: (uso  para produto interno)
+=k^2
2-2-2++=k^2
-=(k^2--)/2
Agora complete quadrados
-2+<(A+B)/2,(A+B)/2> = (k^2
--)/2+<(A+B)/2,(A+B)/2>
 = k^2/2  -<(A-B)/2,(A-B)/2> = k^2/2 - a^2
||P - (A+B)/2|| ^ 2 = k^2/2 - a^2
Ou seja, a distancia de P a M=(A+B)/2 eh fixa e igual a k^2/2-a^2

Abraco, Ralph.

2017-08-22 19:31 GMT-03:00 André Lauer :

> Boa noite, preciso de ajuda no seguinte problema:
> São dados dois pontos A e B. Determine o lugar geométrico de P tal que
> d(A,P)^2 + d(P,B)^2 = k^2 onde k é uma constante dada.
> Se d(A,B) = 2a, determine para que valores de k o problema tem solução.
>
> --
> Esta mensagem foi verificada pelo sistema de antivírus e
> acredita-se estar livre de perigo.
>

-- 
Esta mensagem foi verificada pelo sistema de antiv�rus e
 acredita-se estar livre de perigo.



[obm-l] Re: [obm-l] Geometria Analítica

2017-08-22 Por tôpico Francisco Barreto
acho que faltou dr nome aos bois, as coordenadas.

On Tue, 22 Aug 2017 at 19:45 Francisco Barreto 
wrote:

> a hipotenusa tem que ser d(A,B), não? Se for o caso vale k ao quadrado e
> 2a.
>
> On Tue, 22 Aug 2017 at 19:37 André Lauer 
> wrote:
>
>> Boa noite, preciso de ajuda no seguinte problema:
>> São dados dois pontos A e B. Determine o lugar geométrico de P tal que
>> d(A,P)^2 + d(P,B)^2 = k^2 onde k é uma constante dada.
>> Se d(A,B) = 2a, determine para que valores de k o problema tem solução.
>>
>> --
>> Esta mensagem foi verificada pelo sistema de antivírus e
>> acredita-se estar livre de perigo.
>>
>

-- 
Esta mensagem foi verificada pelo sistema de antiv�rus e
 acredita-se estar livre de perigo.



[obm-l] Re: [obm-l] Geometria Analítica

2017-08-22 Por tôpico Francisco Barreto
a hipotenusa tem que ser d(A,B), não? Se for o caso vale k ao quadrado e 2a.

On Tue, 22 Aug 2017 at 19:37 André Lauer  wrote:

> Boa noite, preciso de ajuda no seguinte problema:
> São dados dois pontos A e B. Determine o lugar geométrico de P tal que
> d(A,P)^2 + d(P,B)^2 = k^2 onde k é uma constante dada.
> Se d(A,B) = 2a, determine para que valores de k o problema tem solução.
>
> --
> Esta mensagem foi verificada pelo sistema de antivírus e
> acredita-se estar livre de perigo.
>

-- 
Esta mensagem foi verificada pelo sistema de antiv�rus e
 acredita-se estar livre de perigo.



[obm-l] Re: [obm-l] Geometria plana

2017-07-08 Por tôpico Julio César Saldaña



Note que os triângulos ABD e BCE são equivalentes (mesma área).
Baseado nisso podemos concluir que BE=AD; pois areas iguais e alturas iguais
implica bases iguais.

Então os triângulos ABD e BCE além de equivalente são congruentes (L.A.L.).
Portanto 
Num triângulo equilátero ABC, as cevianas BD e CE se encontram em P, se a
área do triângulo BCP é igual a área do quadrilátero ADPE ,  determine o
ângulo BPC.

Douglas Oliveira.

--
Esta mensagem foi verificada pelo sistema de antivírus e
acredita-se estar livre de perigo.




__
Si desea recibir, semanalmente, el Boletín Electrónico de la PUCP, ingrese a:
http://www.pucp.edu.pe/puntoedu/suscribete/


--
Esta mensagem foi verificada pelo sistema de antivírus e
acredita-se estar livre de perigo.

=
Instruções para entrar na lista, sair da lista e usar a lista em
http://www.mat.puc-rio.br/~obmlistas/obm-l.html
=


Re: [obm-l] Re: [obm-l] Geometria plana (Ajuda)

2017-07-03 Por tôpico Pedro José
Boa noite!

Bela e simples solução!

Saudações,
PJMS

Em 29 de junho de 2017 18:21, Julio César Saldaña 
escreveu:

>
>
> Aproveitando que APC é isósceles (pois CA=CP), eu desenhei a altura CH,
> então
> AH=HP e anguloACH=anguloHCP=20; mas como também anguloPCB=20, decidi
> desenhar a
> perpendicular PN sobre BC, así temos PN=PH=HA. Aí não resisti e estiquei
> PN até
> K, onde NK=PN. Desenhei a linha BK também.
>
> Nesse ponto me encontrei com um problema que já tinha resolvido faz algum
> tempo
> mas não lembro. Então ensaiei outra solução. O problema é provar que P é o
> circuncentro do triângulo ABK. Desta vez argumento assim: como anguloBAK=30
> então BK é igual ao circunrádio do triângulo ABK. Mas note que BK=BP (pois
> BC é
> mediatriz de PK). Então pronto, P encontrase na mediatriz de AK e também se
> encontra a uma distância de B igual ao circunradio, logo P é o
> circuncentro do
> triângulo ABK.
>
> Com isso, o triângulo PBK é equilátero e portanto anguloPBN=30. Portanto
> anguloBEC=90 => EM é mediana relativa à hipotenusa do triângulo retângulo
> BEC.
> Resposta: anguloMEC=60
>
> Espero não ter me engando, mas vou fazer um double check e também vou
> tentar
> lembrar a outra forma de provar que P é circuncentro de ABK
>
> Julio Saldaña
>
>
> -- Mensaje original ---
> De : obm-l@mat.puc-rio.br
> Para : obm-l@mat.puc-rio.br
> Fecha : Wed, 28 Jun 2017 14:43:07 -0300
> Asunto : Re: [obm-l] Geometria plana (Ajuda)
> >Opa desculpe, CF é ceviana que passa por P.
> >
> >Em 28 de jun de 2017 11:05 AM, "Pedro José" 
> escreveu:
> >
> >> Bom dia!
> >>
> >> O ponto F não foi definido, mas foram definidas duas medidas de
> ângulos
> >> aos quais o ponto F pertence: BCF=20 graus e FCA=40 graus.
> >> Não faltou definir o ponto F?
> >>
> >> Sds,
> >> PJMS
> >>
> >> Em 28 de junho de 2017 09:15, Douglas Oliveira de Lima <
> >> profdouglaso.del...@gmail.com> escreveu:
> >>
> >>> Olá meus amigos preciso de uma ajuda pra resolver a seguinte questão:
> >>>
> >>> Num triângulo ABC , tracam-se as cevianas AD e BE, que se encontram no
> >>> ponto P, tal que BAD= 10 graus, DAC=70 graus, BCF=20 graus e FCA=40
> graus,
> >>> traçando a ceviana BE que passa por P e o segmento de reta que une os
> >>> pontos E e M, sendo M ponto médio de BC, determinar o ângulo CME.
> >>>
> >>> GRATO!!
> >>> Douglas Oliveira.
> >>>
> >>> --
> >>> Esta mensagem foi verificada pelo sistema de antivírus e
> >>> acredita-se estar livre de perigo.
> >>
> >>
> >>
> >> --
> >> Esta mensagem foi verificada pelo sistema de antivírus e
> >> acredita-se estar livre de perigo.
> >
> >--
> >Esta mensagem foi verificada pelo sistema de antivírus e
> > acredita-se estar livre de perigo.
> >
>
>
> __
> Si desea recibir, semanalmente, el Boletín Electrónico de la PUCP, ingrese
> a:
> http://www.pucp.edu.pe/puntoedu/suscribete/
>
>
> --
> Esta mensagem foi verificada pelo sistema de antivírus e
>  acredita-se estar livre de perigo.
>
> =
> Instruções para entrar na lista, sair da lista e usar a lista em
> http://www.mat.puc-rio.br/~obmlistas/obm-l.html
> =
>

-- 
Esta mensagem foi verificada pelo sistema de antiv�rus e
 acredita-se estar livre de perigo.



[obm-l] Re: [obm-l] Geometria plana (Ajuda)

2017-06-29 Por tôpico Julio César Saldaña



Aproveitando que APC é isósceles (pois CA=CP), eu desenhei a altura CH, então 
AH=HP e anguloACH=anguloHCP=20; mas como também anguloPCB=20, decidi desenhar a 
perpendicular PN sobre BC, así temos PN=PH=HA. Aí não resisti e estiquei PN até 
K, onde NK=PN. Desenhei a linha BK também.


Nesse ponto me encontrei com um problema que já tinha resolvido faz algum tempo 
mas não lembro. Então ensaiei outra solução. O problema é provar que P é o 
circuncentro do triângulo ABK. Desta vez argumento assim: como anguloBAK=30 
então BK é igual ao circunrádio do triângulo ABK. Mas note que BK=BP (pois BC é 
mediatriz de PK). Então pronto, P encontrase na mediatriz de AK e também se 
encontra a uma distância de B igual ao circunradio, logo P é o circuncentro do 
triângulo ABK.


Com isso, o triângulo PBK é equilátero e portanto anguloPBN=30. Portanto 
anguloBEC=90 => EM é mediana relativa à hipotenusa do triângulo retângulo BEC. 
Resposta: anguloMEC=60


Espero não ter me engando, mas vou fazer um double check e também vou tentar 
lembrar a outra forma de provar que P é circuncentro de ABK


Julio Saldaña


-- Mensaje original ---
De : obm-l@mat.puc-rio.br
Para : obm-l@mat.puc-rio.br
Fecha : Wed, 28 Jun 2017 14:43:07 -0300
Asunto : Re: [obm-l] Geometria plana (Ajuda)

Opa desculpe, CF é ceviana que passa por P.

Em 28 de jun de 2017 11:05 AM, "Pedro José"  escreveu:


Bom dia!

O ponto F não foi definido, mas foram definidas duas medidas de ângulos
aos quais o ponto F pertence: BCF=20 graus e FCA=40 graus.
Não faltou definir o ponto F?

Sds,
PJMS

Em 28 de junho de 2017 09:15, Douglas Oliveira de Lima <
profdouglaso.del...@gmail.com> escreveu:


Olá meus amigos preciso de uma ajuda pra resolver a seguinte questão:

Num triângulo ABC , tracam-se as cevianas AD e BE, que se encontram no
ponto P, tal que BAD= 10 graus, DAC=70 graus, BCF=20 graus e FCA=40 graus,
traçando a ceviana BE que passa por P e o segmento de reta que une os
pontos E e M, sendo M ponto médio de BC, determinar o ângulo CME.

GRATO!!
Douglas Oliveira.

--
Esta mensagem foi verificada pelo sistema de antivírus e
acredita-se estar livre de perigo.




--
Esta mensagem foi verificada pelo sistema de antivírus e
acredita-se estar livre de perigo.


--
Esta mensagem foi verificada pelo sistema de antivírus e
acredita-se estar livre de perigo.




__
Si desea recibir, semanalmente, el Boletín Electrónico de la PUCP, ingrese a:
http://www.pucp.edu.pe/puntoedu/suscribete/


--
Esta mensagem foi verificada pelo sistema de antivírus e
acredita-se estar livre de perigo.

=
Instruções para entrar na lista, sair da lista e usar a lista em
http://www.mat.puc-rio.br/~obmlistas/obm-l.html
=


Re: [obm-l] Re: [obm-l] Geometria

2017-04-20 Por tôpico Marcelo de Moura Costa
Grato a todos pela atenção

Em 19 de abr de 2017 11:36 PM,  escreveu:

> Brilliant!
>
>
> Quoting Julio César Saldaña :
>
> Imagino que D esteja sobre BC. Se for esse o caso:
>>
>> ABD e AEC são congruentes.
>>
>> Ángulo BAD = ángulo ECA e por isso ângulo DFC = 60, logo BEFD é
>> inscritível.
>>
>> EB = 2. BD e como ângulo B = 60 então ângulo EDB=90.
>>
>> Como BEFD é inscritível então ângulo BFE=90 e finalmente ângulo BFC=90
>>
>> Julio Saldaña
>>
>>
>> -- Mensaje original ---
>> De : obm-l@mat.puc-rio.br
>> Para : obm-l@mat.puc-rio.br
>> Fecha : Mon, 17 Apr 2017 11:55:34 -0300
>> Asunto : [obm-l] Geometria
>>
>>> Bom dia a todos,
>>>
>>> Gostaria de uma ajuda com o seguinte problema:
>>>
>>> Dado um triângulo equilátero ABC, tal que sobre o lado AB tenhamos um
>>> ponto
>>> E e sobre o AC tenhamos um ponto D, com AE=BD=AB/3. Se as cevianas AD e
>>> CE
>>> intersectam  no ponto F, qual a medida do ângulo BFC?
>>>
>>> Grato pela atenção.
>>>
>>> Abraços,
>>>
>>> Marcelo
>>>
>>>
>>>
>>>
>>> "Matemática é o alfabeto com o qual Deus escreveu o Universo"
>>> Galileu Galilei
>>>
>>> --
>>> Esta mensagem foi verificada pelo sistema de antivírus e
>>> acredita-se estar livre de perigo.
>>>
>>> 
>> __
>> Si desea recibir, semanalmente, el Boletín Electrónico de la PUCP,
>> ingrese a:
>> http://www.pucp.edu.pe/puntoedu/suscribete/
>>
>>
>> --
>> Esta mensagem foi verificada pelo sistema de antivírus e
>> acredita-se estar livre de perigo.
>>
>> =
>> Instruções para entrar na lista, sair da lista e usar a lista em
>> http://www.mat.puc-rio.br/~obmlistas/obm-l.html
>> =
>>
>
>
>
>
> --
> Esta mensagem foi verificada pelo sistema de antivírus e
> acredita-se estar livre de perigo.
>
> =
> Instru�ões para entrar na lista, sair da lista e usar a lista em
> http://www.mat.puc-rio.br/~obmlistas/obm-l.html
> =
>

-- 
Esta mensagem foi verificada pelo sistema de antiv�rus e
 acredita-se estar livre de perigo.



Re: [obm-l] Re: [obm-l] Geometria

2017-04-19 Por tôpico wagner

Brilliant!


Quoting Julio César Saldaña :


Imagino que D esteja sobre BC. Se for esse o caso:

ABD e AEC são congruentes.

Ángulo BAD = ángulo ECA e por isso ângulo DFC = 60, logo BEFD é inscritível.

EB = 2. BD e como ângulo B = 60 então ângulo EDB=90.

Como BEFD é inscritível então ângulo BFE=90 e finalmente ângulo BFC=90

Julio Saldaña


-- Mensaje original ---
De : obm-l@mat.puc-rio.br
Para : obm-l@mat.puc-rio.br
Fecha : Mon, 17 Apr 2017 11:55:34 -0300
Asunto : [obm-l] Geometria

Bom dia a todos,

Gostaria de uma ajuda com o seguinte problema:

Dado um triângulo equilátero ABC, tal que sobre o lado AB  
tenhamos um ponto

E e sobre o AC tenhamos um ponto D, com AE=BD=AB/3. Se as cevianas AD e CE
intersectam  no ponto F, qual a medida do ângulo BFC?

Grato pela atenção.

Abraços,

Marcelo




"Matemática é o alfabeto com o qual Deus escreveu o Universo"
Galileu Galilei

--
Esta mensagem foi verificada pelo sistema de antivírus e
acredita-se estar livre de perigo.


__
Si desea recibir, semanalmente, el Boletín Electrónico de la PUCP, ingrese a:
http://www.pucp.edu.pe/puntoedu/suscribete/


--
Esta mensagem foi verificada pelo sistema de antivírus e
acredita-se estar livre de perigo.

=
Instruções para entrar na lista, sair da lista e usar a lista em
http://www.mat.puc-rio.br/~obmlistas/obm-l.html
=





--
Esta mensagem foi verificada pelo sistema de antiv�rus e
acredita-se estar livre de perigo.

=
Instru��es para entrar na lista, sair da lista e usar a lista em
http://www.mat.puc-rio.br/~obmlistas/obm-l.html
=


[obm-l] Re: [obm-l] Geometria

2017-04-17 Por tôpico Julio César Saldaña



Imagino que D esteja sobre BC. Se for esse o caso:

ABD e AEC são congruentes.

Ángulo BAD = ángulo ECA e por isso ângulo DFC = 60, logo BEFD é inscritível.

EB = 2. BD e como ângulo B = 60 então ângulo EDB=90.

Como BEFD é inscritível então ângulo BFE=90 e finalmente ângulo BFC=90

Julio Saldaña


-- Mensaje original ---
De : obm-l@mat.puc-rio.br
Para : obm-l@mat.puc-rio.br
Fecha : Mon, 17 Apr 2017 11:55:34 -0300
Asunto : [obm-l] Geometria

Bom dia a todos,

Gostaria de uma ajuda com o seguinte problema:

Dado um triângulo equilátero ABC, tal que sobre o lado AB tenhamos um ponto
E e sobre o AC tenhamos um ponto D, com AE=BD=AB/3. Se as cevianas AD e CE
intersectam  no ponto F, qual a medida do ângulo BFC?

Grato pela atenção.

Abraços,

Marcelo




"Matemática é o alfabeto com o qual Deus escreveu o Universo"
Galileu Galilei

--
Esta mensagem foi verificada pelo sistema de antivírus e
acredita-se estar livre de perigo.




__
Si desea recibir, semanalmente, el Boletín Electrónico de la PUCP, ingrese a:
http://www.pucp.edu.pe/puntoedu/suscribete/


--
Esta mensagem foi verificada pelo sistema de antivírus e
acredita-se estar livre de perigo.

=
Instruções para entrar na lista, sair da lista e usar a lista em
http://www.mat.puc-rio.br/~obmlistas/obm-l.html
=


[obm-l] Re: [obm-l] Geometria Analítica em 3 dimensões

2015-10-30 Por tôpico Rígille Scherrer Borges Menezes
Hmmm, me confundi. Mas a equação de um segmento de reta com certeza é:
d(a, x) + d(x, b) = d(a, b)
Onde x é a variável e d(x, y) é a distância entre x e y.

Em sexta-feira, 30 de outubro de 2015, Rígille Scherrer Borges Menezes <
rigillesbmene...@gmail.com> escreveu:

> Vc quer dizer de segmento de reta talveZ? Acho que uma boa ideia é usar a
> desigualdade triangular.
>
> Em sexta-feira, 30 de outubro de 2015, Israel Meireles Chrisostomo <
> israelmchrisost...@gmail.com> escreveu:
>
>> Olá pessoal alguém sabe como provar que a equação da reta é
>> (x_1-x_0)²+(y_1-y_0)²+(z_1-z_0)²=r²? onde r é o comprimento da reta
>>
>> --
>> Esta mensagem foi verificada pelo sistema de antivírus e
>> acredita-se estar livre de perigo.
>
>

-- 
Esta mensagem foi verificada pelo sistema de antiv�rus e
 acredita-se estar livre de perigo.



[obm-l] Re: [obm-l] Geometria Analítica em 3 dimensões

2015-10-30 Por tôpico Sávio Ribas
Mas isso eh uma esfera de raio r (assumindo que x_1, y_1 e z_1 são
variáveis). Eh soh uma aplicação de Pitagoras...

Em 30 de outubro de 2015 14:57, Israel Meireles Chrisostomo <
israelmchrisost...@gmail.com> escreveu:

> Olá pessoal alguém sabe como provar que a equação da reta é
> (x_1-x_0)²+(y_1-y_0)²+(z_1-z_0)²=r²? onde r é o comprimento da reta
>
> --
> Esta mensagem foi verificada pelo sistema de antivírus e
> acredita-se estar livre de perigo.

-- 
Esta mensagem foi verificada pelo sistema de antiv�rus e
 acredita-se estar livre de perigo.



[obm-l] Re: [obm-l] Re: [obm-l] Geometria analítica em Três dimensões

2015-07-23 Por tôpico Israel Meireles Chrisostomo
Ogrigado Ralph, vc sempre respondendo rápido, obrigado mesmo!Vlw, era isso
mesmo o t era fixovlw

Em 23 de julho de 2015 23:04, Ralph Teixeira  escreveu:

> Hm, pera, tem 4 variaveis ai. A letra t representa um numero fixo, e as
> variaveis sao x, y e z? Vou supor que sim, senao eh uma "superficie" em 4
> dimensoes.
>
> Bom, entao a resposta eh sim, representa. Se esta figura tem nome proprio,
> bom, ok, nao sei. :)
>
> Mas notei que se voce botar x=t.sina, y=t.sinb e z=t.sinc e ignorar alguns
> sinais chatos, voce fica com tana.tanb+tanb.tanc+tana.tanc=1, ou seja,
> tana=(tanb.tanc-1)/(tanb+tanc)=-cot(b+c)=tan(b+c-pi/2). Entao sua equacao
> eh "quase" equivalente a
>
> a=k.pi+b+c-pi/2.
>
> E isto eh um plano... Ou seja, sua superficie seria tomar um plano do tipo
> a-b-c=k.pi=pi/2 (ou alguns planos, variando k, e mais alguns variando os
> sinais ali) no espaco abc e entao "senificar" as coordenadas.
>
> Abraco, Ralph.
>
> 2015-07-23 21:15 GMT-03:00 Israel Meireles Chrisostomo <
> israelmchrisost...@gmail.com>:
>
>> Alguém sabe se a equação abaixo representa alguma figura geométrica em 3
>> dimensões?
>>
>> xy/(sqrt{t²-x²}sqrt{t²-y²})+xz/(sqrt{t²-x²}sqrt{t²-z²})+yz/(sqrt{t²-y²}sqrt{t²-z²})=1
>>
>> --
>> Esta mensagem foi verificada pelo sistema de antivírus e
>> acredita-se estar livre de perigo.
>
>
>
> --
> Esta mensagem foi verificada pelo sistema de antivírus e
> acredita-se estar livre de perigo.

-- 
Esta mensagem foi verificada pelo sistema de antiv�rus e
 acredita-se estar livre de perigo.



[obm-l] Re: [obm-l] Geometria analítica em Três dimensões

2015-07-23 Por tôpico Ralph Teixeira
Hm, pera, tem 4 variaveis ai. A letra t representa um numero fixo, e as
variaveis sao x, y e z? Vou supor que sim, senao eh uma "superficie" em 4
dimensoes.

Bom, entao a resposta eh sim, representa. Se esta figura tem nome proprio,
bom, ok, nao sei. :)

Mas notei que se voce botar x=t.sina, y=t.sinb e z=t.sinc e ignorar alguns
sinais chatos, voce fica com tana.tanb+tanb.tanc+tana.tanc=1, ou seja,
tana=(tanb.tanc-1)/(tanb+tanc)=-cot(b+c)=tan(b+c-pi/2). Entao sua equacao
eh "quase" equivalente a

a=k.pi+b+c-pi/2.

E isto eh um plano... Ou seja, sua superficie seria tomar um plano do tipo
a-b-c=k.pi=pi/2 (ou alguns planos, variando k, e mais alguns variando os
sinais ali) no espaco abc e entao "senificar" as coordenadas.

Abraco, Ralph.

2015-07-23 21:15 GMT-03:00 Israel Meireles Chrisostomo <
israelmchrisost...@gmail.com>:

> Alguém sabe se a equação abaixo representa alguma figura geométrica em 3
> dimensões?
>
> xy/(sqrt{t²-x²}sqrt{t²-y²})+xz/(sqrt{t²-x²}sqrt{t²-z²})+yz/(sqrt{t²-y²}sqrt{t²-z²})=1
>
> --
> Esta mensagem foi verificada pelo sistema de antivírus e
> acredita-se estar livre de perigo.

-- 
Esta mensagem foi verificada pelo sistema de antiv�rus e
 acredita-se estar livre de perigo.



[obm-l] Re: {Disarmed} Re: [obm-l] Re: {Disarmed} Re: [obm-l] Re: [obm-l] Geometria plana

2015-03-04 Por tôpico Julio César Saldaña



Muito boa, vou guardar.

Obrigado

Julio Saldaña


-- Mensaje original ---
De : obm-l@mat.puc-rio.br
Para : obm-l@mat.puc-rio.br
Fecha : Tue, 3 Mar 2015 22:13:54 -0300
Asunto : {Disarmed} Re: [obm-l] Re: {Disarmed} Re: [obm-l] Re: [obm-l] Geometria
plana

Vou compartilhar uma para termos soluções alternativas:
1)Circunscreva um círculo ao triângulo ABC.
2) Prolongue AD até tocar o círculo em F.
3)Trace de B para F e de C para F.
4)Encontre BFA=AFC=90-(BAC)/2
5)Como FA é uma bissetriz teremos BF=2FC.
6)Como BEF é isósceles,  tome um ponto M médio  de BF e Trace EM.
7)Os triângulos EMF e EFC são congruentes,  assim FEC=(BAC) /2

Douglas Oliveira.
Em 03/03/2015 16:04, "Julio César Saldaña" 
escreveu:




Isso mesmo, M �ponto medio de BE,

obrigado


Julio Salda馻


-- Mensaje original ---
De : obm-l@mat.puc-rio.br
Para : obm-l@mat.puc-rio.br
Fecha : Tue, 3 Mar 2015 15:33:26 -0300
Asunto : {Disarmed} Re: [obm-l] Re: [obm-l] Geometria plana
>Bela solu莽茫o.
>
>houve s贸 um pequeno erro de digita莽茫o : M 茅 ponto m茅dio de BE, ok ?
>
>Pacini
>
>Em 3 de mar莽o de 2015 11:53, Julio C茅sar Salda帽a 
>escreveu:
>
>>
>>
>> Fiz assim, mas cuidado, costumo me equivocar muito. Podem verificar?
>>
>> Notar que >
>> Seja N de AC tal que DN 茅 paralelo 脿 AB, ent茫o DN=NC e AN=2.DN
>>
>> Como os tri芒ngulos ABE e ADN s茫o semelhantes ent茫o BE=2.AE
>>
>> Seja M o ponto medio de AE, ent茫o BM=ME=AE, e >
>> Os tri芒ngulos BAM e EAC s茫o congruentes, por tanto igualamos 芒ngulos
>> externos
>> respectivos: >
>>
>> Julio Salda帽a
>>
>>
>> -- Mensaje original ---
>> De : obm-l@mat.puc-rio.br
>> Para : obm-l@mat.puc-rio.br
>> Fecha : Mon, 2 Mar 2015 09:23:52 -0300
>> Asunto : [obm-l] Geometria plana
>> >Ol脙隆,  bom dia quero compartilhar uma boa quest脙拢o de geometria

com os

>> >senhores,
>> >Q1) Num tri脙垄ngulo is脙鲁sceles ABC com AB=AC,  toma-se um ponto D no
lado
>> BC
>> >de forma que BD=2CD e um ponto E em AD tal que os 脙垄ngulos BAC e BED
sejam
>> >iguais a 80 graus,   encontrar o valor do 脙垄ngulo DEC.
>> >
>> >Douglas Oliveira.
>> >
>> >--
>> >Esta mensagem foi verificada pelo sistema de antiv铆rus e
>> > acredita-se estar livre de perigo.
>> >
>>
>>
>> __
>> Si desea recibir, semanalmente, el Bolet铆n Electr贸nico de la PUCP,
ingrese
>> a:
>> http://www.pucp.edu.pe/puntoedu/suscribete/
>>
>>
>> --
>> Esta mensagem foi verificada pelo sistema de antiv铆rus e
>>  acredita-se estar livre de perigo.
>>
>>
=
>> Instru莽玫es para entrar na lista, sair da lista e usar a lista em
>> http://www.mat.puc-rio.br/~obmlistas/obm-l.html
>>
=
>>
>
>--
>Esta mensagem foi verificada pelo sistema de antiv韗us e
> acredita-se estar livre de perigo.
>


__
Si desea recibir, semanalmente, el Bolet韓 Electr髇ico de la PUCP, ingrese a:
http://www.pucp.edu.pe/puntoedu/suscribete/


--
Esta mensagem foi verificada pelo sistema de antiv韗us e
 acredita-se estar livre de perigo.

=
Instru珲es para entrar na lista, sair da lista e usar a lista em
http://www.mat.puc-rio.br/~obmlistas/obm-l.html
=



--
Esta mensagem foi verificada pelo sistema de antivírus e
acredita-se estar livre de perigo.




__
Si desea recibir, semanalmente, el Boletín Electrónico de la PUCP, ingrese a:
http://www.pucp.edu.pe/puntoedu/suscribete/


--
Esta mensagem foi verificada pelo sistema de antivírus e
acredita-se estar livre de perigo.

=
Instruções para entrar na lista, sair da lista e usar a lista em
http://www.mat.puc-rio.br/~obmlistas/obm-l.html
=


{Disarmed} Re: [obm-l] Re: {Disarmed} Re: [obm-l] Re: [obm-l] Geometria plana

2015-03-03 Por tôpico Douglas Oliveira de Lima
Vou compartilhar uma para termos soluções alternativas:
1)Circunscreva um círculo ao triângulo ABC.
2) Prolongue AD até tocar o círculo em F.
3)Trace de B para F e de C para F.
4)Encontre BFA=AFC=90-(BAC)/2
5)Como FA é uma bissetriz teremos BF=2FC.
6)Como BEF é isósceles,  tome um ponto M médio  de BF e Trace EM.
7)Os triângulos EMF e EFC são congruentes,  assim FEC=(BAC) /2

Douglas Oliveira.
Em 03/03/2015 16:04, "Julio César Saldaña" 
escreveu:

>
>
> Isso mesmo, M �ponto medio de BE,
>
> obrigado
>
>
> Julio Salda馻
>
>
> -- Mensaje original ---
> De : obm-l@mat.puc-rio.br
> Para : obm-l@mat.puc-rio.br
> Fecha : Tue, 3 Mar 2015 15:33:26 -0300
> Asunto : {Disarmed} Re: [obm-l] Re: [obm-l] Geometria plana
> >Bela solu莽茫o.
> >
> >houve s贸 um pequeno erro de digita莽茫o : M 茅 ponto m茅dio de BE, ok ?
> >
> >Pacini
> >
> >Em 3 de mar莽o de 2015 11:53, Julio C茅sar Salda帽a 
> >escreveu:
> >
> >>
> >>
> >> Fiz assim, mas cuidado, costumo me equivocar muito. Podem verificar?
> >>
> >> Notar que  >>
> >> Seja N de AC tal que DN 茅 paralelo 脿 AB, ent茫o DN=NC e AN=2.DN
> >>
> >> Como os tri芒ngulos ABE e ADN s茫o semelhantes ent茫o BE=2.AE
> >>
> >> Seja M o ponto medio de AE, ent茫o BM=ME=AE, e  >>
> >> Os tri芒ngulos BAM e EAC s茫o congruentes, por tanto igualamos 芒ngulos
> >> externos
> >> respectivos:  >>
> >>
> >> Julio Salda帽a
> >>
> >>
> >> -- Mensaje original ---
> >> De : obm-l@mat.puc-rio.br
> >> Para : obm-l@mat.puc-rio.br
> >> Fecha : Mon, 2 Mar 2015 09:23:52 -0300
> >> Asunto : [obm-l] Geometria plana
> >> >Ol脙隆,  bom dia quero compartilhar uma boa quest脙拢o de geometria com os
> >> >senhores,
> >> >Q1) Num tri脙垄ngulo is脙鲁sceles ABC com AB=AC,  toma-se um ponto D no
> lado
> >> BC
> >> >de forma que BD=2CD e um ponto E em AD tal que os 脙垄ngulos BAC e BED
> sejam
> >> >iguais a 80 graus,   encontrar o valor do 脙垄ngulo DEC.
> >> >
> >> >Douglas Oliveira.
> >> >
> >> >--
> >> >Esta mensagem foi verificada pelo sistema de antiv铆rus e
> >> > acredita-se estar livre de perigo.
> >> >
> >>
> >>
> >> __
> >> Si desea recibir, semanalmente, el Bolet铆n Electr贸nico de la PUCP,
> ingrese
> >> a:
> >> http://www.pucp.edu.pe/puntoedu/suscribete/
> >>
> >>
> >> --
> >> Esta mensagem foi verificada pelo sistema de antiv铆rus e
> >>  acredita-se estar livre de perigo.
> >>
> >>
> =
> >> Instru莽玫es para entrar na lista, sair da lista e usar a lista em
> >> http://www.mat.puc-rio.br/~obmlistas/obm-l.html
> >>
> =
> >>
> >
> >--
> >Esta mensagem foi verificada pelo sistema de antiv韗us e
> > acredita-se estar livre de perigo.
> >
>
>
> __
> Si desea recibir, semanalmente, el Bolet韓 Electr髇ico de la PUCP, ingrese a:
> http://www.pucp.edu.pe/puntoedu/suscribete/
>
>
> --
> Esta mensagem foi verificada pelo sistema de antiv韗us e
>  acredita-se estar livre de perigo.
>
> =
> Instru珲es para entrar na lista, sair da lista e usar a lista em
> http://www.mat.puc-rio.br/~obmlistas/obm-l.html
> =
>

-- 
Esta mensagem foi verificada pelo sistema de antiv�rus e
 acredita-se estar livre de perigo.



[obm-l] Re: {Disarmed} Re: [obm-l] Re: [obm-l] Geometria plana

2015-03-03 Por tôpico Julio César Saldaña



Isso mesmo, M é ponto medio de BE,

obrigado


Julio Saldaña


-- Mensaje original ---
De : obm-l@mat.puc-rio.br
Para : obm-l@mat.puc-rio.br
Fecha : Tue, 3 Mar 2015 15:33:26 -0300
Asunto : {Disarmed} Re: [obm-l] Re: [obm-l] Geometria plana

Bela solução.

houve só um pequeno erro de digitação : M é ponto médio de BE, ok ?

Pacini

Em 3 de março de 2015 11:53, Julio César Saldaña 
escreveu:




Fiz assim, mas cuidado, costumo me equivocar muito. Podem verificar?

Notar que Olá,  bom dia quero compartilhar uma boa questão de geometria com os
>senhores,
>Q1) Num triângulo isósceles ABC com AB=AC,  toma-se um ponto D no lado
BC
>de forma que BD=2CD e um ponto E em AD tal que os ângulos BAC e BED sejam
>iguais a 80 graus,   encontrar o valor do ângulo DEC.
>
>Douglas Oliveira.
>
>--
>Esta mensagem foi verificada pelo sistema de antivírus e
> acredita-se estar livre de perigo.
>


__
Si desea recibir, semanalmente, el Boletín Electrónico de la PUCP, ingrese
a:
http://www.pucp.edu.pe/puntoedu/suscribete/


--
Esta mensagem foi verificada pelo sistema de antivírus e
 acredita-se estar livre de perigo.

=
Instruções para entrar na lista, sair da lista e usar a lista em
http://www.mat.puc-rio.br/~obmlistas/obm-l.html
=



--
Esta mensagem foi verificada pelo sistema de antivírus e
acredita-se estar livre de perigo.




__
Si desea recibir, semanalmente, el Boletín Electrónico de la PUCP, ingrese a:
http://www.pucp.edu.pe/puntoedu/suscribete/


--
Esta mensagem foi verificada pelo sistema de antivírus e
acredita-se estar livre de perigo.

=
Instruções para entrar na lista, sair da lista e usar a lista em
http://www.mat.puc-rio.br/~obmlistas/obm-l.html
=


{Disarmed} Re: [obm-l] Re: [obm-l] Geometria plana

2015-03-03 Por tôpico Pacini Bores
Bela solução.

houve só um pequeno erro de digitação : M é ponto médio de BE, ok ?

Pacini

Em 3 de março de 2015 11:53, Julio César Saldaña 
escreveu:

>
>
> Fiz assim, mas cuidado, costumo me equivocar muito. Podem verificar?
>
> Notar que 
> Seja N de AC tal que DN é paralelo à AB, então DN=NC e AN=2.DN
>
> Como os triângulos ABE e ADN são semelhantes então BE=2.AE
>
> Seja M o ponto medio de AE, então BM=ME=AE, e 
> Os triângulos BAM e EAC são congruentes, por tanto igualamos ângulos
> externos
> respectivos: 
>
> Julio Saldaña
>
>
> -- Mensaje original ---
> De : obm-l@mat.puc-rio.br
> Para : obm-l@mat.puc-rio.br
> Fecha : Mon, 2 Mar 2015 09:23:52 -0300
> Asunto : [obm-l] Geometria plana
> >Olá,  bom dia quero compartilhar uma boa questão de geometria com os
> >senhores,
> >Q1) Num triângulo isósceles ABC com AB=AC,  toma-se um ponto D no lado
> BC
> >de forma que BD=2CD e um ponto E em AD tal que os ângulos BAC e BED sejam
> >iguais a 80 graus,   encontrar o valor do ângulo DEC.
> >
> >Douglas Oliveira.
> >
> >--
> >Esta mensagem foi verificada pelo sistema de antivírus e
> > acredita-se estar livre de perigo.
> >
>
>
> __
> Si desea recibir, semanalmente, el Boletín Electrónico de la PUCP, ingrese
> a:
> http://www.pucp.edu.pe/puntoedu/suscribete/
>
>
> --
> Esta mensagem foi verificada pelo sistema de antivírus e
>  acredita-se estar livre de perigo.
>
> =
> Instruções para entrar na lista, sair da lista e usar a lista em
> http://www.mat.puc-rio.br/~obmlistas/obm-l.html
> =
>

-- 
Esta mensagem foi verificada pelo sistema de antiv�rus e
 acredita-se estar livre de perigo.



[obm-l] Re: [obm-l] Geometria plana

2015-03-03 Por tôpico Julio César Saldaña



Fiz assim, mas cuidado, costumo me equivocar muito. Podem verificar?

Notar que Seja M o ponto medio de AE, então BM=ME=AE, e 

Os triângulos BAM e EAC são congruentes, por tanto igualamos ângulos externos
respectivos: 
Olá,  bom dia quero compartilhar uma boa questão de geometria com os
senhores,
Q1) Num triângulo isósceles ABC com AB=AC,  toma-se um ponto D no lado BC
de forma que BD=2CD e um ponto E em AD tal que os ângulos BAC e BED sejam
iguais a 80 graus,   encontrar o valor do ângulo DEC.

Douglas Oliveira.

--
Esta mensagem foi verificada pelo sistema de antivírus e
acredita-se estar livre de perigo.




__
Si desea recibir, semanalmente, el Boletín Electrónico de la PUCP, ingrese a:
http://www.pucp.edu.pe/puntoedu/suscribete/


--
Esta mensagem foi verificada pelo sistema de antivírus e
acredita-se estar livre de perigo.

=
Instruções para entrar na lista, sair da lista e usar a lista em
http://www.mat.puc-rio.br/~obmlistas/obm-l.html
=


Re: [obm-l] Re: [obm-l] Geometria Plana

2014-05-23 Por tôpico Raphael Aureliano
Valeu pessoal,  obrigado.

Raphael Aureliano

Praticante de Oficial de Náutica (Piloto)
Guarda-Marinha (RM-2)
Em 23/05/2014 11:26, "Julio César Saldaña" 
escreveu:

>
>
> Seja M a interseção de BC com a circunferência, então AM é altura. Então
>  
> Devido a ter os mesmos ângulos, os triângulos BEC e MEC são semeljantes,
> então
> EC / 1 = 2/ EC, por tanto EC = sqrt(2).
>
> Julio Saldaña
>
>
> -- Mensaje original ---
> De : obm-l@mat.puc-rio.br
> Para : obm-l@mat.puc-rio.br
> Fecha : Fri, 23 May 2014 00:46:24 -0300
> Asunto : [obm-l] Geometria Plana
> >Olá,
> >Alguém pode me ajudar no exercício que segue
> >
> >Seja ABC um triângulo isósceles, com AB=AC. Com centro no ponto médio
> de
> >AC, traça-se uma circunferência de diâmetro AB. Por B, traçamos uma
> altura
> >do triângulo, que intercepta a circunferência em E. Sabendo que BC=2,
> >determine o valor de CE.
> >
> >Desde já, agradeço pela devida atenção
> >
> >--
> >Esta mensagem foi verificada pelo sistema de antivírus e
> > acredita-se estar livre de perigo.
> >
>
>
> __
> Si desea recibir, semanalmente, el Boletín Electrónico de la PUCP, ingrese
> a:
> http://www.pucp.edu.pe/puntoedu/suscribete/
>
>
> --
> Esta mensagem foi verificada pelo sistema de antivírus e
>  acredita-se estar livre de perigo.
>
> =
> Instruções para entrar na lista, sair da lista e usar a lista em
> http://www.mat.puc-rio.br/~obmlistas/obm-l.html
> =
>

-- 
Esta mensagem foi verificada pelo sistema de antiv�rus e
 acredita-se estar livre de perigo.



[obm-l] Re: [obm-l] Geometria Plana

2014-05-23 Por tôpico Julio César Saldaña



Seja M a interseção de BC com a circunferência, então AM é altura. Então 
Olá,
Alguém pode me ajudar no exercício que segue

Seja ABC um triângulo isósceles, com AB=AC. Com centro no ponto médio de
AC, traça-se uma circunferência de diâmetro AB. Por B, traçamos uma altura
do triângulo, que intercepta a circunferência em E. Sabendo que BC=2,
determine o valor de CE.

Desde já, agradeço pela devida atenção

--
Esta mensagem foi verificada pelo sistema de antivírus e
acredita-se estar livre de perigo.




__
Si desea recibir, semanalmente, el Boletín Electrónico de la PUCP, ingrese a:
http://www.pucp.edu.pe/puntoedu/suscribete/


--
Esta mensagem foi verificada pelo sistema de antivírus e
acredita-se estar livre de perigo.

=
Instruções para entrar na lista, sair da lista e usar a lista em
http://www.mat.puc-rio.br/~obmlistas/obm-l.html
=


[obm-l] Re: [obm-l] Geometria Plana - Relações Trigonométricas

2014-04-29 Por tôpico luiz silva
So vale ressaltar que no caso, 

12 divide xy

e

5 divide xyz

Abs
Felipe


Em Terça-feira, 29 de Abril de 2014 1:42, Listeiro 037 
 escreveu:
 

Eu tinha umas relações da forma (ab+ac+bc)/abc com alturas e senos, mas
não sei onde guardei. Sobre as ternas:

Sabe-se que

(m²-n²)² + (2mn)² = (m²+n²)²

Seja a=(m²-n²), b=2mn e c = (m²+n²)

Divisibilidade por 4:

Para m par e n par é automático 4|abc
Para m ímpar e n par, 4|2mn, então 4|abc
Para m ímpar e n ímpar, é garantido que m² e n² são divisíveis por 2
(melhor, por 4 ja que ambos são da forma 4k+1 m² é côngruo a n² módulo
4), como b é divisível por 2 fica 4|abc.

(2k+1)² = 4k²+4k+1 = 4(k²+k)+1 -> 4r+1

Divisibilidade por 3:

Caso em que a ou b é da forma 3k é automático 3|abc.

Caso em que a ou b são da forma 3k+1 ou 3k+2:7

(3k+1)² = 9k²+6k+1 -> 3(3k²+2k)+1 -> 3r+1
(3k+2)² = 9k²+12k+4 -> 3(3k²+4k+1)+1 -> 3r+1

Ou seja, a² é côngruo com b² módulo 3.

m²-n² garante 3|abc.

Divisibilidade por 5:

Caso em que a ou b é da forma 5k é automático 5|abc.

Caso em que a ou b são da forma 5k+1 ou 5k+4:
(5k+1)² = 25k²+10k+1 -> 5(5k²+2k)+1 -> 5r+1
(5k+4)² = 25k²+40k+16 -> 5(5k²+8k+3)+1 -> 5r+1

Caso em que a ou b são da forma 5k+2 ou 5k+3:
(5k+2)² = 25k²+20k+4 -> 5(5k²+4k)+4 -> 5s+4
(5k+3)² = 25k²+30k+9 -> 5(5k²+6k+1)+4 -> 5s+4

Para o caso de a e b serem da forma 5k+1 ou 5k+4, m² é côngruo a n²
módulo 5, logo m²-n² garante 5|abc.

Para o caso de a e b serem da forma 5k+2 ou 5k+3, m² é côngruo a n²
módulo 5, logo m²-n² garante 5|abc.

No caso de m² ser incôngruo a n², temos que suas somas são côngruas
módulo 5. Um é da forma 5k+1 ou 5k+4 e o outro é da forma 5k+2 ou 5k+3.
Logo um deles assume a forma 5r+1 e o outro a forma 5s+4 oui
vice-versa. Portanto m²+n² garante 5|abc.

Portanto 3.4.5 = 30|abc sendo a,b,c uma terna pitagórica.



Em Mon, 28 Apr 2014 19:31:59 -0700 (PDT)
luiz silva  escreveu:

> Ola Pessoal,
> 
> Eu não sei se já postei isso aqui, mas trabalhando em alguns
> problemas, encontrei algumas coisas interessantes :
> 
> A) Relações Trigonométrica entre os ângulos de um triângulo
> qualquer (fiz os cálculos usando um triangulo acutângulo qqer de
> lados x,y e z) 
> 1) Cos2X + Cos2Y + Cos2Z +
> 2CosXCosYCosZ = 1
>  
> Quando um dos ângulos é 90º , a relação se reduz a :
>  
> Cos2X + Cos2Y  = 1
>  
> Como X+Y = 90º
>  
> Cos2X + Sen2X  = 1
>  
> De (1),  resultam as seguintes relações :
>  
> 2) Cos2Y + Cos2Z + 2CosXCosYCosZ = Sen2X
>  
> 3) Cos2X + Cos2Z + 2CosXCosYCosZ = Sen2Y
>  
> 4) Cos2X + Cos2Y + 2CosXCosYCosZ = Sen2Z
>  
> 5) 4R2 (Cos2X + Cos2Y + 2CosXCosYCosZ)
> = z2 
> 
> E as outras relações envolvendo R e x  e R e y
> 
>  
> R raio do círculo circunscrito e x,y e z lados do triangulo.
>  
> 6) 2 = Sen2X + Sen2Y + Sen2Z
> - 2CosXCosYCosZ
> 
> 6) 1 + Sen2X + Cos2X  = Sen2X + Sen2Y + Sen2Z - 2CosXCosYCosZ
> 
>  
> Pela lei dos Senos, temos que SenX, SenY e SenZ formam um
> triangulo semelhante ao triângulo de lados x, y e z. Dessa forma,
> temos : 
> Sen2Z = Sen2X + Sen2Y - 2SenXSenYCosZ 
>  
> De (4) temos que :
>  
> Sen2Z = Cos2X + Cos2Y + 2CosXCosYCosZ 
>  
> Ou seja, os triângulos de lado SenX, SenY e SenZ e CosX,
> CosY e SenZ formam um quadrilátero inscritível com diagonal SenZ, em
> um cíuculo cujo raio R = ½
> 
> A) Ternos Pitagóricos Primitivos
> 
> Dado o terno pitagórico a,b e c,  3 x 4 x 5 = 60 divide abc
> 
> Eu procurei na internet e não achei essas relações. Vcs sabem de
> alguma coisa?
> 
> Abs
> Felipe





-- 
Esta mensagem foi verificada pelo sistema de antiv�us e
acredita-se estar livre de perigo.


=
Instru�es para entrar na lista, sair da lista e usar a lista em
http://www.mat.puc-rio.br/~obmlistas/obm-l.html

=
-- 
Esta mensagem foi verificada pelo sistema de antiv�rus e
 acredita-se estar livre de perigo.



[obm-l] Re: [obm-l]Geometria - OBM2012 - Terceira Fase - Nível 2

2014-04-06 Por tôpico PONCE
 Amigo Raphael,
 Vai abaixo uma resolução simplificada.Inicialmente,prova-se
facilmente,EB = EC  e  BEC = 36 graus.Devido a simétria, em
relação a mediatriz do lado CD, conclui-se que o triangulo BME  é
equilátero.Dai   EC = EB =EM e, portanto, conclui-se que E é o
centro de uma circunferência que passa pelos pontos  M,B e C. 
 Das propriedades de ângulos inscrito e central de uma
circunferência, tem-se:  2 BMC = BEC
= 36 graus, o que implica  BMC = 18 graus.
 Ainda da simétria mencionada acima, EMD = BMC = 18
graus.Consequentemente, do vértice M do triãngulo equilátero MNP,  
 CMD = 60 - EMD - BMC = 60 - 36 =
24 graus.RESPOSTA: 24 grausDesculpe-me por qualquer falha e a
resolução simplificada acima.Do sempre amigo
 LUIZ PONCE 
 On Dom 06/04/14 12:15 , Raphael Aureliano raphael0...@gmail.com sent:
Boa tarde pessoal, 
 Alguém poderia me ajudar no problema que segue abaixo?   

Seja ABCDE um pentágono regular inscrito em um triângulo equilatero
MNP, determine o ângulo CMD. 

Na figura, CD está em NP,  B em MN e E em MP.  

Obrigado pela atenção  

Cordialmente,  

Raphael Aureliano 

Praticante de Oficial de Náutica (Piloto)
 Guarda-Marinha (RM-2) 
 -- 
 Esta mensagem foi verificada pelo sistema de antivírus e  
  acredita-se estar livre de perigo. 

-- 
Esta mensagem foi verificada pelo sistema de antiv�rus e
 acredita-se estar livre de perigo.



[obm-l] Re: [obm-l] Geometria

2013-03-13 Por tôpico Julio César Saldaña



Vamos supor que AB=c é o maior dos lados.

Se desde o vértice C desenhamos um diámtro CP, teremos que PA**2 = 4R**2-b**2, e
também que PB**2 = 4R**2-a**2, logo, no triángulo APB temos que a soma dos
quadrados de dois lados é: PA**2+PB**2=8R**2-a**2-b**2, que segundo dado do
problema é igual a c**2, ou seja: PA**2+PB**2 = AB**2, logo o triángulo APB é
retângulo e portanto AB é diâmetro, ouseja o ABC também é retángulo.

cofesso que não estou convencido com a minha solução, acho que é incompleta.

Julio Saldaña


-- Mensaje original ---
De : obm-l@mat.puc-rio.br
Para : obm-l@mat.puc-rio.br
Fecha : Wed, 13 Mar 2013 18:25:40 +
Asunto : [obm-l] Geometria

Saja um triângulo cujos lados medem a,b e c e R o raio da circunferência

circunscrita.Mostre que a^2 + b^2 + c^2 = 8R^2 se,e somente se,o triângulo é
retângulo. Se o triangulo é retangulo,considerando a < = b < c,temos que a^2 +
b^2 = c^2a^2 + b^2 + c^2= 2c^2 e,como c = 2R,segue quea^2 + b^2 + c^2 = 2.(2R)^2
= 8R^2 Estou tentando a segunda parte da demonstração e não sai. :  		 	   		  

--
Esta mensagem foi verificada pelo sistema de antivírus e
acredita-se estar livre de perigo.




__
Si desea recibir, semanalmente, el Boletín Electrónico de la PUCP, ingrese a:
http://www.pucp.edu.pe/puntoedu/suscribete/

=
Instruções para entrar na lista, sair da lista e usar a lista em
http://www.mat.puc-rio.br/~obmlistas/obm-l.html
=


[obm-l] RE: [obm-l] Re: [obm-l] Geometria(questão meio estranha)

2012-11-01 Por tôpico marcone augusto araújo borges

Existe,teoricamente.
 > Date: Thu, 1 Nov 2012 08:21:14 -0400
> Subject: [obm-l] Re: [obm-l] Geometria(questão meio estranha)
> From: bernardo...@gmail.com
> To: obm-l@mat.puc-rio.br
> 
> 2012/11/1 marcone augusto araújo borges :
> > Se todos os lados de um triangulo forem maiores do que 1000 cm ,sua área
> > pode ser menor que 1cm^2 ?
> > Como responder?
> Bom, esse tipo de problema merece um chute. Chute uma das respostas
> (sim ou não) e tente ver se dá. Nesse caso, o mais fácil de "testar" é
> a resposta sim: bastaria achar um triângulo satisfazendo todas essas
> condições. Para o "não", você teria que provar que qualquer que serja
> a configuração, não funciona. E ter tentado responder o "sim" pode
> ajudar.
> 
> Dica: a área tem a ver com os lados (lembre da fórmula
> p(p-a)(p-b)(p-c) = A^2, ou alguma coisa assim). Mas área é
> principalmente base*altura/2. A base é um lado, que é maior do que
> 1000 cm. Se a área for < 1 cm^2, então a altura é < 2/1000 cm. Tem um
> triângulo assim?
> 
> Abraços,
> -- 
> Bernardo Freitas Paulo da Costa
> 
> =
> Instruções para entrar na lista, sair da lista e usar a lista em
> http://www.mat.puc-rio.br/~obmlistas/obm-l.html
> =
  

[obm-l] RE: [obm-l] Re: [obm-l] Geometria( Questão di ficil(?))

2012-11-01 Por tôpico marcone augusto araújo borges

Que interessante!Obrigado!
 > From: saldana...@pucp.edu.pe
> To: obm-l@mat.puc-rio.br
> CC: 
> Subject: [obm-l] Re: [obm-l] Geometria( Questão di ficil(?))
> Date: Thu, 1 Nov 2012 11:24:24 -0500
> 
> 
> 
> Esta vez va en español,
> 
> por el punto C trace una recta paralela a AD. Sea P el punto de intersección 
> de
> esa recta con la recta que contiene a los puntos A y B. Entonces por el 
> teorema
> de thales: AB/BP = BD/BC, es decir ABxBC = BDxBP, por lo tanto los triángulos
> ABC y PBD son equivalentes. Si M es el punto medio de BP, el área del 
> triángulo
> BMD es la mitad del área del triángulo BPD, es decir, será igual a la mitad 
> del
> área del triángulo ABC. Por lo tanto la recta buscada es MD.
> 
> Si alguien pudiera traducir?
> 
> 
> Julio Saldaña
> 
> 
> -- Mensaje original ---
> De : obm-l@mat.puc-rio.br
> Para : obm-l@mat.puc-rio.br
> Fecha : Thu, 1 Nov 2012 11:43:49 +
> Asunto : [obm-l] Geometria( Questão di ficil(?))
> >
> >
> >
> >
> >Dado um triangulo ABC e D um ponto do lado BC,determine a reta passando por D
> que separa op triangulo em duas regiões de igual área. Se D for ponto
> médio,claro que a reta procurada contem a mediana de BC.No mais...é socorro
> mesmo.   
> 
> __
> Si desea recibir, semanalmente, el Boletín Electrónico de la PUCP, ingrese a:
> http://www.pucp.edu.pe/puntoedu/suscribete/
> 
> =
> Instruções para entrar na lista, sair da lista e usar a lista em
> http://www.mat.puc-rio.br/~obmlistas/obm-l.html
> =
  

Re: [obm-l] Re: [obm-l] Geometria(questão meio estranha)

2012-11-01 Por tôpico J. R. Smolka

Tem triângulos assim, sim.

Fixe qualquer um dos lados como base, e seja b o comprimento deste lado 
(em cm). Trace uma reta paralela à base e chame h à distância desta reta 
à base (também em cm). Todos os triângulos cconstruídos com aquela base 
e vértice oposto sobre a reta paralela terão a mesma área S = bh / 2 (em 
cm²).


Queremos que S < 2 cm², então basta que bh < 4 (independente da medida 
b). Lógico que é perfeitamente possível que b > 1000 cm, desde que h 
seja pequeno o suficiente. Para que os outros dois lados também tenham 
comprimento maior que 1000 cm basta escolher o vértice sobre a reta 
paralela afastado mais que 1000 cm do extremo mais próximo do lado 
escolhido como base.


[ ]'s

J. R. Smolka

Em 01/11/2012 09:21, Bernardo Freitas Paulo da Costa escreveu:

2012/11/1 marcone augusto araújo borges :

Se todos os lados de um triangulo forem maiores do que 1000 cm ,sua área
pode ser menor que 1cm^2 ?
Como responder?

Bom, esse tipo de problema merece um chute. Chute uma das respostas
(sim ou não) e tente ver se dá. Nesse caso, o mais fácil de "testar" é
a resposta sim: bastaria achar um triângulo satisfazendo todas essas
condições. Para o "não", você teria que provar que qualquer que serja
a configuração, não funciona. E ter tentado responder o "sim" pode
ajudar.

Dica: a área tem a ver com os lados (lembre da fórmula
p(p-a)(p-b)(p-c) = A^2, ou alguma coisa assim). Mas área é
principalmente base*altura/2. A base é um lado, que é maior do que
1000 cm. Se a área for < 1 cm^2, então a altura é < 2/1000 cm. Tem um
triângulo assim?

Abraços,


=
Instruções para entrar na lista, sair da lista e usar a lista em
http://www.mat.puc-rio.br/~obmlistas/obm-l.html
=


[obm-l] Re: [obm-l] Geometria( Questão di ficil(?))

2012-11-01 Por tôpico Julio César Saldaña



Esta vez va en español,

por el punto C trace una recta paralela a AD. Sea P el punto de intersección de
esa recta con la recta que contiene a los puntos A y B. Entonces por el teorema
de thales: AB/BP = BD/BC, es decir ABxBC = BDxBP, por lo tanto los triángulos
ABC y PBD son equivalentes. Si M es el punto medio de BP, el área del triángulo
BMD es la mitad del área del triángulo BPD, es decir, será igual a la mitad del
área del triángulo ABC. Por lo tanto la recta buscada es MD.

Si alguien pudiera traducir?


Julio Saldaña


-- Mensaje original ---
De : obm-l@mat.puc-rio.br
Para : obm-l@mat.puc-rio.br
Fecha : Thu, 1 Nov 2012 11:43:49 +
Asunto : [obm-l] Geometria( Questão di ficil(?))





Dado um triangulo ABC e D um ponto do lado BC,determine a reta passando por D

que separa op triangulo em duas regiões de igual área. Se D for ponto
médio,claro que a reta procurada contem a mediana de BC.No mais...é socorro
mesmo. 		 	   		   


__
Si desea recibir, semanalmente, el Boletín Electrónico de la PUCP, ingrese a:
http://www.pucp.edu.pe/puntoedu/suscribete/

=
Instruções para entrar na lista, sair da lista e usar a lista em
http://www.mat.puc-rio.br/~obmlistas/obm-l.html
=


RE: [obm-l] Re: [obm-l] Geometria

2012-09-14 Por tôpico marcone augusto araújo borges

Obrigado,Julio.Se for possível detalhar um pouco mais a sulução,agradeço.De 
qualquer forma vou procurar a questão nos arquivos da lista.
 > From: saldana...@pucp.edu.pe
> To: obm-l@mat.puc-rio.br
> CC: 
> Subject: [obm-l] Re: [obm-l] Geometria
> Date: Thu, 13 Sep 2012 05:33:50 -0500
> 
> 
> 
> Eu acho que já resolvimos esse problema aquí na lista. Mas vou tentar uma
> solução diferente:
> 
> Chame de M aquele ponto de CD para o qual BM=BC. Então  então DM=BM (=BC).
> 
> Desenhe a linha reta que vai B a M, e na prolongação ubique o ponto T para o
> qual MT=BM (=DM), então  quadrilátero BFDT é, en español, inscriptible. E como ademá  à
> conclusão que M é o centro da circunferencia circunscrita a BFDT. Então
> MF=MB=MT=MD são raios dessa circunferencia. Façendo algumas somas de ângulos,
> poderá descobri que  
> Tomara que estja certo. Me desculpe o portunhol.
> 
> 
> Julio Saldaña
> 
> 
> -- Mensaje original ---
> De : obm-l@mat.puc-rio.br
> Para : obm-l@mat.puc-rio.br
> Fecha : Thu, 13 Sep 2012 01:48:27 +
> Asunto : [obm-l] Geometria
> >
> >Seja ABC um triangulo isosceles com base BC  e BAC mede 20 graus.Seja D um
> ponto do lado AC distinto de A tal que DBC mede 60 graus.
> >Sejam E e F pontos de AB tais que DE é paralelo a BC e DF perpendicular a
> EC.Determine a madida do angulo BCF   
> >
> 
> __
> Si desea recibir, semanalmente, el Boletín Electrónico de la PUCP, ingrese a:
> http://www.pucp.edu.pe/puntoedu/suscribete/
> 
> =
> Instruções para entrar na lista, sair da lista e usar a lista em
> http://www.mat.puc-rio.br/~obmlistas/obm-l.html
> =
  

[obm-l] Re: [obm-l] Geometria

2012-09-13 Por tôpico Julio César Saldaña



Eu acho que já resolvimos esse problema aquí na lista. Mas vou tentar uma
solução diferente:

Chame de M aquele ponto de CD para o qual BM=BC. Então 

Seja ABC um triangulo isosceles com base BC  e BAC mede 20 graus.Seja D um

ponto do lado AC distinto de A tal que DBC mede 60 graus.

Sejam E e F pontos de AB tais que DE é paralelo a BC e DF perpendicular a
EC.Determine a madida do angulo BCF   
		 	   		   


__
Si desea recibir, semanalmente, el Boletín Electrónico de la PUCP, ingrese a:
http://www.pucp.edu.pe/puntoedu/suscribete/

=
Instruções para entrar na lista, sair da lista e usar a lista em
http://www.mat.puc-rio.br/~obmlistas/obm-l.html
=


[obm-l] RE: [obm-l] Re: [obm-l] Geometria(Construção(2))

2012-09-12 Por tôpico Luís Lopes

Sauda,c~oes, oi Ralph, 

Gostei da sua construção do triângulo. Eu começaria 
traçando o Â. Depois a bissetriz etc. Mas a sua 
construção é melhor. 

No quadrilátero APIQ o PÎQ = 180 - Â. Então o 
tamanho do arco PQ não seria 180 - Â ?? 

>Para que a construção funcione, precisamos que Q esteja entre A e C, isto é, 
>que b>r.sin(Â/2)AQ = r.cot(Â/2). Logo, b>r.cot(Â/2) 

[]'s 
Luís 



Date: Tue, 11 Sep 2012 13:47:40 -0300
Subject: [obm-l] Re: [obm-l] Geometria(Construção(2))
From: ralp...@gmail.com
To: obm-l@mat.puc-rio.br

1) Considere o círculo de diâmetro P1P2. Ele contém o vértice A do quadrado... 
Mas, melhor ainda, pense na diagonal AC! Como ela é a bissetriz do ângulo 
P1AP2, então ela passa pelo ponto D1, médio do arco P1P2 daquele círculo (veja 
figura anexa, viva Geogebra!), que é DETERMINADO A PARTIR DE P1 e P2!

Analogamente, você pode encontrar D3, médio do arco P3P4 do círculo de diâmetro 
P3P4. Como D1D3 será a diagonal do quadrado, você pode intersectar esta 
diagonal com os círculos para achar A e C.

Agora repita para B e D, e acabou!
(Se alguém quiser o Geogebra da construção, mando por E-mail -- acho que a 
lista não aceitaria o anexo)
2) Este é bem mais simples:
i) Desenhe o círculo inscrito, marque nele um arco PQ de tamanho 180-2A.ii) 
Trace as tangentes ao círculo por P e Q, intersecte-as, este é o ponto A (note 
que PÂQ=Â, de fato).iii) Agora é só marcar b (a partir de A) em cima da reta AQ 
para achar o ponto C...
iv)... e traçar a tangente ao círculo por C para achar o ponto B (sobre AP).
Para que a construção funcione, precisamos que Q esteja entre A e C, isto é, 
que b>r.sin(Â/2)

Abraço,   Ralph
P.S.: Note que há um outro arco P1P2! O que aconteceria se a gente escolhesse 
D1 como médio desse OUTRO arco, assim como D3, D2 e D4? :) :) :)

2012/9/10 marcone augusto araújo borges 








1)Os pontos P1,P2,P3,P4 pertencem aos lados consecutivos de um quadrado 
ABCD.Construa com régua e compasso o quadrado.Justifique sua construção.
 
 
 .P1
 
   .P2

 
 
 
.P3  
 

 .P4 
 
 2) Construa o triangulo ABC conhecendo o angulo A,o lado b e o raio r do 
círculo inscrito.Justifique.   

  

  

[obm-l] Re: [obm-l] Geometria(Construção(2))

2012-09-11 Por tôpico Ralph Teixeira
1) Considere o círculo de diâmetro P1P2. Ele contém o vértice A do
quadrado... Mas, melhor ainda, pense na diagonal AC! Como ela é a bissetriz
do ângulo P1AP2, então ela passa pelo ponto D1, médio do arco P1P2 daquele
círculo (veja figura anexa, viva Geogebra!), que é DETERMINADO A PARTIR DE
P1 e P2!

Analogamente, você pode encontrar D3, médio do arco P3P4 do círculo de
diâmetro P3P4. Como D1D3 será a diagonal do quadrado, você pode intersectar
esta diagonal com os círculos para achar A e C.

Agora repita para B e D, e acabou!

(Se alguém quiser o Geogebra da construção, mando por E-mail -- acho que a
lista não aceitaria o anexo)

2) Este é bem mais simples:
i) Desenhe o círculo inscrito, marque nele um arco PQ de tamanho 180-2A.
ii) Trace as tangentes ao círculo por P e Q, intersecte-as, este é o ponto
A (note que PÂQ=Â, de fato).
iii) Agora é só marcar b (a partir de A) em cima da reta AQ para achar o
ponto C...
iv)... e traçar a tangente ao círculo por C para achar o ponto B (sobre AP).

Para que a construção funcione, precisamos que Q esteja entre A e C, isto
é, que b>r.sin(Â/2)

Abraço,
   Ralph

P.S.: Note que há um outro arco P1P2! O que aconteceria se a gente
escolhesse D1 como médio desse OUTRO arco, assim como D3, D2 e D4? :) :) :)

2012/9/10 marcone augusto araújo borges 

>  1)Os pontos P1,P2,P3,P4 pertencem aos lados consecutivos de um quadrado
> ABCD.Construa com régua e compasso o quadrado.Justifique sua construção.
>
>
>  .P1
>
>.P2
>
>
>
> .P3
>
>
>  .P4
>
>  2) Construa o triangulo ABC conhecendo o angulo A,o lado b e o raio r do
> círculo inscrito.Justifique.
>
<>

[obm-l] Re:[obm-l] Geometria(Construção(2))

2012-09-11 Por tôpico Eduardo Wilner
Parece que há uma inversão na posição dos pontos, não?

[ ]'s



[obm-l] Re:[obm-l] Re: [obm-l] Geometria Plana - Triângulo

2012-08-31 Por tôpico Eduardo Wilner
Carlos Vitor, poderia explicar por que o quadrilatero ACHE eh ciclico?

Vc. estah considerando EH paralelo a AC? Por que?
 
[ ]'s



[obm-l] Re: [obm-l] Geometria Plana - Triângulo

2012-08-28 Por tôpico Carlos Victor
Olá  Arkon ,
Uma solução é :

Seja O o ortocentro de ABC . Observe que o triângulo AOC é semelhante ao
triângulo OEH  , pois o quadrilátero ACHE é inscritível . Seja x  = EH ,
então 7/x = AO/EO e como OE = OA.cosB . Usando a lei dos cosenos  encontre
cosB = 1/5 e daí x =7/5 , ok ? .Acredito que pensar no círculo dos nove
pontos  pode também resolver .

Confira as contas .

Abraços

Carlos  Victor

Em 28 de agosto de 2012 19:31, arkon  escreveu:

> Pessoal, qual o bizu?
> Â
> Em um triângulo ABC, traçam-se as alturas AH e CE. Se AB=5m, BC=6m e
> AC=7m, calcule EH.
> Â
> (A) 7/5 m (B) 9/5 m (C) 10/7 m (D) 10/3 m (E) 2
> =
> Instruções para entrar na lista, sair da lista e usar a lista em
> http://www.mat.puc-rio.br/~obmlistas/obm-l.html=


Re: [obm-l] Re: [obm-l] Geometria OBM

2011-07-24 Por tôpico Carlos Victor
Linda Solução
 do Julio.

Carlos Victor

2011/7/24 Julio César Saldaña 

>
>
> Uma solução geométrica:
>
> Sabemos que O2A=O2B.
> Prolongue ou estique BO1, (desculpem o protunhol) até um ponto P tal que
> O2P=O2B
> (=O2A). Calculemos uns ángulos: BPO2=20, PO2C=40, AO2P=60, então o
> triángulo
> O2AP é equilátero, ouseja PA=AO2, PAC=30 e a reta AO1 será mediatriz de
> PO2.
> Então PO1=O1O2, então O1O2P=20, finalmente  BO1O2=20+20=40
>
>
>
> Julio Saldaña
>
>
> -- Mensaje original ---
> De : obm-l@mat.puc-rio.br
> Para : obm-l@mat.puc-rio.br
> Fecha : Sun, 24 Jul 2011 19:18:18 -0300
> Asunto : Re: [obm-l] Geometria OBM
> >Olá João ,
> >
> >Houve um erro na digitação : onde está  AO1 =y  lê-se  BO1 =y . ok ? .
> >Desculpe o erro .
> >
> >Abraços
> >
> >Carlos  Victor
> >
> >Em 24 de julho de 2011 19:11, Carlos Victor  >escreveu:
> >
> >> Olá João ,
> >>
> >> Vamos inicialmente a uma solução trigonométrica :
> >>
> >> Seja z o ângulo pedido .Sejam também AB =a ; AO2 = x  e AO1= y.Então
> >> teremos :
> >>
> >> Triang  *BO1O2 : y/sen(160-z) = x/sen*z
> >>
> >> Triang ABO2 : x/sen50 = a/sen80
> >>
> >> Triang BO1A : y/sen80 = a/sen70
> >>
> >> Logo :sen(20+z) = 4cos10.sen20.senz  ; ou  sen(20+z) -senz = 2senz.sen10
> >>
> >> Donde  cos(z+10) = senz ; ou seja  z = 40 ° .
> >>
> >> Tentarei uma solução geométrica . ok ?
> >>
> >> Abraços
> >>
> >> Carlos  Victor
> >>
> >>
> >>
> >>
> >>
> >>
> >> 2011/7/24 João Maldonado 
> >>
> >>>  Inglaterra -- 1970
> >>>
> >>> No triângulo ABC, AB = AC e A=80°,  dado O1 em AC tal que O1BC =  20° e
> O2
> >>> em  BC tal que   CAO2 = 30°,  calcule   BO1O2
> >>>
> >>> Obrigado
> >>> João
> >>>
> >>
> >>
> >
>
> __
> Si desea recibir, semanalmente, el Boletín Electrónico de la PUCP, ingrese
> a:
> http://www.pucp.edu.pe/puntoedu/suscribete/
>
> =
> Instruções para entrar na lista, sair da lista e usar a lista em
> http://www.mat.puc-rio.br/~obmlistas/obm-l.html
> =
>


[obm-l] Re: [obm-l] Geometria OBM

2011-07-24 Por tôpico Julio César Saldaña



Uma solução geométrica:

Sabemos que O2A=O2B.
Prolongue ou estique BO1, (desculpem o protunhol) até um ponto P tal que O2P=O2B
(=O2A). Calculemos uns ángulos: BPO2=20, PO2C=40, AO2P=60, então o triángulo
O2AP é equilátero, ouseja PA=AO2, PAC=30 e a reta AO1 será mediatriz de PO2.
Então PO1=O1O2, então O1O2P=20, finalmente  BO1O2=20+20=40



Julio Saldaña


-- Mensaje original ---
De : obm-l@mat.puc-rio.br
Para : obm-l@mat.puc-rio.br
Fecha : Sun, 24 Jul 2011 19:18:18 -0300
Asunto : Re: [obm-l] Geometria OBM

Olá João ,

Houve um erro na digitação : onde está  AO1 =y  lê-se  BO1 =y . ok ? .
Desculpe o erro .

Abraços

Carlos  Victor

Em 24 de julho de 2011 19:11, Carlos Victor escreveu:


Olá João ,

Vamos inicialmente a uma solução trigonométrica :

Seja z o ângulo pedido .Sejam também AB =a ; AO2 = x  e AO1= y.Então
teremos :

Triang  *BO1O2 : y/sen(160-z) = x/sen*z

Triang ABO2 : x/sen50 = a/sen80

Triang BO1A : y/sen80 = a/sen70

Logo :sen(20+z) = 4cos10.sen20.senz  ; ou  sen(20+z) -senz = 2senz.sen10

Donde  cos(z+10) = senz ; ou seja  z = 40 ° .

Tentarei uma solução geométrica . ok ?

Abraços

Carlos  Victor






2011/7/24 João Maldonado 


 Inglaterra -- 1970

No triângulo ABC, AB = AC e A=80°,  dado O1 em AC tal que O1BC =  20° e O2
em  BC tal que   CAO2 = 30°,  calcule   BO1O2

Obrigado
João







__
Si desea recibir, semanalmente, el Boletín Electrónico de la PUCP, ingrese a:
http://www.pucp.edu.pe/puntoedu/suscribete/

=
Instruções para entrar na lista, sair da lista e usar a lista em
http://www.mat.puc-rio.br/~obmlistas/obm-l.html
=


[obm-l] Re: [obm-l] Geometria - qual a menor distância

2011-05-05 Por tôpico Bernardo Freitas Paulo da Costa
2011/5/3 João Maldonado :
> Olá colegas  da lista
Oi João e colegas da obm-l.

> Proponho outro problema a vocês
> Também do livro de  Geometria de Morgado:
> Dois A e  B pontos estão sobre o plano. Determine o ponto M sobre uma reta
> qualquer r tal que AM + MB seja mínimo.
Esse exercício é de que capítulo?

> Usei derivada e achei
> x=h(H+h)/d
> sendo x =  MA, h a distância de A a r e H a distância de B a r.
Quem é d?

> Logo um meio de encontrar  M é  traçar AB, achar o ponto médio P, traçar
> PX (X sendo o ponto de intersecção de um segmento perpendicular a r que
> passa por A), e encontrar Y sobre PX tal que a distância de Y a r é igual  a
> h , o ponto de intersecção  da reta perpendicular a r que passa por Y é M.
Caramba, eu não sei como você tirou essa conclusão da fórmula acima,
mas até parece que dá certo.

> Dado isso tentei resolver o problema novamente usndo apenas geometria básica
> (sem derivar), mas não cheguei  a nada. Alguém pode me ajudaar?
Se você já viu elipses, tem uma idéia: MA + MB é a soma da distância
de M aos focos A e B. Assim, se você quer minimizar a distância, você
quer um ponto que esteja na "menor elipse possível" de focos A e B, e
na reta r. Assim, a reta r é tangente à essa elipse. Agora, traçando
as linhas MA e MB, como M está numa elipse, os ângulos de "incidência
e reflexão" são iguais. Traçando as perpendiculares a r por A e B,
(com pés X e Z) você obtém uma semelhança de triângulos AMX e BMZ que
mostra que AM / h = MB / H e principalmente MX / h = MZ / H e daí você
sabe posicionar M no intervalo XZ em proporção h:H, ou seja M = (X * H
+ Z * h)/(h+H).

> []'s
>
> João

Abraços,
-- 
Bernardo Freitas Paulo da Costa

=
Instruções para entrar na lista, sair da lista e usar a lista em
http://www.mat.puc-rio.br/~obmlistas/obm-l.html
=


[obm-l] Re:[obm-l] Geometria - qual a menor distância

2011-05-04 Por tôpico Eduardo Wilner
Esta eh a lei da reflexão, na optica, e a demonstração mais simples é a do 
Teixeira.

[ ]'s


[obm-l] Re: [obm-l] Geometria - qual a menor distância

2011-05-03 Por tôpico Ralph Teixeira
(Suponho que A e B estao do mesmo lado de r)

Seja A' o simetrico de A com relacao aa reta r.

Note que, dado um ponto S qualquer na reta r, o comprimento do caminho
poligonal ASB eh igual ao comprimento do caminho poligonal A'SB.

Assim, minimizar ASB eh o mesmo que minimizar A'SB. Mas o menor
caminho de A` a B eh um segmento de reta, que corta r no ponto pedido
M.

Abraco,
Ralph

2011/5/3 João Maldonado :
> Olá colegas  da lista
>
> Obrigado  pela resolução anterior, já descobri porque não consegui resolver
> (é difícil  mesmo)
>
> Proponho outro problema a vocês
> Também do livro de  Geometria de Morgado:
> Dois A e  B pontos estão sobre o plano. Determine o ponto M sobre uma reta
> qualquer r tal que AM + MB seja mínimo.
>
> Usei derivada e achei
> x=h(H+h)/d -> sendo x =  MA, h a distância de A a r e H a distância de B a
> r. Logo um meio de encontrar  M é  traçar AB, achar o ponto médio P, traçar
> PX (X sendo o ponto de intersecção de um segmento perpendicular a r que
> passa por A), e encontrar Y sobre PX tal que a distância de Y a r é igual  a
> h , o ponto de intersecção  da reta perpendicular a r que passa por Y é M.
>
> Dado isso tentei resolver o problema novamente usndo apenas geometria básica
> (sem derivar), mas não cheguei  a nada. Alguém pode me ajudaar?
> []'s
>
> João

=
Instruções para entrar na lista, sair da lista e usar a lista em
http://www.mat.puc-rio.br/~obmlistas/obm-l.html
=


[obm-l] Re: [obm-l] Geometria

2011-04-26 Por tôpico Julio César Saldaña



Dado que  QC=BC

Seja R um ponto de PB tal que RC=CB. Então  
triângulo RQC é equilátero => RQ=QC=RC=BC. De outro lado, como 
PR=RC, logo PR=RQ, ou seja que o triângulo PRQ é isósceles. Como 

O seguinte problema está no livro  Geometria I de Morgado, e não sei porque 

não estou conseguindo resolvê-lo. Sei que a resposta é 30º, se alguém  puder
ajudar fico grato.


Em um triângulo isósceles ABC, se base BC, o ângulo  vale 20º. P é um ponto

sobre AB tal que o ângulo PCB = 60º. Q é um ponto em AC tal que QBC = 50º. Qual
a medida do ângulo CPQ?
[]\'sJoão 		 	   		   


__
Si desea recibir, semanalmente, el Boletín Electrónico de la PUCP, ingrese a:
http://www.pucp.edu.pe/puntoedu/suscribete/

=
Instruções para entrar na lista, sair da lista e usar a lista em
http://www.mat.puc-rio.br/~obmlistas/obm-l.html
=


[obm-l] Re: [obm-l] Geometria Analítica

2011-02-24 Por tôpico Ralph Teixeira
(Tecnicamente, soh o ramo com x<3)

2011/2/24 Ralph Teixeira :
> Como o titulo eh Geometria Analitica -- seja C=(x,y). Note que o
> triangulo tem que ser agudo em B -- entao x<3.
>
> Agora
> tan CAB = y/x
> tan CBA = y/(3-x)
>
> Agora use que tan2z=2tanz/(1-(tanz)^2). Entao se CAB=2CBA faca as 
> contas
>
> ...dah uma hiperbole.
>
> Abraco,
>         Ralph
>
> 2011/2/24 Vinícius Harlock :
>> Os extremos da base de um triângulo são A(0,0) e B(3,0). Determinar a
>> equação do lugar geométrico do vértice oposto C se este se move de maneira
>> que o ângulo da base CAB é sempre igual a duas vezes o ângulo da base CBA.
>>
>

=
Instruções para entrar na lista, sair da lista e usar a lista em
http://www.mat.puc-rio.br/~obmlistas/obm-l.html
=


[obm-l] Re: [obm-l] Geometria Analítica

2011-02-24 Por tôpico Ralph Teixeira
Como o titulo eh Geometria Analitica -- seja C=(x,y). Note que o
triangulo tem que ser agudo em B -- entao x<3.

Agora
tan CAB = y/x
tan CBA = y/(3-x)

Agora use que tan2z=2tanz/(1-(tanz)^2). Entao se CAB=2CBA faca as contas

...dah uma hiperbole.

Abraco,
 Ralph

2011/2/24 Vinícius Harlock :
> Os extremos da base de um triângulo são A(0,0) e B(3,0). Determinar a
> equação do lugar geométrico do vértice oposto C se este se move de maneira
> que o ângulo da base CAB é sempre igual a duas vezes o ângulo da base CBA.
>

=
Instruções para entrar na lista, sair da lista e usar a lista em
http://www.mat.puc-rio.br/~obmlistas/obm-l.html
=


[obm-l] Re: [obm-l] Re: [obm-l] GEOMETRIA PLANA DEMONSTRAÇ ÃO

2010-12-21 Por tôpico Marcelo Costa
Muitíssimo obrigado e boas festas!


Em 20 de dezembro de 2010 23:11, Eduardo Beltrao  escreveu:

> Prezado Marcelo,
> Após algum tempo solucionando o problema proposto, cheguei a uma
> resposta muito próxima da que você postou aqui. A solução transcrevo abaixo,
> porém peço para que verifique se o resultado correto é realmente (OG)^2 =
> R^2 - 1/3*(A^2 + B^2 + C^2), e não (OG)^2 = R^2 - 1/9*(A^2 + B^2 + C^2).
>
> Atenciosamente,
>
> Eduardo Beltrão
>
> _
>
> Sejam AB = c, AC = b e BC = a os lados do triângulo ABC.
> Sejam M, N e P os pontos médios de BC, AC e AB, respectivamente.
>
> OBS: Para efeito de visualização, considere BC o lado do triângulo mais
> próximo do centro O do círculo.
> Observe que o triângulo OMC é retângulo, e assim:
> (OM)^2 + (CM)^2 = (OC)^2  ( I )
>
> No triângulo AMC temos que, pela lei dos cossenos:
> (AC)^2 = (AM)^2 + (CM)^2 - 2*(AM)*(CM)*cos(A^MC)  ( II )
>
> Também pela lei dos cossenos, temos, no triângulo ABM, que:
> (AB)^2 = (AM)^2 + (BM)^2 - 2*(AM)*(BM)*cos(180º - A^MC)  ( III )
>
> Em ( III ), como M é ponto médio de BC temos:
> (AB)^2 = (AM)^2 + (CM)^2 + 2*(AM)*(CM)*cos(A^MC)  ( IV )
>
> Somando membro a membro as equações ( II ) e ( IV ), temos:
> (AC)^2 + (AB)^2 = 2*(AM)^2 + 2*(CM)^2
> (AM)^2 = [(AC)^2 + (AB)^2 - 2*(CM)^2]/2  ( V )
>
> Como G é baricentro do triângulo ABC, então:
> GM = (AM)/3  ( VI )
>
> No triângulo OBM temos, pela lei dos cossenos:
> (OA)^2 = (OM)^2 + (AM)^2 - 2*(OM)*(AM)*cos(O^MA)  ( VII )
>
>  Também pela lei dos cossenos, no triângulo OGM, temos:
> (OG)^2 = (OM)^2 + (GAM)^2 - 2*(OM)*(GM)*cos(O^MG)  ( VIII )
>
> Observe que os ângulos O^MA e O^MG são iguais, pois A e G são pontos do
> mesmo segmento AM. Assim, manipulando as equações (VII) e (VIII) temos:
> [(OM)^2 + (AM)^2 - (OA)^2]/AM = [(OM)^2 + (GM)^2 - (OG)^2]/GM  ( IX
> )
>
> Substituindo (I) e (VI) em (IX), temos:
> (OC)^2 - (CM)^2 + (AM)^2 - (OA)^2 = 3*[(OC)^2 - (CM)^2 + ((AM)/3)^2 -
> (OG)^2]
> (OC)^2 - (CM)^2 + (AM)^2 - (OA)^2 = 3*(OC)^2 - 3*(CM)^2 + [(AM)^2]/3 -
> 3*(OG)^2
>
> Como OA = OC = R, temos:
> R^2 - (CM)^2 + (AM)^2 - R^2 = 3*(R^2) - 3*(CM)^2 + [(AM)^2]/3 - 3*(OG)^2
>  (AM)^2 - (CM)^2 = 3*(R^2) - 3*(CM)^2 + [(AM)^2]/3 - 3*(OG)^2  ( X
> )
>
> por fim, substituindo (V) em (X), temos:
> [(AC)^2 + (AB)^2 - 2*(CM)^2]/2 - (CM)^2 = 3*(R^2) - 3*(CM)^2 + [(AC)^2 +
> (AB)^2 - 2*(CM)^2]/6 - 3*(OG)^2
>
> Manipulando a equação acima, de modo a isolar o termo (OC)^2, temos que:
> (OG)^2 = R^2 - (a^2 + b^2 + c^2)/9
>
>
> Em 17 de dezembro de 2010 07:39, Marcelo Costa escreveu:
>
> CONSIDERE UM TRIÂNGULO OBTUSÂNGULO ABC, CUJOS OS LADOS MEDEM A, B e C,
>> INSCRITO NUM CÍRUCULO DE RAIO R E CENTRO O.
>> SENDO G O BARICENTRO DO TRIÂNGULO ABC, MOSTRE QUE:
>> (OG)^2 = R^2 - 1/3*(A^2 + B^2 + C^2)
>>
>>
>> AGRADEÇO DESDE JÁ A ATENÇÃO DOS COLEGAS, OBRIGADO!
>>
>
>


[obm-l] Re: [obm-l] GEOMETRIA PLANA DEMONSTRAÇÃO

2010-12-20 Por tôpico Eduardo Beltrao
Prezado Marcelo,
Após algum tempo solucionando o problema proposto, cheguei a uma
resposta muito próxima da que você postou aqui. A solução transcrevo abaixo,
porém peço para que verifique se o resultado correto é realmente (OG)^2 =
R^2 - 1/3*(A^2 + B^2 + C^2), e não (OG)^2 = R^2 - 1/9*(A^2 + B^2 + C^2).

Atenciosamente,

Eduardo Beltrão

_

Sejam AB = c, AC = b e BC = a os lados do triângulo ABC.
Sejam M, N e P os pontos médios de BC, AC e AB, respectivamente.

OBS: Para efeito de visualização, considere BC o lado do triângulo mais
próximo do centro O do círculo.
Observe que o triângulo OMC é retângulo, e assim:
(OM)^2 + (CM)^2 = (OC)^2  ( I )

No triângulo AMC temos que, pela lei dos cossenos:
(AC)^2 = (AM)^2 + (CM)^2 - 2*(AM)*(CM)*cos(A^MC)  ( II )

Também pela lei dos cossenos, temos, no triângulo ABM, que:
(AB)^2 = (AM)^2 + (BM)^2 - 2*(AM)*(BM)*cos(180º - A^MC)  ( III )

Em ( III ), como M é ponto médio de BC temos:
(AB)^2 = (AM)^2 + (CM)^2 + 2*(AM)*(CM)*cos(A^MC)  ( IV )

Somando membro a membro as equações ( II ) e ( IV ), temos:
(AC)^2 + (AB)^2 = 2*(AM)^2 + 2*(CM)^2
(AM)^2 = [(AC)^2 + (AB)^2 - 2*(CM)^2]/2  ( V )

Como G é baricentro do triângulo ABC, então:
GM = (AM)/3  ( VI )

No triângulo OBM temos, pela lei dos cossenos:
(OA)^2 = (OM)^2 + (AM)^2 - 2*(OM)*(AM)*cos(O^MA)  ( VII )

 Também pela lei dos cossenos, no triângulo OGM, temos:
(OG)^2 = (OM)^2 + (GAM)^2 - 2*(OM)*(GM)*cos(O^MG)  ( VIII )

Observe que os ângulos O^MA e O^MG são iguais, pois A e G são pontos do
mesmo segmento AM. Assim, manipulando as equações (VII) e (VIII) temos:
[(OM)^2 + (AM)^2 - (OA)^2]/AM = [(OM)^2 + (GM)^2 - (OG)^2]/GM  ( IX
)

Substituindo (I) e (VI) em (IX), temos:
(OC)^2 - (CM)^2 + (AM)^2 - (OA)^2 = 3*[(OC)^2 - (CM)^2 + ((AM)/3)^2 -
(OG)^2]
(OC)^2 - (CM)^2 + (AM)^2 - (OA)^2 = 3*(OC)^2 - 3*(CM)^2 + [(AM)^2]/3 -
3*(OG)^2

Como OA = OC = R, temos:
R^2 - (CM)^2 + (AM)^2 - R^2 = 3*(R^2) - 3*(CM)^2 + [(AM)^2]/3 - 3*(OG)^2
 (AM)^2 - (CM)^2 = 3*(R^2) - 3*(CM)^2 + [(AM)^2]/3 - 3*(OG)^2  ( X )

por fim, substituindo (V) em (X), temos:
[(AC)^2 + (AB)^2 - 2*(CM)^2]/2 - (CM)^2 = 3*(R^2) - 3*(CM)^2 + [(AC)^2 +
(AB)^2 - 2*(CM)^2]/6 - 3*(OG)^2

Manipulando a equação acima, de modo a isolar o termo (OC)^2, temos que:
(OG)^2 = R^2 - (a^2 + b^2 + c^2)/9


Em 17 de dezembro de 2010 07:39, Marcelo Costa escreveu:

> CONSIDERE UM TRIÂNGULO OBTUSÂNGULO ABC, CUJOS OS LADOS MEDEM A, B e C,
> INSCRITO NUM CÍRUCULO DE RAIO R E CENTRO O.
> SENDO G O BARICENTRO DO TRIÂNGULO ABC, MOSTRE QUE:
> (OG)^2 = R^2 - 1/3*(A^2 + B^2 + C^2)
>
>
> AGRADEÇO DESDE JÁ A ATENÇÃO DOS COLEGAS, OBRIGADO!
>


[obm-l] Re: [obm-l] Re: [obm-l] Geometria (ângulos) bem int eressante!

2010-04-27 Por tôpico Marcelo Costa
obrigado!
Faltou "enxergar" o triângulo isósceles!!

Em 21 de abril de 2010 12:20, Eduardo Wilner
escreveu:

> Ocorreu uma rotação de 90° em torno do vértice C. Assim, o triângulo BEC é
> isoceles e retângulo, logo o < CBE = 45° 
>
> Abraços
>
> Wilner
>
>
> --- Em *qua, 21/4/10, Marcelo Costa * escreveu:
>
>
> De: Marcelo Costa 
> Assunto: [obm-l] Geometria (ângulos) bem interessante!
> Para: obm-l@mat.puc-rio.br
> Data: Quarta-feira, 21 de Abril de 2010, 6:59
>
>
>
> Temos um triângulo ABC, com base AC, onde CAB = 40°, CBA = 60° e BCA = 80°.
> Constuimos um triângulo CDE, congruente ao triângulo ABC, de maneira que ele
> seria o ABC girando sobre o vértice C de tal modo que BCD = 10°. Traçamos
> os segmento BE que intercepta CD no ponto F. O valor do ângulo BFC é em
> graus:
>
> a) 120
> b) 125
> c) 130
> d) 135
>
>
> Gabarito: b
> Agradeço desde já a atenção dos colegas, obrigado!
>
>
>
>
> --
> "Matemática é o alfabeto com o qual Deus escreveu o Universo"
> Galileu Galilei
>
>
>




-- 
"Matemática é o alfabeto com o qual Deus escreveu o Universo"
Galileu Galilei


[obm-l] Re: [obm-l] Geometria (ângulos) bem interessan te!

2010-04-21 Por tôpico Eduardo Wilner
Ocorreu uma rotação de 90° em torno do vértice C. Assim, o triângulo BEC é 
isoceles e retângulo, logo o < CBE = 45° 

Abraços

Wilner  


--- Em qua, 21/4/10, Marcelo Costa  escreveu:

De: Marcelo Costa 
Assunto: [obm-l] Geometria (ângulos) bem interessante!
Para: obm-l@mat.puc-rio.br
Data: Quarta-feira, 21 de Abril de 2010, 6:59


Temos um triângulo ABC, com base AC, onde CAB = 40°, CBA = 60° e BCA = 80°. 
Constuimos um triângulo CDE, congruente ao triângulo ABC, de maneira que ele 
seria o ABC girando sobre o vértice C de tal modo que BCD = 10°. Traçamos os 
segmento BE que intercepta CD no ponto F. O valor do ângulo BFC é em graus:



a) 120
b) 125
c) 130
d) 135


Gabarito: b
Agradeço desde já a atenção dos colegas, obrigado! 







 







-- 
"Matemática é o alfabeto com o qual Deus escreveu o Universo"
Galileu Galilei




  

[obm-l] Re: [obm-l] geometria analítica

2009-10-31 Por tôpico Luciana Rodrigues
 
Carpe Dien
Em 31/10/2009 08:19, Robério Alves < prof_robe...@yahoo.com.br > escreveu:




Como é que resolve essa questão ?Encontre o foco da parábola y = x^2 + 2x + i





Veja quais são os assuntos do momento no Yahoo! + Buscados: Top 10 - Celebridades - Música - Esportes 
=
Instruções para entrar na lista, sair da lista e usar a lista em
http://www.mat.puc-rio.br/~obmlistas/obm-l.html
=


[obm-l] Re: [obm-l] GEOMETRIA COMBINATÓRIA!

2009-06-03 Por tôpico lucianarodriggues
Em 03/06/2009 13:03, Paulo Santa Rita < paulo.santar...@gmail.com > escreveu:
Ola Jorge e demais colegasdesta lista ... OBM-L,>> Dados n pontos no plano (n>=3), o número de distâncias distintas entre eles> é, pelo menos (n-3/4)^1/2-1/2 . (Problema Difícil!)>1) No excerto de mensagem acima voce deve estar se referindo a { [ N -(3/4) ]^(1/2) } - (1/2) e nao a { [ (N - 3 ) /4) ]^(1/2) } - (1/2),pois esta ultima expressao fornece ZERO distancias distintas quandoN=4, o que esta obviamente errado, visto que e facil ver que qualquerconfiguracao de 4 pontos distintos num plano darao origem ao menos aduas distancias distintas.2) Mesmo se a expressao correta for { [ N - (3/4) ]^(1/2) } - (1/2),tambem aqui ocorrem problemas, pois, sendo obvio ululante que talexpressao fornece numeros irracionais para diversos N's, comoentender, por exem
 plo, que 5 pontos ( N=5 ) distintos no plano daraoorigem ao menos a ( sqrt(17) - 1) / 2 distancias distintas ?Voce esta se referindo ao piso de { [ N - (3/4) ]^(1/2) } - (1/2),vale dizer, ao maior inteiro que ultrapassa { [ N - (3/4) ]^(1/2) } -(1/2) ?3) Mesmo se a expressao correta for PISO( { [ N - (3/4) ]^(1/2) } -(1/2) ) ha problemas, pois para N=5 teremos PISO( { [ 5 - (3/4)]^(1/2) } - (1/2) ) = 1. Ora, 5 pontos distintos no plano daraoorigem ao menos a 2 distancias distintas ( um pentagono regularconvexo, por exemplo).Sera TETO( { [ N - (3/4) ]^(1/2) } - (1/2) ), onde TETO(X) e o menorinteiro maior ou igual a X ?Enfim, qual a expressao correta ? O problema pode ser formulado como segue :PROBLEMA : Determine o numero minimo de distancias distintas exibidaspor N pontos distintos de um plano.?Um AbracaoPSR, 40306090D02
 =Instru��es para entrar na lista, sair da lista e usar a lista emhttp://www.mat.puc-rio.br/~obmlistas/obm-l.html=
=
Instruções para entrar na lista, sair da lista e usar a lista em
http://www.mat.puc-rio.br/~obmlistas/obm-l.html
=


[obm-l] Re: [obm-l] GEOMETRIA COMBINATÓRIA!

2009-06-03 Por tôpico Paulo Santa Rita
Ola Jorge e demais colegas
desta lista ... OBM-L,

>
> Dados n pontos no plano (n>=3), o número de distâncias distintas entre eles
> é, pelo menos (n-3/4)^1/2-1/2 . (Problema Difícil!)
>

1) No excerto de mensagem acima voce deve estar se referindo a { [ N -
(3/4) ]^(1/2) } - (1/2) e nao a { [ (N - 3 ) /4) ]^(1/2) } - (1/2),
pois esta ultima expressao fornece ZERO distancias distintas quando
N=4, o que esta obviamente errado, visto que e facil ver que qualquer
configuracao de 4 pontos distintos num plano darao origem ao menos a
duas distancias distintas.

2) Mesmo se a expressao correta for { [ N - (3/4) ]^(1/2) } - (1/2),
tambem aqui ocorrem problemas, pois, sendo obvio ululante que tal
expressao fornece numeros irracionais para diversos N's, como
entender, por exemplo, que 5 pontos ( N=5 ) distintos no plano darao
origem ao menos a ( sqrt(17) - 1) / 2 distancias distintas ?

Voce esta se referindo ao piso de { [ N - (3/4) ]^(1/2) } - (1/2),
vale dizer, ao maior inteiro que ultrapassa { [ N - (3/4) ]^(1/2) } -
(1/2) ?

3) Mesmo se a expressao correta for PISO(  { [ N - (3/4) ]^(1/2) } -
(1/2)  ) ha problemas, pois para N=5 teremos  PISO(  { [ 5 - (3/4)
]^(1/2) } - (1/2)  ) = 1. Ora, 5 pontos distintos no plano darao
origem ao menos a 2 distancias distintas ( um pentagono regular
convexo, por exemplo).

Sera TETO(  { [ N - (3/4) ]^(1/2) } - (1/2)  ), onde TETO(X) e o menor
inteiro maior ou igual a X ?

Enfim, qual a expressao correta ? O problema pode ser formulado como segue :

PROBLEMA : Determine o numero minimo de distancias distintas exibidas
por N pontos distintos de um plano.

?

Um Abracao
PSR, 40306090D02

=
Instru��es para entrar na lista, sair da lista e usar a lista em
http://www.mat.puc-rio.br/~obmlistas/obm-l.html
=


[obm-l] Re: [obm-l] GEOMETRIA COMBINATÓR IA!

2009-06-02 Por tôpico lucianarodriggues
Em 02/06/2009 13:06, Jorge Luis Rodrigues e Silva Luis < jorgelrs1...@hotmail.com > escreveu:

.hmmessage P { margin:0px; padding:0px } body.hmmessage { font-size: 10pt; font-family:Verdana }
Turma! Continuo intrigado com uma possível resolução combinatória proposta pelo colega Fernando já que 9 em cada 10 livros de Análise Combinatória abordam o problema. Quem sabe, não seja um problema típico de Geometria Combinatória? O que acham da probabilidade do encontro ocorrer entre 12 e 13h se nenhum deles espera mais de 10min pelo outro? Alguma dicaDados n pontos no plano, o número máximo de retas distintas que eles determinam é, no máximo, Cn,2. (Demonstre isso!)Dados n pontos no plano (n>=3), o número de distâncias distintas entre eles é, pelo menos (n-3/4)^1/2-1/2 . (Problema Difícil!)Nota: Encontrei na RPM-16 pg.66 a engenhosa resolução do problema proposto pelo colega Nehab. "Dada uma circunferência qualquer, achar o centro da mesma utilizando apenas o compasso". Vale a pena conferir. Incrivelmente Fantástico!Abraços!

Instale o novo Internet Explorer 8 otimizado para o MSN. Download aqui 
=
Instruções para entrar na lista, sair da lista e usar a lista em
http://www.mat.puc-rio.br/~obmlistas/obm-l.html
=


[obm-l] Re: [obm-l] Re: [obm-l] Re: [obm-l] Geometria Plana - 3 problema s clássicos

2009-05-30 Por tôpico lucianarodriggues

Em 26/05/2009 09:00, Fernando Lima Gama Junior < fgam...@gmail.com > escreveu:Começou...
Fernando GamaSent from Brasilia, DF, Brazil 
2009/5/26 
Em 25/05/2009 22:05, Carlos Nehab < ne...@infolink.com.br > escreveu: 
Aos aficcionados:Três problemas clássicos e interessantes de geometria plana:1) Dado um triângulo ABC, identifique o triângulo de perímetro mínimo 

nele inscrito (cada vértice - P, Q e R, em um lado distinto de ABC).2) Determinar o centro de uma circunferência dada utilizando apenas compasso.3) Determinar o ponto médio de um segmento dado, utilizando apenas 

compasso (difícil).Nehab=Instruções para entrar na lista, sair da lista e usar a lista emhttp://www.mat.puc-rio.br/~obmlistas/obm-l.html

== Instruções para entrar na lista, sair da lista e usar a lista em http://www.mat.puc-rio.br/~obmlistas/obm-l.html = 




=
Instruções para entrar na lista, sair da lista e usar a lista em
http://www.mat.puc-rio.br/~obmlistas/obm-l.html
=


[obm-l] Re: [obm-l] Geometria Plana - 3 problemas clássico s

2009-05-26 Por tôpico luiz silva
Olá Carlos,
 
Não sou muito bom nestes tipos de problemas. Porém, com relação ao 3o., dado um 
segmento qqer AB,  não bastaria utilizarmos o procedimento "padrão" para traçar 
mediatriz, só que, ao invés de unirmos os pontos C e D, obtidos com a 
utilização do compasso, traçaríamos a ciscunferência com centro em A ou B e 
tangente ao segmento ?
 
Um outro problema muito legal :
 
Duas circunferências secantes se interceptam nos pontos A e B. Traçar o 
segmento CD, passando por A ( C em uma circunferência e D na outra), de modo 
que os segmentos CA=AD.
 
Abs
Felipe

--- Em seg, 25/5/09, Carlos Nehab  escreveu:


De: Carlos Nehab 
Assunto: [obm-l] Geometria Plana - 3 problemas clássicos
Para: obm-l@mat.puc-rio.br
Data: Segunda-feira, 25 de Maio de 2009, 22:05


Aos aficcionados:

Três problemas clássicos e interessantes de geometria plana:

1) Dado um triângulo ABC, identifique o triângulo de perímetro mínimo nele 
inscrito (cada vértice - P, Q e R, em um lado distinto de ABC).
2) Determinar o centro de uma circunferência dada utilizando apenas compasso.
3) Determinar o ponto médio de um segmento dado, utilizando apenas compasso 
(difícil).

Nehab

=
Instruções para entrar na lista, sair da lista e usar a lista em
http://www.mat.puc-rio.br/~obmlistas/obm-l.html
=



  Veja quais são os assuntos do momento no Yahoo! +Buscados
http://br.maisbuscados.yahoo.com

[obm-l] Re: [obm-l] Re: [obm-l] Geometria Plana - 3 problema s clássicos

2009-05-26 Por tôpico Fernando Lima Gama Junior
Começou...

Fernando Gama

Sent from Brasilia, DF, Brazil

2009/5/26 

>
>
>
> Em 25/05/2009 22:05, *Carlos Nehab < ne...@infolink.com.br >* escreveu:
>
>
> Aos aficcionados:
>
> Três problemas clássicos e interessantes de geometria plana:
>
> 1) Dado um triângulo ABC, identifique o triângulo de perímetro mínimo
> nele inscrito (cada vértice - P, Q e R, em um lado distinto de ABC).
> 2) Determinar o centro de uma circunferência dada utilizando apenas
> compasso.
> 3) Determinar o ponto médio de um segmento dado, utilizando apenas
> compasso (difícil).
>
> Nehab
>
> =
> Instruções para entrar na lista, sair da lista e usar a lista em
> http://www.mat.puc-rio.br/~obmlistas/obm-l.html
> =
>
> =
> Instruções para entrar na lista, sair da lista e usar a lista em
> http://www.mat.puc-rio.br/~obmlistas/obm-l.html=


[obm-l] Re: [obm-l] Geometria Plana - 3 problemas clássicos

2009-05-26 Por tôpico lucianarodriggues

Em 25/05/2009 22:05, Carlos Nehab < ne...@infolink.com.br > escreveu:Aos aficcionados:Três problemas clássicos e interessantes de geometria plana:1) Dado um triângulo ABC, identifique o triângulo de perímetro mínimo nele inscrito (cada vértice - P, Q e R, em um lado distinto de ABC).2) Determinar o centro de uma circunferência dada utilizando apenas compasso.3) Determinar o ponto médio de um segmento dado, utilizando apenas compasso (difícil).Nehab=Instruções para entrar na lista, sair da lista e usar a lista emhttp://www.mat.puc-rio.br/~obmlistas/obm-l.html=
=
Instruções para entrar na lista, sair da lista e usar a lista em
http://www.mat.puc-rio.br/~obmlistas/obm-l.html
=


[obm-l] Re: [obm-l] Geometria Analítica

2009-02-27 Por tôpico silverratio
Olá Manuela,

Problema 1:

Se v = (a, b, c), e v é ortogonal ao eixo Z, então c = 0, pois < v, k > = c,
onde k = ( 0, 0, 1 ).

Além disso, w = ( 0, 2, 3 ), e da equação < v, w > = 6 tiramos que b = 3.

Resta a condição sobre a norma de v. Como agora sabemos que v = ( a, 3, 0 ),

| v | = raiz{ a^2 + 9 + 0 } = 5, o que implica a^2 + 9 = 25, ou seja, a = +4
ou a = -4.

Portanto existem na verdade duas possibilidades para o vetor v: ( 4, 3, 0 )
ou ( -4, 3, 0 ).


Problema 2:

Seja w = (x, y, z). Primeiro, se queremos w ortogonal a v, basta resolver a
equação:

< w, v > = 0, que é simplesmente 2x -y +z = 0.

Tome uma solução não-nula qualquer, por exemplo, x = 0, y = z = 1.

O vetor ( 0, 1, 1 ) é ortogonal a v. Ele só não é unitário, mas isso sempre
pode ser

resolvido divivindo-o pela sua norma, ou seja, escolhendo: ( 0, 1/raiz{2} ,
1/raiz{2} ).


Problema 3:

Vou assumir que estamos falando da projeção ortogonal aqui.

A projeção ortogonal é dada por: ( < u, v > / < v, v > ) * v. Basta
calcular.

Espero ter ajudado.

Abraço,

- Leandro.


Re: [obm-l] RES: [obm-l] Re: [obm-l] geometria olimpíada

2008-04-10 Por tôpico saulo nilson
seja I a intercessao entre OA e SR , o angulo A e 90 graus, os angulos S  e
R sao 45,
tgx/2=RI/OI
diagonal do quadrado=Lsqr2
distancia entre O e I=L/2
o resto...

2008/4/9 João Gabriel Preturlan <[EMAIL PROTECTED]>:

>  Olá!
>
>
>
> Obrigado pela dica foi de grande ajuda!
>
> mas realmente queria saber de onde o Saulo tirou a relação da primeira
> linha da resolução dele estou tentando encontrá-la mas não tenho progresso.
> Se alguém puder me explicar eu agradeço imensamente.
>
>
>
> *tagx/2=rq2-1=rq(1-cosx)/(1+cosx) (não consigo sacar de que lugar vem esta
> relação)*
>
> (1-w)/(1+w)=2-2rq2+1=3-2rq2
>
> 3-2rq2-1=-w(4-2rq2)
>
> w=-(1-rq2)/(2-rq2)=-(2+rq2-2rq2-2)/2=rq2/2
>
> x=45º
>
>
>
> "Se as retas r e s são paralelas e distam L entre si e o quadrado ABCD tem
> lado L também, prove que o ângulo SÔR tem 45 graus."
>
> [image: cid:image001.png@01C8.99F2A080]
>
>
>
>
>
> *De:* [EMAIL PROTECTED] [mailto:[EMAIL PROTECTED] *Em
> nome de *Arconcher
> *Enviada em:* quarta-feira, 9 de abril de 2008 17:25
> *Para:* obm-l@mat.puc-rio.br
> *Assunto:* [obm-l] Re: [obm-l] geometria olimpíada
>
>
>
> Sendo a largura da faixa igual ao lado do quadrado fica fácil de perceber
> que as linhas TS e VR são bissetrizes dos ângulos obtusos formados entre um
> lado do quadrado e uma das retas da faixa, por exemplo: do ponto S baixe
> perpendiculares ao lado AB do quadrado e à reta suporte de VT, tais
> segmentos perpendiculares medem L daí TS é bissetriz do ângulo VTB. De modo
> análogo para os pontos
>
> R,T e V . Aí fica bem fácil concluir que SOR mede 45º.
>
> Agora mostre que a soma dos perímetros dos triângulos
>
> ASR e VTC é constante ( Asian Pasific , não me lembro do ano ). Use as
> bissetrizes anteriores que sai fácil.
>
> Saludos.
>
> Arconcher
>
>
>
> No virus found in this incoming message.
> Checked by AVG.
> Version: 7.5.519 / Virus Database: 269.22.10/1367 - Release Date:
> 09/04/2008 07:10
>
>  No virus found in this outgoing message.
> Checked by AVG.
> Version: 7.5.519 / Virus Database: 269.22.10/1367 - Release Date:
> 09/04/2008 07:10
>
>
<>

[obm-l] Re: [obm-l] geometria olimpíada

2008-04-09 Por tôpico Anderson Weber
Olá

Uma dica para a solução: note que o ponto R é equidistante da reta s e do lado 
CD. Portanto, R pertence à bissetriz do ângulo DVT.

Uma outra maneira de resolver esse lindo problema...

prolongue o lado AD até obter P na intersecção com a reta s.
prolongue o lado CB até obter K na intersecção com a reta r.

Note que o quadrilátero SPTK é um quadrilátero notável (losango) - isso pode 
ser provado por congruência entre triângulos.

Um abraço

Anderson Weber

  - Original Message - 
  From: João Gabriel Preturlan 
  To: obm-l@mat.puc-rio.br 
  Sent: Tuesday, April 08, 2008 5:00 PM
  Subject: [obm-l] geometria olimpíada


  Saudações!

   

  Gostaria que vocês me ajudassem neste problema.

   

  "Se as retas r e s são paralelas e distam L entre si e o quadrado ABCD tem 
lado L também, prove que o ângulo SÔR tem 45 graus."

   



   

  Agradeço muito pela ajuda.

   

  JG.

   

   



  No virus found in this outgoing message.
  Checked by AVG.
  Version: 7.5.519 / Virus Database: 269.22.10/1366 - Release Date: 08/04/2008 
17:03


<>

[obm-l] Re: [obm-l] geometria olimpíada

2008-04-09 Por tôpico Anderson Weber
Olá

Uma dica para a solução: note que o ponto R é equidistante da reta s e do lado 
CD. Portanto, R pertence à bissetriz do ângulo DVT.

Uma outra maneira de resolver esse lindo problema...

prolongue o lado AD até obter P na intersecção com a reta s.
prolongue o lado CB até obter K na intersecção com a reta r.

Note que o quadrilátero SPTK é um quadrilátero notável (losango) - isso pode 
ser provado por congruência entre triângulos.

Um abraço

Anderson Weber
  - Original Message - 
  From: João Gabriel Preturlan 
  To: obm-l@mat.puc-rio.br 
  Sent: Tuesday, April 08, 2008 5:00 PM
  Subject: [obm-l] geometria olimpíada


  Saudações!

   

  Gostaria que vocês me ajudassem neste problema.

   

  "Se as retas r e s são paralelas e distam L entre si e o quadrado ABCD tem 
lado L também, prove que o ângulo SÔR tem 45 graus."

   



   

  Agradeço muito pela ajuda.

   

  JG.

   

   



  No virus found in this outgoing message.
  Checked by AVG.
  Version: 7.5.519 / Virus Database: 269.22.10/1366 - Release Date: 08/04/2008 
17:03


<>

[obm-l] RES: [obm-l] Re: [obm-l] geometria olimpíada

2008-04-09 Por tôpico João Gabriel Preturlan
Olá!

 

Obrigado pela dica foi de grande ajuda!

mas realmente queria saber de onde o Saulo tirou a relação da primeira linha
da resolução dele estou tentando encontrá-la mas não tenho progresso. Se
alguém puder me explicar eu agradeço imensamente.

 

tagx/2=rq2-1=rq(1-cosx)/(1+cosx) (não consigo sacar de que lugar vem esta
relação)

(1-w)/(1+w)=2-2rq2+1=3-2rq2

3-2rq2-1=-w(4-2rq2)

w=-(1-rq2)/(2-rq2)=-(2+rq2-2rq2-2)/2=rq2/2

x=45º

 

"Se as retas r e s são paralelas e distam L entre si e o quadrado ABCD tem
lado L também, prove que o ângulo SÔR tem 45 graus."

cid:image001.png@01C8.99F2A080

 

 

De: [EMAIL PROTECTED] [mailto:[EMAIL PROTECTED] Em nome
de Arconcher
Enviada em: quarta-feira, 9 de abril de 2008 17:25
Para: obm-l@mat.puc-rio.br
Assunto: [obm-l] Re: [obm-l] geometria olimpíada

 

Sendo a largura da faixa igual ao lado do quadrado fica fácil de perceber
que as linhas TS e VR são bissetrizes dos ângulos obtusos formados entre um
lado do quadrado e uma das retas da faixa, por exemplo: do ponto S baixe
perpendiculares ao lado AB do quadrado e à reta suporte de VT, tais
segmentos perpendiculares medem L daí TS é bissetriz do ângulo VTB. De modo
análogo para os pontos

R,T e V . Aí fica bem fácil concluir que SOR mede 45º.

Agora mostre que a soma dos perímetros dos triângulos

ASR e VTC é constante ( Asian Pasific , não me lembro do ano ). Use as
bissetrizes anteriores que sai fácil.

Saludos.

Arconcher

 

No virus found in this incoming message.
Checked by AVG.
Version: 7.5.519 / Virus Database: 269.22.10/1367 - Release Date: 09/04/2008
07:10


No virus found in this outgoing message.
Checked by AVG. 
Version: 7.5.519 / Virus Database: 269.22.10/1367 - Release Date: 09/04/2008
07:10
 
  
<>

[obm-l] Re: [obm-l] geometria olimpíada

2008-04-09 Por tôpico Arconcher
Sendo a largura da faixa igual ao lado do quadrado fica fácil de perceber que 
as linhas TS e VR são bissetrizes dos ângulos obtusos formados entre um lado do 
quadrado e uma das retas da faixa, por exemplo: do ponto S baixe 
perpendiculares ao lado AB do quadrado e à reta suporte de VT, tais segmentos 
perpendiculares medem L daí TS é bissetriz do ângulo VTB. De modo análogo para 
os pontos
R,T e V . Aí fica bem fácil concluir que SOR mede 45º.
Agora mostre que a soma dos perímetros dos triângulos
ASR e VTC é constante ( Asian Pasific , não me lembro do ano ). Use as 
bissetrizes anteriores que sai fácil.
Saludos.
Arconcher


[obm-l] Re: [obm-l] Geometria Analítica

2007-11-04 Por tôpico João Luís Gomes Guimarães
O problema pede um ponto que pertença ao eixo das abcissas e que também seja 
equidistante de A e B;

Então, de todos os pontos que  que sejam equidistantes de A e B (e que você 
encontrou ao resolver a equação d(AP)=d(BP)), basta que você escolha aquele que 
tem ordenada zero (pois se pertence ao eixo das abcissas, tem coordenadas 
(x,0))!

Essa é a interpretação do problema.

Com relação às implicações que você falou, olha só: você disse que "Se o ponto 
P é eqüidistante dos pontos A e B. Logo, ele deve estar entre A e B. Isso 
implicaria em : d(AP)=d(PB)". Vamos escrever isso assim:

P equidistante de A e B => P está entre A e B => d(AP) = d(PB)

Primeiramente, é necessário definir bem aqui o conceito de "estar entre".  
Assim a primeira implicação fica clara. mas a segunda implicação é falsa, 
concorda?  Se "P está entre A e B => d(AP) = d(PB)" não é verdade!!!

Ficou claro? 
  - Original Message - 
  From: araketu 
  To: obm-l@mat.puc-rio.br 
  Sent: Sunday, November 04, 2007 11:44 AM
  Subject: [obm-l] Geometria Analítica


  Deparei-me com a seguinte questão:

  Determinar, no eixo Ox, um ponto P que seja eqüidistante dos pontos A(-1,-2) 
e B(5,-4).
  Solução do livro: O ponto pocurado é do tipo P(x,0). Deve-se ter:

  d(P,A)=d(P,B) =>|PA|=|PB|

  Minha dúvida é: Se o ponto P é eqüidistante dos pontos A e B. Logo, ele deve 
estar entre A e B. Isso implicaria em : d(AP)=d(PB). Só que fiz por esse método 
e não cheguei a solução dada pelo livro que é: x=3.
  Gostaria de saber se essa dúvida foi conceitual ou errei na interpretação da 
questão.

Re: [obm-l] Re: [obm-l] Geometria e aritmética super básicas

2006-08-29 Por tôpico Paulo Cesar
 Sejam x e y os números em questão, tais que x + y = 96. Como MDC{x,y} = 12, podemos escrever x = 12a e y = 12b, com MDC{a,b
} = 1, isto é, a e b são primos entre si.
 
daí temos: 12a + 12b = 96 --> a + b = 8.
 
Foi dito no enunciado que o produto entre x e y deve ser máximo. Para tal, basta dizer que o produto entre a e b deve ser máximo (xy = 144ab).
 
Usaremos agora uma importante desigualdade: Média Aritmética entre a e b maior ou igual do que a Média Geométrica entre a e b. A igualdade só é válida para a = b. 
Fazendo as contas: (a + b)/2 > sqrt(ab) --> 4 > sqrt(ab) --> ab < 16.
Mas no enunciado está explícito que existe um MAIOR número. Logo ab <=15. Como se deseja o produto máximo, temos ab = 15. Agora basta resolver o sistema de variáveis inteiras:
a + b = 8
ab = 15
A solução é a = 5 e b = 3 (ou vice-versa).
Os números procurados são 12.5 e 12.3, ou seja, 60 e 36.
 
Espero ter ajudado
 
Abraços
 
PC


[obm-l] Re: [obm-l] Geometria e aritmética super básicas

2006-08-29 Por tôpico Ricardo



Geometria: 
Aplique potencia de pontos em A, AB^2=AC*AD => 
CD=6
Area(ABC) = AC*CB*sen(BCD)/2, 
Area(BCD)=CD*CB*sen(180-BCD)/2. 
Como sen(180-BCD)=sen(BCD) seque que 
razao=AC/CD=5/6
 
Espero ter ajudado
Ricardo
Abcos

  - Original Message - 
  From: 
  its matematico 
  To: obm-l@mat.puc-rio.br 
  Sent: Tuesday, August 29, 2006 4:59 
  PM
  Subject: [obm-l] Geometria e aritmética 
  super básicas 
  Olá gente,Estou com duas dúvidas super básicas mesmo... 
  Mas aí vão:Primeiro:   Como resolver formalmente:
  Dois números inteiros 
  positivos tem soma 96 e o máximo divisor comum igual  
  a 12. Dar o maior dos dois números sabendo que o produto deles deve ser 
  o maior possível. 
  Eu cheguei a resposta (60 e 
  36) mas a forma como resolvi acho q não é a mais prática e nem a mais 
  elegante, mesmo sendo melhor q por tentativas ; )E a outra é de 
  geometria plana (como é desaconselhável mandar anexos para a lista) está no 
  link:http://br.geocities.com/matematica.italo/duvida_cn.htmlAmbas 
  são do Colégio Naval.Obrigado,Ítalo
  
  
  Novidade no Yahoo! Mail: receba alertas de novas mensagens no seu celular. Registre 
  seu aparelho agora! 
  
  
  Novidade no Yahoo! Mail: receba alertas de novas mensagens no seu celular. Registre 
  seu aparelho agora! 
  
  

  Internal Virus Database is out-of-date.Checked by AVG Free 
  Edition.Version: 7.1.394 / Virus Database: 268.11.5/425 - Release Date: 
  22/8/2006


[obm-l] RE: [obm-l] Geometria Anlítica

2005-04-24 Por tôpico caiosg
se vc quiser girar esse vetor em volta da origem de 60 graus.. trata ele
como complexo e multiplica pelo complexo cis(pi/3)...



 '>'-- Mensagem Original --
 '>'Date: Sun, 24 Apr 2005 18:16:18 -0300 (ART)
 '>'From: Robÿe9rio Alves <[EMAIL PROTECTED]>
 '>'Subject: [obm-l] Geometria Anlítica
 '>'To: obm-l@mat.puc-rio.br
 '>'Reply-To: obm-l@mat.puc-rio.br
 '>'
 '>'
 '>'Se eu tenho o ponto (4,3) e girá-lo 60° anti-horário com mesmo módulo,
qual
 '>'será esse novo ponto?
 '>'
 '>'
 '>'-
 '>'Yahoo! Acesso Grátis: Internet rápida e grátis. Instale o discador agora!



=
Instruções para entrar na lista, sair da lista e usar a lista em
http://www.mat.puc-rio.br/~nicolau/olimp/obm-l.html
=


[obm-l] Re: [obm-l] Geometria Anlítica

2005-04-24 Por tôpico Renato Ghini Bettiol



Basta perceber que se temos o ponto A(4,3), sua 
distância à origem dos eixos é 5. d(A,O)=sqrt(4^2+3^2)=5
considere alpha o angulo entre o segmento AO e o 
eixo das abscissas.
somando 60º à alpha, teremos um segmento novo, BO, 
tal que B é o ponto desejado,
não é muito trabalhoso perceber que B será 
B(0,5).
 
resp: (0,5)
 
Lembre, 3 4 e 5 é uma terna pitagórica e forma, 
necessariamente, triangulos retangulos
 

  - Original Message - 
  From: 
  Robÿe9rio Alves 
  To: obm-l@mat.puc-rio.br 
  Sent: Sunday, April 24, 2005 6:16 
PM
  Subject: [obm-l] Geometria Anlítica
  
  Se eu tenho o ponto (4,3) e girá-lo 60° anti-horário com mesmo módulo, 
  qual será esse novo ponto?
  
  
  Yahoo! 
  Acesso Grátis: Internet rápida e grátis. Instale o discador 
agora!


[obm-l] Re: [obm-l] Geometria Anlítica

2005-04-24 Por tôpico Alvaro Netto



Saudações.
 
(4,3) = 4 + 3.i (1)
cis 60º = 1/2 + i.sqrt(3)/2 (2)
(1).(2): (4 + 3i).(1/2 + i.sqrt(3)/2) = 2 - 
3.sqrt(3)/2 + i.(3/2 + 2.sqrt(3)) =  (2 - 3.sqrt(3)/2, 3/2 + 
2.sqrt(3)).
 
Alvaro.

  - Original Message - 
  From: 
  Robÿe9rio Alves 
  To: obm-l@mat.puc-rio.br 
  Sent: Sunday, April 24, 2005 6:16 
PM
  Subject: [obm-l] Geometria Anlítica
  
  Se eu tenho o ponto (4,3) e girá-lo 60° anti-horário com mesmo módulo, 
  qual será esse novo ponto?
  
  
  Yahoo! 
  Acesso Grátis: Internet rápida e grátis. Instale o discador 
agora!


[obm-l] Re: [obm-l] Geometria II - A.C. Morgado, E. Wagner e M.Jorge - Duas questões conflitantes.

2005-03-14 Por tôpico caiosg
Eu tive o prazer de ter aula com esses 3 grandes professores. Graças a eles
e outros excelentes professores que tive, hj estou no ITA. Um grande abraço
pra eles,

Caio
 '>'-- Mensagem Original --
 '>'Date: Mon, 14 Mar 2005 17:35:23 -0300 (BRT)
 '>'From: Sergio Lima Netto <[EMAIL PROTECTED]>
 '>'To: obm-l@mat.puc-rio.br
 '>'Subject: Re: [obm-l] Geometria II -  A.C. Morgado, E. Wagner e 
 '>' M.Jorge - Duas questões conflitantes.
 '>'Reply-To: obm-l@mat.puc-rio.br
 '>'
 '>'
 '>'
 '>'oi Dymitri,
 '>'O livro que voce esta? seguindo e? excelente.
 '>'Me trouxe muita alegria le-lo. Voce cita duas divergencias,
 '>'mas nao diz quais. Da? para deduzir apenas uma: a questao 34.
 '>'Acessei o site que voce indicou
 '>'e tentei seguir seu raciocinio mas me perdi.
 '>'
 '>'Pelo teorema das bissetrizes:
 '>'
 '>'EA   BA   15
 '>'-- = -- = --
 '>'EC   BC   14
 '>'
 '>'Logo
 '>'
 '>'EA/15 = EC/14 = (EA+EC)/(15+14) = AC/29 = 6/29
 '>'
 '>'pois (EA+EC) = AC
 '>'
 '>'Logo
 '>'
 '>'EA = 90/29 e EC = 84/29
 '>'
 '>'Do triiangulo ABE:
 '>'
 '>'IE   AE   (90/29)   6
 '>'-- = -- = --- = --
 '>'IB   AB15   29
 '>'
 '>'Resp: letra A
 '>'
 '>'Abraco,
 '>'sergio
 '>'
 '>'
 '>'On Mon, 14 Mar 2005, Dymitri Cardoso Leão wrote:
 '>'
 '>'> Estou estudando geometria plana pelo excelente livro Geometria
II,
 '>'de 
 '>'> A.C. Morgado, E. Wagner e M.Jorge. Contudo, encontrei duas divergências
 '>'de 
 '>'> gabarito até agora. As questões e suas respectivas soluções, elaboradas
 '>'por 
 '>'> mim, encontram-se no endereço 
 '>'> http://dymitri.leao.vila.bol.com.br/geometria.htm
 '>'> 
 '>'> OBS: Particularmente a questão 34 deu muito trabalho nas contas, talvez
 '>'pela 
 '>'> estratégia adotada por mim. Citando Carlos Yuzo Shine em seu artigo

 '>'> publicado na Eureka 17: "Nunca perca a fé!!!".
 '>'> 
 '>'>  Gostaria de ver a opinião dos partcipantes desta excelente lista
sobre
 '>'
 '>'> as questões solicitadas.
 '>'> 
 '>'>   Atenciosamente,
 '>'
 '>'> Dymitri Cardoso Leão.
 '>'> 
 '>'> _
 '>'> Chegou o que faltava: MSN Acesso Grátis. Instale Já! 
 '>'> http://www.msn.com.br/discador
 '>'> 
 '>'> =
 '>'> Instruções para entrar na lista, sair da lista e usar a lista em
 '>'> http://www.mat.puc-rio.br/~nicolau/olimp/obm-l.html
 '>'> =
 '>'> 
 '>'
 '>'=
 '>'Instruções para entrar na lista, sair da lista e usar a lista em
 '>'http://www.mat.puc-rio.br/~nicolau/olimp/obm-l.html
 '>'=



=
Instruções para entrar na lista, sair da lista e usar a lista em
http://www.mat.puc-rio.br/~nicolau/olimp/obm-l.html
=


[obm-l] Re: [obm-l] Geometria e Aritmética

2004-07-07 Por tôpico Igor Castro
*1º -> Reta: mx + py + q = 0
Distancia de A(Xa,Ya) à r - > |mXa + pYa + q| / (m^2 + p^2)^1/2
 // B|mXb + pYb + q| / (m^2 +
p^2)^1/2
//  C|mXc + pYc + q| / (m^2 +
p^2)^1/2
Soma das distancias - > | m(Xa + Xb + Xc) + p(Ya + Yb + Yc) + 3q| / (m^2 +
p^2)^1/2
Distancias ao baricentro(Xa+Xb+Xc/3, Ya + Yb + Yc/3) - > | m(Xa + Xb + Xc)/3
+ p(Ya + Yb + Yc)/3 + q| / (m^2 + p^2)^1/2 = 1/3 da soma das distancias.
Acho q eh isso se ñ me enrolei nos modulos..

*2º use o 1º

[]´s
Igor


- Original Message - 
From: "victor machado" <[EMAIL PROTECTED]>
To: <[EMAIL PROTECTED]>
Sent: Wednesday, July 07, 2004 9:19 PM
Subject: [obm-l] Geometria e Aritmética


> Amigos da OBM-l, gostaria de saber a solução dos seguintes exercícios:
>
> - ABC é um triângulo cujo baricentro é G e R é uma reta externa ao
triângulo.
> Demonstrar que a soma das distâncias dos vértices A, B e C do
> triângulo à reta R é igual ao triplo da distância de G a esta mesma
> reta.
>
> - As distâncias dos três vértices de um triângulo a uma reta medem 7m,
> 9, e 14m, respectivamente. Calcular a distância do baricentro do
> triângulo à mesma reta.
>
> - Uma urna contém 100 bolas pretas e 100 bolas brancas. De forma
> repetitiva, retiramos três bolas de urna e as substituírmos por outras
> existentes num monte de bolas fora da urna, da seguintes forma:
>
> BOLAS REMOVIDAS / SUBSTITUÍDAS POR
> 3 pretas / 1 preta
> 2 pretas e 1 branca / 1 preta e 1 branca
> 1 preta e 2 brancas / 2 brancas
> 3 brancas / 1 preta e 1 branca
>
> Qual dos conjuntos de bolas abaixo pode ser o conteúdo da urna após
> isto ser feito muitas vezes ?
>
> A) 2 bolas pretas
> B) 2 bolas brancas
> C) 1 bola preta
> D) 1 bola preta e 1 bola branca
> E) 1 bola branca
>
> - Uma raposa está adiantada 60 pulos seus sobre um cão que a persegue.
> Enquanto que a raposa dá 10 pulos, o cão dá 8; 3 pulos do cão valem 5
> pulos da raposa. Quantos pulos dará o cão para alcançar a raposa ?
>
> - Duas estradas de iguais dimensões começam simultâneamente a ser
> construídas por 15 operários cada uma delas. Mas, exclusivamente
> devido a dificuldades no terreno, percebe-se que enquanto uma turma
> avançous 2/3 na sua obra, a outra avançou 4/5 da sua. O número de
> operários que devemos tirar de uma e por na outra para que as duas
> obras fiquem prontas ao mesmo tempo é ?
>
> Eu sei que são muitas, mas agradeço mesmo assim.
> muito obrigado,
> victor.
>
> =
> Instruções para entrar na lista, sair da lista e usar a lista em
> http://www.mat.puc-rio.br/~nicolau/olimp/obm-l.html
> =

=
Instruções para entrar na lista, sair da lista e usar a lista em
http://www.mat.puc-rio.br/~nicolau/olimp/obm-l.html
=


[obm-l] Re: [obm-l] Geometria e teoria dos números

2004-02-24 Por tôpico Alexandre Augusto da Rocha
2^(2^24) + 1 = 7537 ( mod 1 )
logo a soma dos 4 ultimo algarismos e 22

Tem certeza que o enunciado era esse?  deu um trabalhao pra chegar no resultado
Teorema Chines do Resto, Teorema de Euler e um tanto de braco, quer dizer um
tanto de calculadora.
Fiquei com uma duvida:
Primeiro eu fiz fiz N mod 2 e N mod 5 pra achar o algarismo das unidades  ('mod
10').
Depois vi que com Euler nao ficava dificil achar logo (mod 100) ou (mod 1000)
Foi ate facil chegar que N = 12 (mod 25)  e que N = 37 (mod 125). O problema e
que N = 37 (mod 625) foi bastante trabalhoso.  Tem alguma maneira direta de
tirar N (mod p^m) partindo de N (mod p^(m-1))?

-Auggy

- Original Message -
From: "Pacini bores" <[EMAIL PROTECTED]>
To: <[EMAIL PROTECTED]>
Sent: Monday, February 23, 2004 6:02 AM
Subject: [obm-l] Geometria e teoria dos números


> Olá , obrigado  ao Fábio pelas  soluções  anteriores .
>
> Deculpem na  repetição da  questão (1),mas será  que  ela tem alguma
> resposta inteira ou é impossível  determiná-la?
>
> 1)Considere o retãngulo  ABCD com  
> 2)Determine a soma  dos  quatro últimos  algarismos  do número
>
> 2^(2^24)+1.Consegui  descobri na internet  que  este número não é primo .
>
> []s   Pacini
>
>
>
> =
> Instruções para entrar na lista, sair da lista e usar a lista em
> http://www.mat.puc-rio.br/~nicolau/olimp/obm-l.html
> =
>


=
Instruções para entrar na lista, sair da lista e usar a lista em
http://www.mat.puc-rio.br/~nicolau/olimp/obm-l.html
=


[obm-l] Re: [obm-l] Geometria Analítica

2003-10-28 Por tôpico Daniel Melo Wanzeller



Claudio,
 
    Não é possivel usar o 
baricentro, pois apesar de formar um triangulo, seria necessario que todas as 
medianas tivessem o mesmo comprimento, o que acontece no triangulo 
equilatero.
    Fazendo a distancia entre os 
pontos e um ponto generico (x,y), tem-se:
    x^2 + y^2 = (x-1)^2 + 
(y-2)^2
    x^2 + y^2 = (x-3)^2 + 
(y+1)^2
 
tem-se que x= 3/14 e y = 9/14, o que não é o mesmo 
que o baricentro do trinagulo.
 
    Espero ter ajudado
 
        Daniel 
Wanzeller

  - Original Message - 
  From: 
  Claudio 
  
  To: [EMAIL PROTECTED] 
  Sent: Tuesday, October 28, 2003 9:33 
  AM
  Subject: [obm-l] Geometria 
Analítica
  
  Pessoal nesta questão simples de GA, posso usar o 
  baricentro  para calcular o ponto equidistante?
   
  Veja.
   
  O Unico ponto que é equidistante de (0,0) (1,2) e 
  (3,-1) é?
   
  Desde ja 
agradeço.


[obm-l] Re: [obm-l] Geometria Analítica

2003-10-28 Por tôpico Cláudio \(Prática\)



Oi, chará:
 
Não, pois este ponto não é o baricentro (em geral), 
mas sim o circumcentro.
 
Talvez seja mais fácil calcular o ponto de 
interseccção das mediatrizes de dois dos lados do triângulo que tem estes pontos 
como vértices.
 
Um abraço,
Cláudio.

  - Original Message - 
  From: 
  Claudio 
  
  To: [EMAIL PROTECTED] 
  Sent: Tuesday, October 28, 2003 9:33 
  AM
  Subject: [obm-l] Geometria 
Analítica
  
  Pessoal nesta questão simples de GA, posso usar o 
  baricentro  para calcular o ponto equidistante?
   
  Veja.
   
  O Unico ponto que é equidistante de (0,0) (1,2) e 
  (3,-1) é?
   
  Desde ja 
agradeço.


[obm-l] Re: [obm-l] Geometria Analítica e Plana

2003-09-18 Por tôpico Artur Costa Steiner
Estes enderecos nao existem.
Artur

Olá Pessoal,

Gostaria que alguém me ajudasse nesses dois exercícios:

exercício 1 (geometria analitica)

http://www.paraisodovestibulando.kit.net/questoes/exercic
io_geometria_analitica.htm


exercicio 2 (geometria plana)

http://www.paraisodovestibulando.kit.net/questoes/exercic
io_geometria_area3.htm



Grato

Mr. Crowley

("`-''-/").___..--''"`-._
`6_ 6 ) `-. ( ).`-.__.`)
(_Y_.)' ._ ) `._ `.``-..-'
_..`--'_..-_/ /--'_.' ,'
(il),-'' (li),' ((!.-'


__
Acabe com aquelas janelinhas que pulam na sua tela.
AntiPop-up UOL - É grátis!
http://antipopup.uol.com.br/


=
Instruções para entrar na lista, sair da lista e usar a lista em
http://www.mat.puc-rio.br/~nicolau/olimp/obm-l.html
=


OPEN
Internet
@ Primeiro provedor do DF com anti-vírus no servidor de e-mails @

=
Instruções para entrar na lista, sair da lista e usar a lista em
http://www.mat.puc-rio.br/~nicolau/olimp/obm-l.html
=


[obm-l] Re: [obm-l] geometria plana

2003-09-14 Por tôpico luizhenriquerick
AC = sqrt34  ->>  sqrt34 . PB = 15
PB = 15/sqrt34

81 = 225/34 + AP^2
AP = sqrt2529
AC/AE = 5/AP

AE = (sqrt34 . sqrt2529)/5

Muita conta 

Abraços

Luiz H. Barbosa 





-- Mensagem original --

>Alguém me dá uma luz?
>
>Considere um retângulo ABCD e um ponto E do lado AD. Determine o comprimento
>do segmento AE, sabendo que BE e AC são perpendiculares e que AB = 3 e
AD
>= 5.
>
>Obrigado,
>André Resende




--
Use o melhor sistema de busca da Internet
Radar UOL - http://www.radaruol.com.br



=
Instruções para entrar na lista, sair da lista e usar a lista em
http://www.mat.puc-rio.br/~nicolau/olimp/obm-l.html
=


[obm-l] Re: [obm-l] Re: [obm-l] geometria 2

2003-07-25 Por tôpico yurigomes
 Se naum me engano na notação adotada no problema, o valor de AX é sempre
igual a p-a, onde p é o semiperímetro e a= BC, independente do triângulo.
No caso em que o triângulo, os pontos A, X, I e o outro pto de tangência
da circunferência inscrita a ABC formam um quadrado, e assim os lados são
todos iguais, ou seja, IX= AX. Mas IX=r, donde r= AX= p- a. 
  Para ver que vale AX= p-a, faça o seguinte: sejm Y e Z os outros ptos
de tangência com os lados AB e BC, resp.. Então:
   AX=AY=x ; CX= CZ= z e  BZ= BY= y. Logo, 
  c=AB= AY+ BY= x+ y
  a=BC= BZ+ CZ= y+ z
  b=AC= CX+ AX= z+ x
  Logo, vc fica com o sistema:
   x+y=c
   y+z=a
   x+z=b
 As soluções dão x=(b+c-a)/2= p- a, onde p= (a+b+c)/2 

-- Mensagem original --

>Ola pessoal,
>
>Direi minha duvida no corpo da mensagem. Para o Yuri ou quem souber.
>
>
>Em uma mensagem de 24/7/2003 23:53:41 Hora padrão leste da Am. Sul, 
>[EMAIL PROTECTED] escreveu:
>
>
>> Sejam a e b os comprimentos dos catetos, I o incentro de C1 e X o ponto
>> de tangência de C1 com AC. Então o raio de C é igual a AX, e eh esse
valor
>> vale r= p- Hipotenusa= (a+b-Hipotenusa)/2= [a+b- sqr(a^2+b^2)]/2= k/2
-
>> sqr(a^2+b^2)/2 (NAO ENTENDI POR QUE VC FEZ r= p- hipotenusa, A PRINCIPIO
>
>> PENSEI QUE p FOSSE O SEMI-PERIMETRO E ESTA EQUACAO TIVESSE ORIGEM NA
FORMULA
>DE 
>> HERON ( S (AREA) = P*R) ,ONDE R EH O RAIO DA CIRCUNFERENCIA INSCRITA
NO
>
>> TRIANGULO, MAS ACHO QUE ME ENGANEI. O QUE FOI FEITO ?)
>>   O raio de C2 é a metade da hipotenusa: R= sqr(a^2+b^2)/2. Assim, a
soma
>> dos comprimentos de C1 e C2 é igual a 2pi(R+ r)= pi(k/2 - sqr(a^2+b^2)/2
>> +sqr(a^2+b^2)/2)= 2pi.k/2= pi.k
>> Ateh mais, 
>> Yuri
>> 
>> -- Mensagem original --
>> 
>> >Rodrigo Salcedo, eu aqui de novo!!!
>> >Consideremos um triangulo retangulo que simultaneamente esta circunscrito
>> >à 
>> >circunferencia C1 e inscrito à circunferencia C2 . Sabendo-se que a
soma
>> >dos 
>> >comprimentos dos catetos do triangulo é K  cm, qual sera a soma dos

>> >comprimentos destas duas circunferencias?
>> >
>> >[EMAIL PROTECTED]
>> >
>> 
>
>
>

[]'s, Yuri
ICQ: 64992515


--
Use o melhor sistema de busca da Internet
Radar UOL - http://www.radaruol.com.br



=
Instruções para entrar na lista, sair da lista e usar a lista em
http://www.mat.puc-rio.br/~nicolau/olimp/obm-l.html
=


[obm-l] Re: [obm-l] Re: [obm-l] geometria

2003-07-25 Por tôpico yurigomes
 É só trocar. 
 É que na minha figura M e N ficaram acima de A, qdo a posição correta é
abaixo.

-- Mensagem original --

>Ola pessoal,
>
>No enunciado foi dito que MB= 7 cm e NC= 4 cm, mas na resolucao eh dito
que
>
>MB= 4 e NC = 7. Eh assim mesmo ?
>
>
>
>Em uma mensagem de 24/7/2003 23:44:20 Hora padrão leste da Am. Sul, 
>[EMAIL PROTECTED] escreveu:
>
>
>> Oi Rodrigo,
>> Seja ABC=B e ACB=C. Então NCP= 90- C/2. Como NP//BC, temos CNP=C. Logo,
>> NPC= 180- (C+ 90- C/2)= 90- C/2 => CNP isósceles => NP=NC=7 => MN+ MP=
>7.
>> 
>> De modo análogo, BMP= 180- B e MBP= B/2 => BPM= B/2 => BMP isósceles
=>
>> MP= MB= 4. 
>> Logo, MN= 7- MP= 7- 4 => MN= 3.
>> Ateh mais, 
>> 
>>   Yuri
>> 
>> -- Mensagem original --
>> 
>> >Num triangulo qualquer ABC, traçamos a bissetriz interna de B e a bissetriz
>> >
>> >externa de C. Pelo ponto P de concurso das bissetrizes, traçamos uma
reta
>> >
>> >paralela ao lado BC , encontrando o lado AB no ponto M e AC no ponto
N.
>> 
>> >Calcule o segmento MN sabendo que MB= 7 cm e NC= 4 cm
>> >
>> >[EMAIL PROTECTED]
>
>
>

[]'s, Yuri
ICQ: 64992515


--
Use o melhor sistema de busca da Internet
Radar UOL - http://www.radaruol.com.br



=
Instruções para entrar na lista, sair da lista e usar a lista em
http://www.mat.puc-rio.br/~nicolau/olimp/obm-l.html
=


Re: [obm-l] Re: [obm-l] geometria 2

2003-07-25 Por tôpico Johann Peter Gustav Lejeune Dirichlet
Vamos fazer assim:se o triangulo ABC e retangulo
em A,inraio r,sejam T_a,T_b,T_c as tangencias do
incirculo.


 --- [EMAIL PROTECTED] escreveu: > Ola pessoal,
> 
> Direi minha duvida no corpo da mensagem. Para o
> Yuri ou quem souber.
> 
> 
> Em uma mensagem de 24/7/2003 23:53:41 Hora
> padrão leste da Am. Sul, 
> [EMAIL PROTECTED] escreveu:
> 
> 
> > Sejam a e b os comprimentos dos catetos, I o
> incentro de C1 e X o ponto
> > de tangência de C1 com AC. Então o raio de C
> é igual a AX, e eh esse valor
> > vale r= p- Hipotenusa= (a+b-Hipotenusa)/2=
> [a+b- sqr(a^2+b^2)]/2= k/2 -
> > sqr(a^2+b^2)/2 (NAO ENTENDI POR QUE VC FEZ r=
> p- hipotenusa, A PRINCIPIO 
> > PENSEI QUE p FOSSE O SEMI-PERIMETRO E ESTA
> EQUACAO TIVESSE ORIGEM NA FORMULA DE 
> > HERON ( S (AREA) = P*R) ,ONDE R EH O RAIO DA
> CIRCUNFERENCIA INSCRITA NO 
> > TRIANGULO, MAS ACHO QUE ME ENGANEI. O QUE FOI
> FEITO ?)
> >   O raio de C2 é a metade da hipotenusa: R=
> sqr(a^2+b^2)/2. Assim, a soma
> > dos comprimentos de C1 e C2 é igual a 2pi(R+
> r)= pi(k/2 - sqr(a^2+b^2)/2
> > +sqr(a^2+b^2)/2)= 2pi.k/2= pi.k
> > Ateh mais, 
> > Yuri
> > 
> > -- Mensagem original --
> > 
> > >Rodrigo Salcedo, eu aqui de novo!!!
> > >Consideremos um triangulo retangulo que
> simultaneamente esta circunscrito
> > >à 
> > >circunferencia C1 e inscrito à
> circunferencia C2 . Sabendo-se que a soma
> > >dos 
> > >comprimentos dos catetos do triangulo é K 
> cm, qual sera a soma dos 
> > >comprimentos destas duas circunferencias?
> > >
> > >[EMAIL PROTECTED]
> > >
> > 
> 
> 
>  

___
Conheça o novo Cadê? - Mais rápido, mais fácil e mais preciso.
Toda a web, 42 milhões de páginas brasileiras e nova busca por imagens!
http://www.cade.com.br
=
Instruções para entrar na lista, sair da lista e usar a lista em
http://www.mat.puc-rio.br/~nicolau/olimp/obm-l.html
=


Re: [obm-l] Re: [obm-l] geometria

2003-07-25 Por tôpico Faelccmm
Ola pessoal,

No enunciado foi dito que MB= 7 cm e NC= 4 cm, mas na resolucao eh dito que MB= 4 e NC = 7. Eh assim mesmo ?



Em uma mensagem de 24/7/2003 23:44:20 Hora padrão leste da Am. Sul, [EMAIL PROTECTED] escreveu:


Oi Rodrigo,
Seja ABC=B e ACB=C. Então NCP= 90- C/2. Como NP//BC, temos CNP=C. Logo,
NPC= 180- (C+ 90- C/2)= 90- C/2 => CNP isósceles => NP=NC=7 => MN+ MP= 7.

De modo análogo, BMP= 180- B e MBP= B/2 => BPM= B/2 => BMP isósceles =>
MP= MB= 4. 
Logo, MN= 7- MP= 7- 4 => MN= 3.
Ateh mais, 

  Yuri

-- Mensagem original --

>Num triangulo qualquer ABC, traçamos a bissetriz interna de B e a bissetriz
>
>externa de C. Pelo ponto P de concurso das bissetrizes, traçamos uma reta
>
>paralela ao lado BC , encontrando o lado AB no ponto M e AC no ponto N.

>Calcule o segmento MN sabendo que MB= 7 cm e NC= 4 cm
>
>[EMAIL PROTECTED]




Re: [obm-l] Re: [obm-l] geometria 2

2003-07-25 Por tôpico Faelccmm
Ola pessoal,

Direi minha duvida no corpo da mensagem. Para o Yuri ou quem souber.


Em uma mensagem de 24/7/2003 23:53:41 Hora padrão leste da Am. Sul, [EMAIL PROTECTED] escreveu:


Sejam a e b os comprimentos dos catetos, I o incentro de C1 e X o ponto
de tangência de C1 com AC. Então o raio de C é igual a AX, e eh esse valor
vale r= p- Hipotenusa= (a+b-Hipotenusa)/2= [a+b- sqr(a^2+b^2)]/2= k/2 -
sqr(a^2+b^2)/2 (NAO ENTENDI POR QUE VC FEZ r= p- hipotenusa, A PRINCIPIO PENSEI QUE p FOSSE O SEMI-PERIMETRO E ESTA EQUACAO TIVESSE ORIGEM NA FORMULA DE HERON ( S (AREA) = P*R) ,ONDE R EH O RAIO DA CIRCUNFERENCIA INSCRITA NO TRIANGULO, MAS ACHO QUE ME ENGANEI. O QUE FOI FEITO ?)
  O raio de C2 é a metade da hipotenusa: R= sqr(a^2+b^2)/2. Assim, a soma
dos comprimentos de C1 e C2 é igual a 2pi(R+ r)= pi(k/2 - sqr(a^2+b^2)/2
+sqr(a^2+b^2)/2)= 2pi.k/2= pi.k
Ateh mais, 
Yuri

-- Mensagem original --

>Rodrigo Salcedo, eu aqui de novo!!!
>Consideremos um triangulo retangulo que simultaneamente esta circunscrito
>à 
>circunferencia C1 e inscrito à circunferencia C2 . Sabendo-se que a soma
>dos 
>comprimentos dos catetos do triangulo é K  cm, qual sera a soma dos 
>comprimentos destas duas circunferencias?
>
>[EMAIL PROTECTED]
>





[obm-l] Re: [obm-l] geometria 2

2003-07-24 Por tôpico yurigomes
  Sejam a e b os comprimentos dos catetos, I o incentro de C1 e X o ponto
de tangência de C1 com AC. Então o raio de C é igual a AX, e eh esse valor
vale r= p- Hipotenusa= (a+b-Hipotenusa)/2= [a+b- sqr(a^2+b^2)]/2= k/2 -
sqr(a^2+b^2)/2 
  O raio de C2 é a metade da hipotenusa: R= sqr(a^2+b^2)/2. Assim, a soma
dos comprimentos de C1 e C2 é igual a 2pi(R+ r)= pi(k/2 - sqr(a^2+b^2)/2
+sqr(a^2+b^2)/2)= 2pi.k/2= pi.k
 Ateh mais, 
 Yuri

-- Mensagem original --

>Rodrigo Salcedo, eu aqui de novo!!!
>Consideremos um triangulo retangulo que simultaneamente esta circunscrito
>à 
>circunferencia C1 e inscrito à circunferencia C2 . Sabendo-se que a soma
>dos 
>comprimentos dos catetos do triangulo é K  cm, qual sera a soma dos 
>comprimentos destas duas circunferencias?
>
>[EMAIL PROTECTED]
>
>_
>MSN Hotmail, o maior webmail do Brasil.  http://www.hotmail.com
>
>=
>Instruções para entrar na lista, sair da lista e usar a lista em
>http://www.mat.puc-rio.br/~nicolau/olimp/obm-l.html
>=
>

[]'s, Yuri
ICQ: 64992515


--
Use o melhor sistema de busca da Internet
Radar UOL - http://www.radaruol.com.br



=
Instruções para entrar na lista, sair da lista e usar a lista em
http://www.mat.puc-rio.br/~nicolau/olimp/obm-l.html
=


[obm-l] Re: [obm-l] geometria

2003-07-24 Por tôpico yurigomes

Oi Rodrigo,
 Seja ABC=B e ACB=C. Então NCP= 90- C/2. Como NP//BC, temos CNP=C. Logo,
NPC= 180- (C+ 90- C/2)= 90- C/2 => CNP isósceles => NP=NC=7 => MN+ MP= 7.

 De modo análogo, BMP= 180- B e MBP= B/2 => BPM= B/2 => BMP isósceles =>
MP= MB= 4. 
 Logo, MN= 7- MP= 7- 4 => MN= 3.
 Ateh mais, 

  Yuri

-- Mensagem original --

>Num triangulo qualquer ABC, traçamos a bissetriz interna de B e a bissetriz
>
>externa de C. Pelo ponto P de concurso das bissetrizes, traçamos uma reta
>
>paralela ao lado BC , encontrando o lado AB no ponto M e AC no ponto N.

>Calcule o segmento MN sabendo que MB= 7 cm e NC= 4 cm
>
>[EMAIL PROTECTED]
>
>_
>MSN Hotmail, o maior webmail do Brasil.  http://www.hotmail.com
>
>=
>Instruções para entrar na lista, sair da lista e usar a lista em
>http://www.mat.puc-rio.br/~nicolau/olimp/obm-l.html
>=
>

[]'s, Yuri
ICQ: 64992515


--
Use o melhor sistema de busca da Internet
Radar UOL - http://www.radaruol.com.br



=
Instruções para entrar na lista, sair da lista e usar a lista em
http://www.mat.puc-rio.br/~nicolau/olimp/obm-l.html
=


[obm-l] RE: [obm-l] Re: [obm-l] geometria analítica

2003-03-12 Por tôpico leandro









Fael,

 

O Claudio apresentou uma
otima solucao e assim voce pode ver que tem diversas formas de resolver o
problema. A ilustracao que o Claudio se referiu e muito boa e as vezes num
vestibular onde o quesito tempo e super-importante voce pode sair na frente. 

 

Leandro. 

 

-Original Message-
From:
[EMAIL PROTECTED] [mailto:[EMAIL PROTECTED] On Behalf Of Cláudio (Prática)
Sent: Wednesday, March 12, 2003
12:13 PM
To: [EMAIL PROTECTED]
Subject: [obm-l] Re: [obm-l] geometria
analítica

 



Oi, Fael (e demais colegas):





 





Eu tenho sempre te aconselhado a
desenhar os gráficos e tentar visualizar a situação do problema antes de sair
escrevendo equações a torto e a direito. 





 





Estes dois problemas são uma boa
ilustração. Espero que o Morgado me apoie nesse ponto





 





 





(FUVEST) A reta y= mx (m>0) é
tangente à circunferência (x-4)^2 + y^2=4. Determine o seno do ângulo que a
reta forma com o eixo x. 





Circunferência: C(4,0); R = 2





Desenhe o gráfico e veja que o seno
desejado é igual a R/D, onde D = distância do centro à origem = 4





 





Logo, seno = 2/4 = 1/2.






resp: 1/2 





**






(U.E. Londrina) Sejam a circunferência (lambda) x^2 + y^2 + 2x - 4y +1=0 e o
ponto P(-1,4) pertencente a lambda. A equação da reta tangente lambda pelo
ponto P é: 





Complete os quadrados e reduza à
forma normal:





x^2 + 2x + 1 + y^2 - 4y + 4 = -1 + 1
+ 4 ==>





(x+1)^2 + (y-2)^2 = 2^2 ==>
C(-1,2); R = 2





 





A reta tangente por P é normal ao
raio CP. 





Mas C e P têm a mesma abscissa
==> 





CP é vertical ==> 





a tangente por P é horizontal ==>





Equação da tangente: y = 4.





 





resp: y=4 





 





Um abraço,





Claudio.





 







- Original Message - 





From: [EMAIL PROTECTED] 





To: [EMAIL PROTECTED]






Sent: Tuesday,
March 11, 2003 6:27 PM





Subject: [obm-l]
geometria analítica





 



Olá Morgado, 

Como resolver estas: 


(FUVEST) A reta y= mx (m>0) é tangente à circunferência (x-4)^2 + y^2=4.
Determine o seno do ângulo que a reta forma com o eixo x. 

resp: 1/2 

(U.E. Londrina) Sejam a circunferência (lambda) x^2 + y^2 + 2x - 4y +1=0 e o
ponto P(-1,4) pertencente a lambda. A equação da reta tangente lambda pelo
ponto P é: 

resp: y=4 










[obm-l] Re: [obm-l] geometria analítica

2003-03-12 Por tôpico Cláudio \(Prática\)



Oi, Fael (e demais colegas):
 
Eu tenho sempre te aconselhado a desenhar os 
gráficos e tentar visualizar a situação do problema antes de sair escrevendo 
equações a torto e a direito. 
 
Estes dois problemas são uma boa ilustração. Espero 
que o Morgado me apoie nesse ponto
 
 
(FUVEST) A reta y= mx (m>0) é tangente à 
circunferência (x-4)^2 + y^2=4. Determine o seno do ângulo que a reta forma com 
o eixo x. 
Circunferência: C(4,0); R = 2
Desenhe o gráfico e veja que o seno desejado é igual a R/D, onde D = 
distância do centro à origem = 4
 
Logo, seno = 2/4 = 1/2.
resp: 1/2 
**
(U.E. Londrina) Sejam a circunferência (lambda) x^2 + y^2 + 2x - 4y 
+1=0 e o ponto P(-1,4) pertencente a lambda. A equação da reta tangente lambda 
pelo ponto P é: 
Complete os quadrados e reduza à forma normal:
x^2 + 2x + 1 + y^2 - 4y + 4 = -1 + 1 + 4 ==>
(x+1)^2 + (y-2)^2 = 2^2 ==> C(-1,2); R = 2
 
A reta tangente por P é normal ao raio CP. 
Mas C e P têm a mesma abscissa ==> 
CP é vertical ==> 
a tangente por P é horizontal ==>
Equação da tangente: y = 4.
 
resp: y=4 
 
Um abraço,
Claudio.
 

  - Original Message - 
  From: 
  [EMAIL PROTECTED] 
  
  To: [EMAIL PROTECTED] 
  Sent: Tuesday, March 11, 2003 6:27 
  PM
  Subject: [obm-l] geometria 
analítica
  Olá Morgado, Como resolver estas: (FUVEST) A reta 
  y= mx (m>0) é tangente à circunferência (x-4)^2 + y^2=4. Determine o seno 
  do ângulo que a reta forma com o eixo x. resp: 1/2 (U.E. 
  Londrina) Sejam a circunferência (lambda) x^2 + y^2 + 2x - 4y +1=0 e o ponto 
  P(-1,4) pertencente a lambda. A equação da reta tangente lambda pelo ponto P 
  é: resp: y=4 


[obm-l] RE: [obm-l] geometria analítica

2003-03-11 Por tôpico Artur Costa Steiner

Olá Morgado, 

Como resolver estas:

Mesmo não sendo o Morgado, vou tentar ajudar
 


(FUVEST) A reta y= mx (m>0) é tangente à circunferência (x-4)^2 + y^2=4.
Determine o seno do ângulo que a reta forma com o eixo x. 

resp: 1/2 

Por ser tangente à circunferencia, a reta intercepta-a em um, e apenas um,
ponto. Logo, a equação (x-4)^2 + (mx)^2 =4 tem uma, e apenas uma, raiz real.
Esta equação é equivalente a (1+ m^2)x^2 -8x + 12 =0, e apresentará uma
única solução real se, e somente se, seu discriminante for zero. Logo, 64 -
48 (1+ m^2) = 16 -48m^2 = 0, cuja solução é m = + ou - 1/raiz(3). Como, por
hipótese, m>0, apenas a solução positiva interessa. A reta, portanto, forma
com o eixo dos x um ângulo a  cuja tangente é 1/raiz(3). Segue-se que
sec(a)^2 = 1+ tan(a)^2 = 1+ 1/3 = 4/3. Logo, cos(a)^2 = 3/4 (o ângulo é do
primeiro quadrante) e sen(a)^2 = 1/4. Finalmente, concluímos que sen(a) =
1/2.  


(U.E. Londrina) Sejam a circunferência (lambda) x^2 + y^2 + 2x - 4y +1=0 e o
ponto P(-1,4) pertencente a lambda. A equação da reta tangente lambda pelo
ponto P é: 
resp: y=4

Calculando-se implicitamente a derivada de y com rel. a x, y', temos pela
regra da cadeia que 2x + 2y y' +2 -4 y'=0 -> y'= (-2x -2)/(2y -4) =
(x+1)/(2-y), y<>2. No ponto dado, temos que y' = 0, logo a tangente é
horizontal. E como esta tangente intercepta a circunferência em um ponto de
ordenada 4 segue-se que sua equação é y =4. 
Vc poderioa chegar rapidamente a esta mesma conclusão observado que a
equação da circunferência pode ser escrita como (x+1)^2 + (y-2)^2 = 4, a
qual tem centro em (-1, 2) e raio 2. Logo, (-1, 4) é ponto mais "alto" da
intersecção com a circunferência da vertical de abcissa -1, a qual passa
pelo centro de lambda. Como a tangente é perpendicular a esta vertical, a
conclusão é imediata. 

Um abraço
Artur 
<>

Re: [obm-l] RE: [obm-l] geometria analítica

2003-03-11 Por tôpico A. C. Morgado




Um errinho de conta!
Onde esta mod(m) = 1/2 deveria estar mod(m) = 1/sqrt(3).
Daih, seguir-se-ia
tg^2(alfa) + 1
= sec^2(alfa) => sec^2(alfa) = 4/3 => sen^2(alfa) = 1/4. => sen(alfa)
= 1/2. 


leandro wrote:
   
  
   
  
  
  

  Fael,
  
   
  
  No
numero 1) eu substitui o valor y=mx na equacao da circunferencia e dai voce
encontra a seguinte equacao do 2o grau 
  
   
  
  (m^2+1)x^2
– 8x + 12 = 0. Como foi dito que m > 0, entao temos que a intersecao da
reta com a circunferencia deve produzir somente 1 ponto, portanto, fazendo
o discriminante da equacao encontrada igual a zero a gente encontra 
  
   
  
  64
– 4(12)(m^2+1) = 0 => mod(m) = ½ = > m=1/2 (pois m > 0). Agora,
m e o coeficiente angular da reta , ou seja, e a tangente do angulo que estamos
querendo encontrar. O valor do seno do angulo pode ser encontrado pela formula
  
  
   
  
  tg^2(alfa) + 1
= sec^2(alfa) => sec^2(alfa) = 5/4 => sen^2(alfa) = 1/5. => sen(alfa)
= 1/sqr(5). 
  
   
  
  Eu
nao encontrei esse resultado ½ que voce me forneceu. Sera que errei em algum
lugar ? 
  
   
  
  Leandro.
  
  
   
  
  -Original
Message-
 From: [EMAIL PROTECTED]
[mailto:[EMAIL PROTECTED]] On Behalf Of [EMAIL PROTECTED]
 Sent: Tuesday, March  11, 2003 1:27 PM
 To: [EMAIL PROTECTED]
 Subject: [obm-l] geometria 
analítica
  
   
  
  Olá Morgado,
  
 
 Como resolver estas: 
 
 
 (FUVEST) A reta y= mx (m>0) é tangente à circunferência (x-4)^2 + y^2=4. 
Determine o seno do ângulo que a reta forma com o eixo x. 
 
 resp: 1/2 
 
 (U.E. Londrina) Sejam a circunferência (lambda) x^2 + y^2 + 2x - 4y +1=0
e o ponto P(-1,4) pertencente a lambda. A equação da reta tangente lambda
pelo ponto P é: 
 
 resp: y=4 
  
  





[obm-l] RE: [obm-l] geometria analítica

2003-03-11 Por tôpico leandro








Fael,

 

No numero 1) eu substitui
o valor y=mx na equacao da circunferencia e dai voce encontra a seguinte
equacao do 2o grau 

 

(m^2+1)x^2 – 8x +
12 = 0. Como foi dito que m > 0, entao temos que a intersecao da reta com a
circunferencia deve produzir somente 1 ponto, portanto, fazendo o discriminante
da equacao encontrada igual a zero a gente encontra 

 

64 – 4(12)(m^2+1) =
0 => mod(m) = ½ = > m=1/2 (pois m > 0). Agora, m e o coeficiente
angular da reta , ou seja, e a tangente do angulo que estamos querendo
encontrar. O valor do seno do angulo pode ser encontrado pela formula 

 

tg^2(alfa) + 1 = sec^2(alfa) => sec^2(alfa)
= 5/4 => sen^2(alfa) = 1/5. => sen(alfa) = 1/sqr(5). 

 

Eu nao encontrei esse
resultado ½ que voce me forneceu. Sera que errei em algum lugar ? 

 

Leandro. 

 

-Original Message-
From:
[EMAIL PROTECTED] [mailto:[EMAIL PROTECTED] On Behalf Of [EMAIL PROTECTED]
Sent: Tuesday, March
 11, 2003 1:27 PM
To: [EMAIL PROTECTED]
Subject: [obm-l] geometria
analítica

 

Olá Morgado, 

Como resolver estas: 


(FUVEST) A reta y= mx (m>0) é tangente à circunferência (x-4)^2 + y^2=4.
Determine o seno do ângulo que a reta forma com o eixo x. 

resp: 1/2 

(U.E. Londrina) Sejam a circunferência (lambda) x^2 + y^2 + 2x - 4y +1=0 e o
ponto P(-1,4) pertencente a lambda. A equação da reta tangente lambda pelo
ponto P é: 

resp: y=4 








[obm-l] Re: [obm-l] geometria analítica (circunferências)

2003-03-11 Por tôpico Cláudio \(Prática\)



Oi, Fael:
 
(UFRS) A circunferência de centro (10, -6), tangente ao eixo dos y, 
intercepta o eixo dos x nos pontos de abcissas: 
Como ela é tangente ao eixo y, a distância do centro a este eixo (dada pelo 
valor absoluto da abscissa do centro) é igual ao raio ==> raio = 10.
 
Equação: (x - 10)^2 + (y + 6)^2 = 10^2.
 
Intercepta o eixo x ==> y = 0 ==> 
(x-10)^2 + 6^2 = 100 ==> 
(x-10)^2 = 64 ==> 
x-10 = 8  ou x-10 = - 8 ==> 
x = 18 ou x = 2
resp: 2 e 18 
*
(U.C. SALVADOR) A reta r, de equação y= 2x +1, e a circunferência C, de 
equação x^2 + y^2=1 interceptam-se nos pontos A e B. A medida do segmento AB é: 

Substitua y = 2x+1 na equação da circunferência a fim de achar a(s) 
abscissa(s) do(s) ponto(s) de interseção
 
Em seguida, substitua o(s) valor(es) de x achado(s) acima em y = 2x + 
1 para determinar o valor da(s) ordenada(s) correspondente(s).
 
Agora, é só usar a fórmula da distância entre dois pontos.
resp: 4*raiz(5)/5 
***
(PUCCAMP) Considere as circunferências (lambda_1): x^2 + y^2 - 8x - 4y 
+ 15=0 e (lambda_2): x^2 + y^2 + 4x + 2y - 75=0; concluímos que: 
Normalize a equações, completando os quadrados. Você terá:
 
Lambda 1: x^2 - 8x + 16 + y^2 - 4y + 4  = -15 + 16 + 4 ==>
(x - 4)^2 + (y - 2)^2 = 5  ==> C1(4,2); R1 = raiz(5)
 
Lambda 2: x^2 + 4x + 4 + y^2 + 2y + 1 = 75 + 4 + 1 ==>
(x + 2)^2 + (y + 1)^2 = 80 ==> C2(-2,-1); R2 = raiz(80) = 
4*raiz(5).
 
Distância entre os centros = raiz[(4+2)^2 + (2+1)^2] = raiz(36+9) = 
3*raiz(5) = R2 - R1 ==> 
Lambda 1 e Lambda 2 são tangentes internamente.
resp:  (lambda_1) e (lambda_2) se tangenciam-se internamente 

*
Um abraço,
Claudio.


[obm-l] Re: [obm-l] geometria analítica

2003-03-09 Por tôpico Henrique P. Sant'Anna Branco
- Original Message -
From: [EMAIL PROTECTED]
To: [EMAIL PROTECTED]
Sent: Sunday, March 09, 2003 6:09 PM
Subject: [obm-l] geometria analítica

> (UFPA) O maior valor inteiro de p para que a equação x^2 + y^2 -6x + 4y
+p=0 represente uma circunferência é:

Completando os quadrados, temos:
(x^2 - 6*x + 9) + p - 9+ (y^2 + 4y + 4) - 4 = 0
(x - 3)^2 + (y + 2)^2 = -p + 13

Temos que o sqrt(-p+13) é o raio da circunferência centrada em (3, -2).
Sabemos que a função raiz quadrada é definida em [0, +infinito), portanto, o
maior valor para que a sqrt(-p+13) fosse definida nos reais seria 13, mas
isso nos daria 0 para o raio da circunferência. Portanto, o menor valor
inteiro é 12 => sqrt(-12+13) = sqrt(1) = 1.

A resolução do outro eu mando depois.

Abraços,
Henrique.

=
Instruções para entrar na lista, sair da lista e usar a lista em
http://www.mat.puc-rio.br/~nicolau/olimp/obm-l.html
O administrador desta lista é <[EMAIL PROTECTED]>
=


  1   2   >